by GF Educators, Inc. Seventh Grade Reading Book … Sample Books/7th Reading Practice...Literary...

163
Step Up to the TEKS by GF Educators, Inc. Seventh Grade Reading Book Teacher Edition Copyright © 2016 GF Educators, Inc. www.StepUpTEKS.com Teacher: Classroom: Written by Kathleen Newman Joyce Tucker Charlotte Anderson Edited by Charity Riley-Rose Kris Fuller GF Educators wishes to thank the many people who contributed to the development of this book with unwavering effort, investing countless hours in an effort to create the highest quality product available. Sample Do Not Duplicate

Transcript of by GF Educators, Inc. Seventh Grade Reading Book … Sample Books/7th Reading Practice...Literary...

Page 1: by GF Educators, Inc. Seventh Grade Reading Book … Sample Books/7th Reading Practice...Literary Text/Drama: Full length passage #2 ..... 54-58 Sample Reporting Category 2 Answer

Step Up to the TEKS by GF Educators, Inc.

Seventh Grade Reading Book

Teacher Edition

Copyright © 2016GF Educators, Inc.

w w w . S t e p U pT E K S . c o m

Teacher:

Classroom:

Written by

Kathleen Newman

Joyce Tucker

Charlotte Anderson

Edited by

Charity Riley-Rose

Kris Fuller

GF Educators wishes to thank the many people who contributed to the development of this book with unwavering effort, investing countless hours in an effort to create the highest quality product available.

Sam

ple

Do Not

Dup

licat

e

Page 2: by GF Educators, Inc. Seventh Grade Reading Book … Sample Books/7th Reading Practice...Literary Text/Drama: Full length passage #2 ..... 54-58 Sample Reporting Category 2 Answer

Step Up to the TEKS

7th Reading Practice Bookby GF Educators, Inc.

Table of ContentsReporting Category 1 Understanding and Analysis Across Genres Understanding the Skill ..............................................................................................1-3 Category 1/Vocabulary: Full length passage #1 ..........................................................4-6 Category 1/Vocabulary: Full length passage #2 ........................................................7-10 Reporting Category 1 Answer Key .............................................................................. 11

Reporting Category 2 Understanding and Analysis of Literary Texts Understanding the Skill ............................................................................................... 13 Literary Text/Sensory Language: Full length passage #1 .......................................14-16 Understanding the Skill ............................................................................................... 17 Literary Text/Media Literacy: Full length passage #1 ............................................18-19 Understanding the Skill ............................................................................................... 20 Literary Text/Fiction: Point of View: Full length passage #1 ..................................21-22 Literary Text/Fiction: Point of View: Full length passage #2 ..................................23-26 Literary Text/Fiction: Point of View: Full length passage #3 ..................................27-30 Understanding the Skill ............................................................................................... 31 LiteraryText/LiteraryNonfiction:Fulllengthpassage#1 .....................................32-33 LiteraryText/LiteraryNonfiction:Fulllengthpassage#2 .....................................34-39 LiteraryText/LiteraryNonfiction:Fulllengthpassage#3 .....................................40-43 Literary Text/Poetry: Full length passage #1...........................................................44-45 Literary Text/Poetry: Full length passage #2...........................................................46-47 Literary Text/Poetry: Full length passage #3................................................................ 48 Understanding the Skill ............................................................................................... 49 Literary Text/Drama: Full length passage #1 ..........................................................50-53 Literary Text/Drama: Full length passage #2 ..........................................................54-58 Reporting Category 2 Answer Key .........................................................................59-61Sa

mpl

e

Do Not

Dup

licat

e

Page 3: by GF Educators, Inc. Seventh Grade Reading Book … Sample Books/7th Reading Practice...Literary Text/Drama: Full length passage #2 ..... 54-58 Sample Reporting Category 2 Answer

Reporting Category 3 Understanding and Analysis of Informational Texts Informational Text/Media Literacy: Full length passage #1 ...................................63-64 Informational Text/Media Literacy: Full length passage #2 ...................................65-67 Informational Text/Procedural: Full length passage #1..........................................68-71 Informational Text/Procedural: Full length passage #2..........................................72-75 Informational Text/Procedural: Full length passage #3..........................................76-79 Informational Text/Expository: Full length passage #1 ..........................................80-83 Informational Text/Expository: Full length passage #2 ..........................................84-87 Informational Text/Expository: Full length passage #3 ..........................................88-92 Informational Text/Expository: Full length passage #4 ..........................................93-96 Understanding the Skill ..........................................................................................97-98 Informational Text/Persuasive: Full length passage #1 .........................................99-101 Informational Text/Persuasive: Full length passage #2 .......................................102-105 Informational Text/Persuasive: Full length passage #3 .......................................106-110 Reporting Category 3 Answer Key .....................................................................111-112

Reporting Categories 1-3 Paired Passages Understanding the Skill ............................................................................................. 114 Literary Text/Paired Passages: Set #1 ..................................................................115-122 Literary Text/Paired Passages: Set #2 ..................................................................123-128 Literary Text/Paired Passages: Set #3 ..................................................................129-136 Literary Text/Paired Passages: Set #4 ..................................................................137-142 Literary Text/Paired Passages: Set #5 ..................................................................143-150 Literary Text/Paired Passages: Set #6 ..................................................................151-158 Paired Passages Answer Key ...................................................................................... 159

Sam

ple

Do Not

Dup

licat

e

Page 4: by GF Educators, Inc. Seventh Grade Reading Book … Sample Books/7th Reading Practice...Literary Text/Drama: Full length passage #2 ..... 54-58 Sample Reporting Category 2 Answer

Reporting Category 1

Understanding and Analysis Across Genres

Vocabulary

Sam

ple

Do Not

Dup

licat

e

Page 5: by GF Educators, Inc. Seventh Grade Reading Book … Sample Books/7th Reading Practice...Literary Text/Drama: Full length passage #2 ..... 54-58 Sample Reporting Category 2 Answer

1 © 2016 GF Educators, Inc.Seventh Grade Reading

Vocabulary/Roots & Affixes Readiness 7.2A Understanding the Skill

Greek and Latin Roots and Affixes

Knowing the meanings of common prefixes and suffixes, together with the meanings of common base and root words, will give you the tools for unlocking the meanings of hundreds of words.

PrefixesPrefixes are small, meaningful letter groups that are added before a base word or root. They change the meaning of the word. The change in meaning can be complete as in mis + behave = misbehave, or it can serve to clarify or make the word more specific as in tele + scope = telescope.

Common PrefixesPrefix Meaning Examplescent- hundred century, centurion,

centennialdis- opposite disgust, distaste, dislike,

dismiss, distrustin- not indirect, insane,

ingratitudemis- make a mistake misbehave, misjudge,

misleadnon- not nonstop, nonsense,

nonfictionover- too much overland, overprotect,

overgrownpre- before prehistoric, precede,

precautionre- again reassure, rebuild, repay,

remarkun- not unexpected, unknown,

unpavedSam

ple

Do Not

Dup

licat

e

Page 6: by GF Educators, Inc. Seventh Grade Reading Book … Sample Books/7th Reading Practice...Literary Text/Drama: Full length passage #2 ..... 54-58 Sample Reporting Category 2 Answer

2Seventh Grade Reading © 2016 GF Educators, Inc.

Vocabulary/Roots & Affixes Readiness 7.2A Understanding the Skill

Suffixes and Inflected Endings

Suffixes and inflected endings are letter groups that are added to the end of a base word or root. Suffixes often add information to the meaning of the word. The following charts list the suffixes by meaning according to their part of speech.

Noun SuffixesSuffix/Ending Meaning Examples

-s, -es more than one bodyguards, dairies, revoluntionaries

-er, -or one who does something freighter, piper, traitor, inspector

-ist one who pianist, chemist, socialist, florist

-tion state or quality exploration, explanation, protection

-ty state or quality probability, possibility, electricity

-ment state or quality attachment, government, judgment

-ship state or quality championship, scholarship

-ness state or quality loneliness, cleverness, tardiness

Verb SuffixesSuffix/Ending Meaning Examples

-d, -ed before; in the past challenged, destroyed, mumbled, protected,

-ing continuing accepting, defending, prancing

-en to make flatten, forgiven, slacken, heightenSa

mpl

e

Do Not

Dup

licat

e

Page 7: by GF Educators, Inc. Seventh Grade Reading Book … Sample Books/7th Reading Practice...Literary Text/Drama: Full length passage #2 ..... 54-58 Sample Reporting Category 2 Answer

3 © 2016 GF Educators, Inc.Seventh Grade Reading

Vocabulary/Roots & Affixes Readiness 7.2A Understanding the Skill

Adjective SuffixesSuffix/Ending Meaning Examples

-ful full of delightful, successful, thoughtful

-ous full of adventurous, continuous, luscious

-less without priceless, fearless, senseless

-er, -est more, most shabbier, crazier, dumbest

-y state or quality crispy, curvy, curly, drafty, fuzzy

-ish like; relating to babyish, childish, greenish, skittish

-al like; relating to alphabetical, horizontal, international

-able inclined to dependable, fashionable, durable

-en, -ed, -ing verb as adjective blended, convincing, defeated

Adverb SuffixesSuffix/Ending Meaning Examples

-d, -ed before; in the past challenged, destroyed, mumbled, protected,

-ing continuing accepting, defending, prancing

-en to make flatten, forgiven, slacken, heightenSa

mpl

e

Do Not

Dup

licat

e

Page 8: by GF Educators, Inc. Seventh Grade Reading Book … Sample Books/7th Reading Practice...Literary Text/Drama: Full length passage #2 ..... 54-58 Sample Reporting Category 2 Answer

4Seventh Grade Reading © 2016 GF Educators, Inc.

1 Helen Keller was born in Alabama in 1880. At the age of 19 months, Helen contracted a fever, which left her deaf and blind. Helen spent the next several years frustrated and angry because of her inability to communicate. Just before she turned seven, Helen’s parents hired Anne Sullivan to be her tutor. Using a method known as finger spelling, Miss Sullivan was able to teach Helen to express her thoughts and needs.

2 Helen quickly learned to read and write. Although Helen began to speak by the time she was 10 years old, it took more than 25 years for her to learn to speak so that others could understand her. At age 20, Helen entered Radcliffe College. Miss Sullivan was at her side, finger spelling every word of every textbook into Helen’s hand. Within four years’ time, Helen graduated with honors from Radcliffe. By this time, she had mastered several forms of communication including touch -lip reading, Braille, speech, typing, and finger spelling.

3 After college, Keller was intent to learn more about the world and how she could improve the lives of others. Before long, Helen discovered her life's calling of helping blind and deaf -blind people. Helen was an active member of the American Federation for the Blind. She spoke with lawmakers in state and national government, pleading with

Category 1/Vocabulary: Full-Length Passage #1 Mixed Practice

Question 1In paragraph 1, the word contracted means —

D caught

The best definition would be that Helen caught a fever.

Question 2The Latin expressus, mean-ing "clearly presented, distinct, articulated precisely," helps the reader understand that express in paragraph 1 means to —

D make known

By learning to finger spell, Helen was now able to make known or express her thoughts and needs.

Question 3Which definition best matches the meaning of the word communication as it is used in paragraph 2?

A Definition 1 (act of communicating)

Touch lip reading, Braille, speech, typing, and finger spelling are ways for Helen to communicate.

Question 4Which synonym is the best replacement for the word intent in paragraph 3?

A Determined

Helen wanted to learn more about the world and what she could do to improve it; this shows that Helen had determination.

Sam

ple

Do Not

Dup

licat

e

Page 9: by GF Educators, Inc. Seventh Grade Reading Book … Sample Books/7th Reading Practice...Literary Text/Drama: Full length passage #2 ..... 54-58 Sample Reporting Category 2 Answer

5 © 2016 GF Educators, Inc.Seventh Grade Reading

them to improve the welfare of blind people. In her work with the American Foundation of Overseas Blind, Helen traveled to 35 countries on five continents to lecture and to visit areas with a high rate of blindness. She wrote numerous articles and books. Miss Keller met every U.S. President from Grover Cleveland to Lyndon Johnson. She spent her lifetime focusing the world’s attention on the problems of the blind and the need to take measures to prevent blindness.

4 Helen was an intelligent and ambitious woman who accomplished great things in her life. Despite her own challenges, Helen devoted her life to helping others. She is known around the world as an amazing woman who faced overwhelming odds. Through her many speeches and public appearances, she brought encouragement and inspiration to millions of people. She was a powerful example of how determination and hard work can help a person overcome adversity.

Question 8Which sentence from the selection states an opinion?

D Helen was an intelligent and ambitious woman who accomplished great things in her life.

Whether the things Helen accomplished are "great" is a matter of opinion.

Choices A, B, and C are all facts that can be proven.

5 Which sentence from the selection helps explain why Helen was successful?

A Helen quickly learned to read and write. Wrong AnsWer: Just because Helen quickly learned to read and write did not mean she would be successful.

B By this time, she had masterd several forms of communication including touch-lip reading, Braille, speech, typing, and finger spelling.

Wrong AnsWer: Mastering these forms of communication did not make Helen successful.

C Despite her own challenges, Helen devoted her life to helping others.

Wrong AnsWer: Helping others alone does not make one successful.

D She was a powerful example of how determination and hard work can help a person overcome adversity.

right AnsWer: The fact that Helen was determined and worked hard to overcome her physical issues shows that she is successful.

Exercise Reasoning

Sam

ple

Do Not

Dup

licat

e

Page 10: by GF Educators, Inc. Seventh Grade Reading Book … Sample Books/7th Reading Practice...Literary Text/Drama: Full length passage #2 ..... 54-58 Sample Reporting Category 2 Answer

6Seventh Grade Reading © 2016 GF Educators, Inc.

Exercise Reasoning6 Read these sentences from the selection.

Using a method known as finger spelling, Miss Sullivan was able to teach Helen to express her thoughts and needs. (paragraph 1)

Miss Sullivan was at her side, finger spelling every word of every textbook into Helen’s hand. (paragraph 2)

What does the author communicate with these sentences?A Anne Sullivan’s devotion to Helen right AnsWer:

Sullivan worked tirelessly to help Helen be successful, which shows her devotion.

B Helen’s strength and determination Wrong AnsWer: These sentences are representative of what Sullivan did for Helen.

C The importance of a college education Wrong AnsWer: There is nothing about college is in these sentences.

D The obstacles faced by blind college students

Wrong AnsWer: These sentences do not mention college.

7 What sentence best expresses the main idea of the selection?

A Although she was stricken with an illness that rendered her deaf and blind, Helen Keller overcame obstacles in her life and helped others.

right AnsWer: This sentence lets the reader know that Helen started out with physical issues, but she went to college and helped others, which is the main idea of this selection.

B With the help of Anne Sullivan, Helen Keller used a variety of techniques to communicate with others.

Wrong AnsWer: Her communication was only one important detail in the selection, not the main idea.

C Helen Keller devoted her life to helping disabled people in the U.S. and all over the world.

Wrong AnsWer: Her helping other disabled people is only one detail in the selection.

D It is much better to prevent blindness than it is to help people who are blind overcome their condition.

Wrong AnsWer: Preventing blindness was not mentioned in the selection.

9 The author organized this selection by —

A stating facts about Helen and her education and telling how she helped others

right AnsWer: The first two paragraphs provide facts about Helen and her education, and then the last two paragraphs tell how she was able to take what she learned and help others.

B describing the role of Helen’s schooling in her future life

Wrong AnsWer: There is more than just information about Helen's schooling.

C comparing Helen’s life to Sullivan’s life Wrong AnsWer: The author is not attempting to compare the two women.

D explaining why it is important for a hearing person to learn sign language

Wrong AnsWer: Although sign language is important, this is not a persuasive selection about learning sign language.

Sam

ple

Do Not

Dup

licat

e

Page 11: by GF Educators, Inc. Seventh Grade Reading Book … Sample Books/7th Reading Practice...Literary Text/Drama: Full length passage #2 ..... 54-58 Sample Reporting Category 2 Answer

7 © 2016 GF Educators, Inc.Seventh Grade Reading

Reptiles and Amphibians1 Reptiles include alligators, crocodiles, snakes,

tortoises, lizards, and turtles. Frogs, toads,

newts, and salamanders are in the amphibian

family. Together, there are over 10,400 species of

these animals. Reptiles and amphibians are often

studied together because they are alike in many

ways. There are also several characteristics that

distinguish reptiles from amphibians.

2 Both reptiles and amphibians are

ectothermic, or cold- blooded animals. Their

body temperature depends on the temperature

of their surroundings. In cold weather, these

animals tend to be cold and lazy, while they are

warm and active in warm weather. Reptiles can

often be found basking in the sun on a hot rock

in an attempt to get warm. Figure 1 shows how a

lizard’s body temperature is closely related to its

behavior during the course of a day. Both reptiles

and amphibians are very efficient animals, and

they are often able to live in environments that

are inaccessible to other species. For example,

many reptiles are able to live in the driest of

deserts.

Category 1/Vocabulary: Full-Length Passage #2 Mixed PracticeQuestion 1

Which synonym is the best replacement for distinguish in paragraph 1?

D Number 4 (differentiate)

The sentence prior lets the reader know that reptiles and amphibians are alike, then this sentence states that they have things that distinguish them. Using the two sentences, the reader can conclude that differentiate would best replace distinguish - seeing the differences.

Question 3The Latin root accessibilis, meaning “to approach,” helps the reader understand that the word inaccessible in paragraph 2 means —

C difficult to get to

Other species find it difficult to get to the environments where reptiles and amphibians live because they cannot survive in those environments.

Question 2Which words from paragraph 2 best help the reader understand the meaning of ectothermic?

A body temperature depends on the temperature of their surroundings

The word “or” in the passage lets the reader know that a word is going to be defined. The sentence that follows then defines cold-blooded, leading the reader to understand that is the meaning of ectothermic.

Question 4The main message of paragraph 2 is that —

A reptiles and amphibians are cold -blooded animals

The first sentence of paragraph 2 is the topic sentence and tells the main idea.Sa

mpl

e

Do Not

Dup

licat

e

Page 12: by GF Educators, Inc. Seventh Grade Reading Book … Sample Books/7th Reading Practice...Literary Text/Drama: Full length passage #2 ..... 54-58 Sample Reporting Category 2 Answer

8Seventh Grade Reading © 2016 GF Educators, Inc.

C GF Educators, Inc.Figure 1

6

69

76

83

90

97

9 12 3 6 9

LIZARD

’S B

OD

Y TE

MPE

RAT

URE

°F

RELATIONSHIP BETWEEN LIZARD’S BODY TEMPERATURE AND TIME OF DAY

TIME OF DAYa.m. p.m.

RETREATS TOSHADE OF TREE

REACHES THE GROUND

COMES OUT OF TREE

3 Amphibians and reptiles and also similar

in body structure. Both types of animals are

vertebrates. They have a bony, internal skeleton

built around a central backbone. Amphibians and

reptiles are alike in the way they obtain their

food. Species of both types of animals either

graze on plants or capture prey. In these ways,

amphibians are similar to reptiles.

4 While reptiles and amphibians have many

similarities, they are also different in many

ways. The most notable difference is in their

relationship to the water. Most reptiles spend

their entire lives on land, completely independent

from water for breeding and moving from place

to place. The reptiles’ young are usually quite

similar in appearance to their parents. Reptile

eggs have thick, hard shells, which hold moisture.

This allows the young to develop within the shell

on dry land. Reptiles usually make nests out of

plant material in damp places, or they dig holes

in the soil in which to lay their eggs. On the

other hand, amphibians are able to move freely

on land, but they need the water to breed. Most

amphibians return to the water to mate and lay

Sam

ple

Do Not

Dup

licat

e

Page 13: by GF Educators, Inc. Seventh Grade Reading Book … Sample Books/7th Reading Practice...Literary Text/Drama: Full length passage #2 ..... 54-58 Sample Reporting Category 2 Answer

9 © 2016 GF Educators, Inc.Seventh Grade Reading

Question 5Which of the following glossary definitions best fits the meaning of the word larvae as used in paragraph 4?

D An immature form of a reptile

The reptiles are young, and they are immature.

eggs. Amphibian eggs do not have a waterproof

outer covering, so they must lay their eggs in

water or damp places and then leave them. Most

amphibians have fish like young called tadpoles

that do not resemble the parents. These larvae

breathe to absorb oxygen from the water. As the

tadpoles develop, they undergo a process known

as metamorphosis when their bodies change

shape. They also change their diet and lifestyle

while developing into adult amphibians.

5 Another important difference between reptiles

and amphibians is the skin coverings of these

animals. Amphibians have soft, smooth skin,

which can be penetrated by water. Since their

skin is not waterproof, amphibians are generally

found in moist places. They also have glands,

which produce a slimy mucus. This mucus helps

keep the skin moist and prevents the amphibian’s

body from drying out. These are the major ways

in which reptiles and amphibians differ from each

other.

6 While reptiles and amphibians have many

similar characteristics, they are also different

in many ways. Because of these differences,

scientists have placed these animals in separate

categories in the scientific classification of

animals.

Question 7 Which definition represents the meaning of penetrated as used in paragraph 5?

A Definition 1 (to pierce or pass into or through)

The water can pass into or through the skin of the amphibians.

Question 6The Greek root meta, meaning “change,” helps the reader understand that the word metamorphosis in paragraph 4 means —

D transformations in the body of a young amphibian

Their bodies develop and change shapes; they are transforming.

Sam

ple

Do Not

Dup

licat

e

Page 14: by GF Educators, Inc. Seventh Grade Reading Book … Sample Books/7th Reading Practice...Literary Text/Drama: Full length passage #2 ..... 54-58 Sample Reporting Category 2 Answer

10Seventh Grade Reading © 2016 GF Educators, Inc.

8 The author wrote this passage to —

A explain the difficulties encountered by the amphibian population

Wrong AnsWer: The selection is not just about amphibians.

B identify the five major types of reptiles and describe them

Wrong AnsWer: The selection is not just about reptiles.

C compare and contrast reptiles and amphibians

right AnsWer: The selection does tell how the animals are the same and different.

D classify animals as to whether they are reptiles or amphibians

Wrong AnsWer: It is not about classifying these animals.

Exercise Reasoning

9 From this passage, the reader can tell that reptiles and amphibians —

A are more alike than they are different Wrong AnsWer: This is not supported by evidence in the passage. The reader can not know if they are more alike than different.

B are dangerous animals Wrong AnsWer: Nothing is mentioned about them being dangerous.

C both need water to breed Wrong AnsWer: Reptiles do not need water to breed.

D reproduce by laying eggs right AnsWerBoth amphibians and reptiles lay eggs.

10 What information about lizards is represented in Figure 1 but is not stated in the written selection?

A The times the lizard moves in and out of the sun

right AnsWer: The passage does not mention times of the day; it just refers to the course of the day.

B How body temperature is affected by the rocks on which they climb

Wrong AnsWer: This is briefly discussed in the article but not in the chart.

C The sun rising and the body temperature Wrong AnsWer: The time the sun rises is not included on the chart.

D Where the lizard makes its home Wrong AnsWer: Nothing is shown on the chart about the animals' home.

Sam

ple

Do Not

Dup

licat

e

Page 15: by GF Educators, Inc. Seventh Grade Reading Book … Sample Books/7th Reading Practice...Literary Text/Drama: Full length passage #2 ..... 54-58 Sample Reporting Category 2 Answer

11 © 2016 GF Educators, Inc.Seventh Grade Reading

Seventh Grade Reading Answer Key

Vocabulary/Context Clues Page 11 A2 B3 C4 C

Page 21 A2 A3 B4 C

Page 31 D2 C3 A4 B

Page 41 C2 D3 B

Vocabulary/Roots&Affixes Page 81 D2 A3 A4 D

Page 91 B2 A3 C4 A

Page 101 D2 C3 B4 D

Page 111 opened his other hand2 the clay did not look like a horse's head3 D

Vocabulary/Dictionary, Glossary, ThesaurusPage 121 B2 D

Page 131 A2 B

Page 141 A2 D3 C4 D

Page 151 C2 A

Page 161 C2 A

Page 171 B2 B3 C

Category 1/VocabularyFull-Length Passage #1Pages 19-201 D2 D3 A4 A5 D6 A7 A8 D9 A

Category 1/VocabularyFull-Length Passage #1Pages 19-201 D2 A3 C4 A5 D6 D7 A8 C9 D10 A

*All page numbers refer to the Student Edition.

Reporting Category 1 Warm-Ups and Selections

Sam

ple

Do Not

Dup

licat

e

Page 16: by GF Educators, Inc. Seventh Grade Reading Book … Sample Books/7th Reading Practice...Literary Text/Drama: Full length passage #2 ..... 54-58 Sample Reporting Category 2 Answer

Reporting Category 2

Understanding and Analysis of

Literary Texts

Sam

ple

Do Not

Dup

licat

e

Page 17: by GF Educators, Inc. Seventh Grade Reading Book … Sample Books/7th Reading Practice...Literary Text/Drama: Full length passage #2 ..... 54-58 Sample Reporting Category 2 Answer

13 © 2016 GF Educators, Inc.Seventh Grade Reading

Theme

The theme is the message of the story. It is what the author wants you to take away from the story. The theme is often a moral or a lesson.

Don’t confuse theme with subject. The theme is the idea the writer conveys about the subject.

Theme can be stated or implied, so the reader must search for it throughout the text.

The theme is the story’s main observation, so it should never ignore major parts of the story.

Many stories have more than one theme.

When determining theme:• Think about the story’s title. Does it suggest a message?• Look at the characters and consider how they have changed.• How is the conflict resolved?

A story’s setting is the time and place in which the events of the story occur. • Time includes the time of day, a season, or a period of time in the past, present or

future.• Place is the geographic location – rural or city, house or school, Earth or space, etc.

How does the setting affect the theme?• It provides the surrounding in which the story occurs.• It causes conflict.• It influences a character’s thoughts, feelings, and actions.• It sets the mood of the story.• It affects how the reader interprets the story.

Literary Text/Theme Supporting 7.3 Understanding the Skill

Sam

ple

Do Not

Dup

licat

e

Page 18: by GF Educators, Inc. Seventh Grade Reading Book … Sample Books/7th Reading Practice...Literary Text/Drama: Full length passage #2 ..... 54-58 Sample Reporting Category 2 Answer

14Seventh Grade Reading © 2016 GF Educators, Inc.

How Troubles Came Into the World

1 A very long time ago, in the Golden Age, everyone was good and happy. The two brothers, Prometheus and Epimetheus, lived in those wonderful days.

2 After stealing the fire for man, Prometheus, knowing that Jupiter would be angry, decided to go away for a time on a distant journey; but before he went, he warned Epimetheus not to receive any gifts from the gods.

3 One day, after Prometheus had been gone for some time, Mercury came to the cottage of Epimetheus, leading by the hand a beautiful young woman, whose name was Pandora. She had a wreath of partly opened rosebuds on her head, a number of delicate gold chains twisted lightly around her neck, and wore a filmy veil which fell nearly to the hem of her tunic. Mercury presented her to Epimetheus, saying the gods had sent this gift that he might not be lonesome.

4 Pandora had such a lovely face that Epimetheus could not help believing that the gods had sent her to him in good faith. So he paid no heed to the warning of Prometheus, but took Pandora into his cottage, and found that the days passed much more quickly and pleasantly when she was with him.

5 Soon, the gods sent Epimetheus another gift. This was a heavy box, which the satyrs brought to the cottage, with directions that it was not to be opened. Epimetheus let it stand in a corner of his cottage; for by this time he had begun to think that the caution of Prometheus about receiving gifts from the gods was altogether unnecessary.

6 Often, Epimetheus was away all day, hunting

Literary Text/Sensory Language: Full-Length Passage #1 Mixed Practice

Question 1This information helps the reader understand that the word delicate in paragraph 3 means —

B fragile

Being twisted lightly shows that the chains are fragile and dainty.

Sam

ple

Do Not

Dup

licat

e

Page 19: by GF Educators, Inc. Seventh Grade Reading Book … Sample Books/7th Reading Practice...Literary Text/Drama: Full length passage #2 ..... 54-58 Sample Reporting Category 2 Answer

15 © 2016 GF Educators, Inc.Seventh Grade Reading

or fishing or gathering grapes from the wild vines that grew along the riverbanks. On such days, Pandora had nothing to do but to wonder what was in the mysterious box. One day her curiosity was so great that she lifted the lid a very little way and peeped in. The result was like lifting the cover off of a beehive. Out rushed a great swarm of little winged creatures, and before Pandora knew what had happened, she was stung. She dropped the lid and ran out of the cottage, screaming. Epimetheus, who was just coming in at the door, was well stung, too. The little winged creatures that Pandora had let out of the box were Troubles, the first that had ever been seen in the world. They soon flew about and spread themselves everywhere, pinching and stinging whenever they got the chance.

Source: Famous Greek Myths by Lillian S. Hyde, D.C. Heath and Co., 1904.

2 The author provides a vivid description of Pandora in paragraph 3 to —

A show Epimetheus’s need for something beautiful in his life

right AnsWer: Epimetheus was lonely and needed someone to be a part of his life, and Pandora’s beauty helped with the gods’ plan.

B explain why Epimetheus is lonely all the time right AnsWer: Pandora's beauty is not the reason why Epimetheus is lonely.

C suggest that Mercury would be jealous of Epimetheus

Wrong AnsWer: Nothing in the passage leads the reader to infer that Mercury would be jealous.

D illustrate that beauty is important in mythology

Wrong AnsWer: Beauty may be important in mythology, but that cannot be inferred.

3 Read this sentence from paragraph 6.

The result was like lifting the cover off of a beehive.

The author uses figurative language in this sentence most likely to —

A demonstrate the magnitude of the fury about to be unleashed

right AnsWer: When a cover is lifted off of a beehive, the bees fly about furiously.

B exaggerate the sound of bees in a hive Wrong AnsWer: No sound is noted in the simile.

C explain why Pandora needed help from Epimetheus to open the box

Wrong AnsWer: Pandora did not need help opening the box.

D identify the contents of the box Wrong AnsWer: Bees were not found in the box.

Exercise Reasoning

Sam

ple

Do Not

Dup

licat

e

Page 20: by GF Educators, Inc. Seventh Grade Reading Book … Sample Books/7th Reading Practice...Literary Text/Drama: Full length passage #2 ..... 54-58 Sample Reporting Category 2 Answer

16Seventh Grade Reading © 2016 GF Educators, Inc.

4 What is the central message of the story?

A Choose your partner carefully. Wrong AnsWer: Choosing a partner is not what is learned in the selection.

B Two heads are better than one. Wrong AnsWer: Prometheus and Epimetheus did not work together.

C Watch out for advice from others. Wrong AnsWer: The advice was good advice and should have been followed.

D Curiosity can often get you into trouble. right AnsWer: Pandora’s curiosity about what was in the box caused the problems.

Exercise Reasoning

5 Which sentence foreshadows that Epimetheus will have troubles?

A but before he went, he warned Epimetheus not to receive any gifts from the gods.

right AnsWer: Epimetheus was warned…foreshadowing that something would happen.

B Soon, the gods sent Epimetheus another gift.

Wrong AnsWer: There wasn’t a problem with the first gift, so this would not foreshadow a problem.

C They soon flew about and spread themselves everywhere, pinching and stinging whenever they got the chance

Wrong AnsWer: This just explains what happened when Pandora opened the box.

D One day her curiosity was so great that she lifted the lid a very little way and peeped in.

Wrong AnsWer: This has more to do with the theme and not foreshadowing.

6 What lesson does Epimetheus learn after so much time away from Prometheus?

A Time makes the heart grow fonder. Wrong AnsWer: The brothers being apart did not cause them to become closer.

B Don’t let a pretty face take an old friend’s place.

Wrong AnsWer: Epimetheus did not allow Pandora to take his brother's place.

C Good advice knows no time limits. right AnsWer: The advice Prometheus gave did stand firm after he had been gone for some time.

D The gods are willing to wait to present you a gift.

Wrong AnsWer: Epimetheus did not learn anything from the gifts from the gods.

Sam

ple

Do Not

Dup

licat

e

Page 21: by GF Educators, Inc. Seventh Grade Reading Book … Sample Books/7th Reading Practice...Literary Text/Drama: Full length passage #2 ..... 54-58 Sample Reporting Category 2 Answer

17 © 2016 GF Educators, Inc.Seventh Grade Reading

Media Literacy

Media is a powerful influence in our lives. We receive messages from many sources. A few of these are television, the Internet, radio, magazines, newspapers, books, billboards, advertisements, video games and music. All of these influence the way we see the world.

It is important to remember that media products are created by people who make decisions about what to include, what to exclude, and how to present information. The creator’s point of view, opinions, and biases determine the way in which the media is presented. Because of this, media products are never 100% accurate reflections of the real world. This is why we need to be media literate people who have the ability to analyze and evaluate media.

People who are media literate are able to:

• Think critically when encountering media• Interpret messages in various forms of media• Understand the ways in which media messages affect society and

culture• Identify strategies used by media to influence and inform audiences• Recognize what the media maker wants you to believe or do

Here are some questions to ask when you encounter media:

• Who created this media?• What is the purpose of the media?• What beliefs or assumptions are reflected in the content?• How is the message of this media communicated?• How does this media make you feel? • Does this media help someone make money?

Literary Text/Media Literacy Supporting 7.13A, 7.13C Understanding the Skill

Sam

ple

Do Not

Dup

licat

e

Page 22: by GF Educators, Inc. Seventh Grade Reading Book … Sample Books/7th Reading Practice...Literary Text/Drama: Full length passage #2 ..... 54-58 Sample Reporting Category 2 Answer

18Seventh Grade Reading © 2016 GF Educators, Inc.

1 When entrepreneur Ruth Handler presented her new

Barbie doll to the world, her creation’s tote bag was

chockfull of messages. Ruth’s Barbie was of average

height, fair-skinned, and in complete graceful command

of an hourglass figure that few could attain without the

aid of a personal trainer as well as some time spent at

a ballet bar. Barbie’s original 1959 advertisement

most strongly presents Barbie as beautiful.

There she stands with a bouquet of roses as the jingle

rings, “Barbie’s small and so petite”. The clothes look

astounding and are obviously very important to Barbie.

When she dances, it says that she casts a spell.

2 Barbie had an array of accessories, but consumers

don’t get the impression that she works for the money

for any of them. Barbie has “all the gadgets gals adore.”

But do all women love accessorizing? The closing lines

ring, “Someday I’m going to be exactly like you…until

that day, I will make believe that I am you.”

3 In 1967, a new Barbie was released. This version

was blonde, had real lashes, and a stronger face shape

that elicited the feel of one who frequently visits the

beach. The biggest emphasis of the new doll was that

she twisted at the waist. But do you know that the

advertisers found it prudent to have a child ask, “But

what do I do with my old Barbie?” The advertisement

narrator responds, “Just take your old Barbie down to

the toy store and trade it in for a new Barbie for half the

cost! That’s right. Take your old Barbie and $1.50 down

to the store and get your new Barbie right away! But

don’t delay: offer only good while supplies last!” Then

the ad shows a preadolescent girl swiftly skipping to

the local store and smiling as she inherits the new doll

without any visible exchange of money.

Literary Text/Media Literacy Full-Length Passage #1 Mixed Practice

Question 1Which meaning of array is used in paragraph 2?

C Definition 3 (collection)

Barbie has all the gadgets or a collection of accessories.

Question 2The Latin adolescens, meaning “to grow up,” helps the reader understand that preadolescent in paragraph 3 means —

B preteen

The “pre” in preadolescent means "before." An adolescent is someone growing up. So a preteen is the time period before a child becomes an adolescent or teen.

Question 6Which sentence best supports the idea that advertisers influence what consumers buy?

A Barbie’s original 1959 advertisement most strongly presents Barbie as beautiful.

Barbie being beautiful would influence young girls to want to buy her.

Sam

ple

Do Not

Dup

licat

e

Page 23: by GF Educators, Inc. Seventh Grade Reading Book … Sample Books/7th Reading Practice...Literary Text/Drama: Full length passage #2 ..... 54-58 Sample Reporting Category 2 Answer

19 © 2016 GF Educators, Inc.Seventh Grade Reading

3 The author wrote this article most likely to —

A compare the original Barbie doll to the 1967 doll

Wrong AnsWer: The article is not comparing the two versions of the doll.

B recount the beginning of the Barbie doll right AnsWer: The article does tell about the first two dolls and the advertising of them.

C describe the modern Barbie doll and its influence

Wrong AnsWer: The dolls are described, but the influence on girls is only discussed after the first doll is described.

D explain why they had to create a new Barbie doll

Wrong AnsWer: There is nothing about why the new doll was created.

Exercise Reasoning

4 The author most likely wants the reader to —

A go out and buy a Barbie doll Wrong AnsWer: This article is not persuasive in nature.

B become an informed consumer Wrong AnsWer: Because the dolls are so old, you would not be informed about the newer dolls.

C learn about the first two models of the Barbie doll

right AnsWer: This article does provide information about the first two Barbie dolls.

D understand why the Barbie doll was created Wrong AnsWer: No explanation about why Barbie was created is mentioned in the article.

5 Read these sentences from the selection.Barbie’s original 1959 advertisement most strongly presents Barbie as beautiful. (paragraph 1)Then the ad shows a preadolescent girl swiftly skipping to the local store and smiling as she inherits the new doll without any visible exchange of money. (paragraph 3)

What idea does the author communicate with these sentences?

A Local stores know that the Barbie doll is the best toy for young girls.

Wrong AnsWer: There is nothing in the text that gives evidence that Barbie is the best toy.

B The Barbie doll includes many features that make them appealing.

Wrong AnsWer: The paragraph 3 line does not include anything about Barbie's features.

C Barbie dolls have become presents that do not cost anything.

Wrong AnsWer: Barbie did cost something when originally purchased. Trading in was not free.

D Advertisers win over young girls to prompt them to buy the Barbie doll.

right AnsWer: Advertising was a big part of the original Barbie, and it was used to encourage young girls to want her.

Sam

ple

Do Not

Dup

licat

e

Page 24: by GF Educators, Inc. Seventh Grade Reading Book … Sample Books/7th Reading Practice...Literary Text/Drama: Full length passage #2 ..... 54-58 Sample Reporting Category 2 Answer

20Seventh Grade Reading © 2016 GF Educators, Inc.

Literary Text/Fiction: Point of View Supporting 7.6C Understanding the Skill

Point of view is the perspective from which the events of the story are related

by the author to the reader. By choosing a specific point of view, the author

decides how to let readers “see” and “hear” the story.

The person telling the story is known as the narrator. In determining point of

view, only the narrator’s voice matters. Always focus on the narration, not the

dialogue.

When determining the point of view of a selection, the reader must ask,

“Who is telling this story?”

“How are they telling it?”

“Are the character’s thoughts and actions revealed?”

Author's Role Signal Characteristics

First Person

The narrator participates as a character in the story.

• I• Me• My• Mine

• We• Us• Our• Ours

• Subjective• Personal• Sometimes unreliable

Second Person

The narrator speaks directly to the reader.

• You• Your

• Least common form• Used to give directions

or advice

Third Person

The narrator is not part of the story.

• He• She• They• Them• Characters' names

• Objective• Impersonal• More reliable

Objective The narrator is a neutral observer. No character’s thoughts or feelings are revealed.

Only the characters’ actions and dialogue are narrated.

Limited The narrator tells the thoughts and feelings of one character.

The storyteller is an observer rather than a participant in the story.

Omniscient The narrator reveals the thoughts and feelings of two or more characters.

The storyteller is all-knowing and possesses information that may be unknown to some or all of the characters.Sa

mpl

e

Do Not

Dup

licat

e

Page 25: by GF Educators, Inc. Seventh Grade Reading Book … Sample Books/7th Reading Practice...Literary Text/Drama: Full length passage #2 ..... 54-58 Sample Reporting Category 2 Answer

21 © 2016 GF Educators, Inc.Seventh Grade Reading

Literary Text/Fiction: Full-Length Passage #1 Mixed Practice

1 It was one of those gray days with steady drizzle. Anna rushed into the house, “Can you drive me to the mall?”

2 “Sure,” I said, picking up a magazine and shuffling its pages. Dead silence! Peeking over the magazine, I caught hold of Anna’s eyes. They could have burned a hole right through the magazine to me!

3 “No, I mean now — I need to go now.”

4 “Okay,” I said. “I didn’t know I needed to jump right up to get you there! Let me find my keys.”

5 Anna sighed as I looked around my chest of drawers. “Got ‘em.”

6 “Okay,” she said nodding her head, “Let’s get going.”

7 As we drove, the urgency of Anna’s voice broke through my daydreaming. “There’s a parking space right by the door.” Sure enough, there was.

8 I wheeled my truck into the convenient spot only to spy that familiar blue and white sign: handicapped. I hit the brakes. “Can’t do it,” I said to Anna. “We aren’t allowed to park in a handicapped space.”

9 “Oh, Greg, come on. Just park in there. We’re in a hurry,” Anna whined.

10 “No way!” I retorted. “Who gets the ticket—you or me? Did you see the fine prices? No. We can find another place.”

11 Wouldn’t you know, just as I backed out, another car pulled into the same parking place? Of course Anna made it her business to evaluate the driver’s physical status. “That lady doesn’t look handicapped!” she said as a woman gets out of the car and walked inside. “We could have parked there!”

12 “Actually, Anna,” I corrected, “whether or not that woman is handicapped will have no bearing whatsoever on my getting a parking fine for not being handicapped. And how would you like it if you, as a handicapped person driving behind us, needed this place only to watch the two of us stroll inside? Shouldn’t we treat people the way we would want to be treated?”

Question 2The figurative language in these sentences emphasizes that Greg thinks Anna is —

A furious

Burning a hole right through me indicates someone that is angry. Note Anna's demanding attitude throughout the story.

Question 3The Latin word urgentem, meaning "to press hard, urge," helps the reader understand that urgency in paragraph 7 means —C insistence

The Latin meaning "pressing hard" helps the reader to infer that Anna's tone of voice was insistent - urgent.

Question 1The point of view from which this story is told gives the reader special insight into —

A Greg’s feelings about doing the right thing

Because Greg is telling the story, we can get insight into how he felt.This is first-person point of view.Sa

mpl

e

Do Not

Dup

licat

e

Page 26: by GF Educators, Inc. Seventh Grade Reading Book … Sample Books/7th Reading Practice...Literary Text/Drama: Full length passage #2 ..... 54-58 Sample Reporting Category 2 Answer

22Seventh Grade Reading © 2016 GF Educators, Inc.

13 Once the car was parked, Anna and I headed up to the mall entrance. As I opened the door, the lady from the handicapped-parking place was heading out – with a man in a wheelchair. Anna hung her head. It was the first time since we began our trip to the mall that Anna was quiet!

Question 4In paragraph 13, the description of Anna suggests that Greg —

D believes that Anna is truly sorry for what she had said earlier

By hanging her head, Anna is showing that she is sorry for saying anything about the woman not being handicapped.

6 How does the setting contribute to Greg’s conflict in the story?

A It causes him feel insecure about dealing with Anna.

Wrong AnsWer: The setting has no effect on how Greg is feeling.

B It permits him to share information he is reading in the magazine.

Wrong AnsWer: Greg did not share information from the magazine.

C It pushes him to have a serious conversation with Anna.

right AnsWer: The setting at the mall with the handicapped parking allowed Greg to talk to Anna about handicapped people and parking.

D It gives him an opportunity to learn about handicapped people.

Wrong AnsWer: Greg was not the one who learned something from the mall parking lot.

5 Which sentence from the selection best states the main message of this story?

A "I didn’t know I needed to jump right up to get you there!"

Wrong AnsWer: There is nothing thematic about this sentence.

B Of course Anna made it her business to evaluate the driver’s physical status.

Wrong AnsWer: Anna checking out what someone looks like is not providing a message.

C "Shouldn’t we treat people the way we would want to be treated?"

right AnsWer: Anna not only treated her brother poorly, but she did not have kind things to say about the woman, making this the most likely message.

D It was the first time since we began our trip to the mall that Anna was quiet!

Wrong AnsWer: This sentence does show Ann’s regret but does not provide any kind of lesson learned.

Exercise Reasoning

Sam

ple

Do Not

Dup

licat

e

Page 27: by GF Educators, Inc. Seventh Grade Reading Book … Sample Books/7th Reading Practice...Literary Text/Drama: Full length passage #2 ..... 54-58 Sample Reporting Category 2 Answer

23 © 2016 GF Educators, Inc.Seventh Grade Reading

Literary Text/Fiction: Full-Length Passage #2 Mixed Practice

Read the selection and choose the best answer to each question.

The Invisible Man by H. G. WellsAn excerpt from Chapter V

1 The facts of the burglary at the vicarage came to us chiefly through the medium of the vicar and his wife. Mrs. Bunting, it seems, woke up suddenly in the stillness that comes before the dawn, with the strong impression that the door of their bedroom had opened and closed. She did not arouse her husband at first, but sat up in bed listening. She then distinctly heard the pad, pad, pad of bare feet coming out of the adjoining dressing-room and walking along the passage towards the staircase. As soon as she felt assured of this, she aroused the Rev. Mr. Bunting as quietly as possible. He did not strike a light, but putting on his spectacles, her dressing-gown and his bath slippers, he went out on the landing to listen. He heard quite distinctly a fumbling going on at his study desk down-stairs, and then a violent sneeze.

2 At that he returned to his bedroom, armed himself with the most obvious weapon, the poker, and descended the staircase as noiselessly as possible. Mrs. Bunting came out on the landing.

3 The hour was about four, and the ultimate darkness of the night was past. There was a faint shimmer of light in the hall, but the study doorway yawned impenetrably black. Everything was still except the faint creaking of the stairs under Mr. Bunting's tread, and the slight movements in the study. Then something snapped, the drawer was opened, and there was a rustle of papers. Then came an imprecation, and a match was struck and the study was flooded with yellow light. Mr. Bunting was now in the hall, and through the crack of the door he could see the desk and the open drawer and a candle burning on the desk. But the robber he could not see. He stood there in the hall undecided what to do, and Mrs. Bunting, her face

Question 2The Latin root penetrabillis, meaning “able to be pierced,” helps the reader understand that the word impenetrably in paragraph 3 means —

B unable to penetrate

The affix “im” means unable, so the definition that fits best is unable to penetrate – the doorway was completely black.

Question 1The details in paragraph 2 help the reader infer that —

A Rev. Mr. Bunting feared the intruder

He decided to get a weapon to protect himself and his wife which illustrates his fear.

Question 7Which sentence best shows that Mrs. Bunting was cautious about revealing that an intruder was in the house?

A She did not arouse her husband at first, but sat up in bed listening.

By just listening in bed at first, she is careful to make sure that there is someone in the house before rousing her husband. When she does wake him up, she does it quietly to make sure the intruder does not hear them.

Sam

ple

Do Not

Dup

licat

e

Page 28: by GF Educators, Inc. Seventh Grade Reading Book … Sample Books/7th Reading Practice...Literary Text/Drama: Full length passage #2 ..... 54-58 Sample Reporting Category 2 Answer

24Seventh Grade Reading © 2016 GF Educators, Inc.

white and intent, crept slowly downstairs after him. One thing kept Mr. Bunting's courage; the persuasion that this burglar was a resident in the village.

4 They heard the chink of money, and realized that the robber had found the housekeeping reserve of gold—two pounds ten in half Sovereigns altogether. At that sound Mr. Bunting was nerved to abrupt action. Gripping the poker firmly, he rushed into the room, closely followed by Mrs. Bunting. "Surrender!" cried Mr. Bunting, fiercely, and then stooped amazed. Apparently the room was perfectly empty.

5 Yet their conviction that they had, that very moment, heard somebody moving in the room had amounted to a certainty. For half a minute, perhaps, they stood gaping, then Mrs. Bunting went across the room and looked behind the screen, while Mr. Bunting, by a kindred impulse, peered under the desk. Then Mrs. Bunting turned back the window-curtains, and Mr. Bunting looked up the chimney and probed it with the poker. Then Mrs. Bunting scrutinized the wastepaper basket and Mr. Bunting opened the lid of the coal-scuttle. Then they came to a stop and stood with eyes interrogating each other.

6 "I could have sworn—" said Mr. Bunting.

7 "The candle!" said Mr. Bunting. "Who lit the candle?"

8 "The drawer!" said Mrs. Bunting. "And the money's gone!"

9 She went hastily to the doorway.

10 "Of all the strange occurrences—"

11 There was a violent sneeze in the passage. They rushed out, and as they did so the kitchen door slammed. "Bring the candle," said Mr. Bunting, and led the way. They both heard a sound of bolts being hastily shot back.

12 As he opened the kitchen door he saw through the scullery that the back door was just opening, and the faint light of early dawn displayed the dark masses of the garden beyond. He is certain that nothing went out of the door. It opened, stood open for a moment, and then closed with a slam. As it did so, the candle Mrs.

Question 3The author uses this sentence to show that Mr. Bunting —

C had enough courage to face the robber

The word nerved in this sentence means that he found the courage (nerve) he needed to go after the robber.

Question 4In paragraph 5, the description of the search for the robber emphasizes the Buntings’ —

D annoyance that they were wrong about him still being in the room

They looked all over the room and did not find the robber, and then even looked accusingly at each other. They were looking for evidence, not ignoring it. They were neither excited or disappointed in this paragraph.

Question 5The author uses short sentences in paragraphs 6 – 10 to help create a feeling of —

B frustration

They felt frustrated because they could not find the robber anywhere. There is no indication of disaster or doom or feeling of anger. Sa

mpl

e

Do Not

Dup

licat

e

Page 29: by GF Educators, Inc. Seventh Grade Reading Book … Sample Books/7th Reading Practice...Literary Text/Drama: Full length passage #2 ..... 54-58 Sample Reporting Category 2 Answer

25 © 2016 GF Educators, Inc.Seventh Grade Reading

Bunting was carrying from the study flickered and flared. It was a minute or more before they entered the kitchen.

13 The place was empty. They refastened the back door, examined the kitchen, pantry, and scullery thoroughly, and at last went down into the cellar. There was not a soul to be found in the house, search as they would.

14 Daylight found the vicar and his wife, a quaintly-costumed little couple, still marveling about on their own ground floor by the unnecessary light of a guttering candle.

Source: The Invisible Man by H. G. Wells, 1897 (public domain)

6 The point of view from which the story is told gives the reader insight into —

A both Mr. and Mrs. Buntings’ fears right AnsWer: Third-person POV lets the reader experience both of the Buntings' fears.

B the reason the robber went to the Buntings’ home

Wrong AnsWer: Although we can assume the robber needed money, we are not given any idea that that is the reason he robbed the Buntings.

C Mrs. Bunting’s suspicion about Mr. Bunting Wrong AnsWer: Mrs. Bunting only appeared to look at Mr. Bunting with interrogating eyes…as he did too.

D the Buntings’ feelings about each other Wrong AnsWer: Nothing in the passage indicates how the Buntings feel about each other – inferring or stated outright.

8 Read the sentences from the selection.He stood there in the hall undecided what to do, and Mrs. Bunting, her face white and intent, crept slowly downstairs after him. (paragraph 3)

Gripping the poker firmly, he rushed into the room, closely followed by Mrs. Bunting. (paragraph 4)

Based on these sentences, the reader can infer that Mrs. Bunting —

A doesn’t trust Mr. Bunting Wrong AnsWer: There is no evidence of a lack of trust.

B wants to keep an eye on Mr. Bunting Wrong AnsWer: Although she is following Mr. Bunting, there is nothing to infer her keeping an eye on him.

C has poor eyesight and needs Mr. Bunting to go first

Wrong AnsWer: Her eyesight is never mentioned in the passage.

D is scared and prefers to follow Mr. Bunting right AnsWer: The fact that Mrs. Bunting's face was white, she was creeping, and Mr. Bunting had a poker in his hand lead the reader to infer that she was fearful and wanted him to go first.

Exercise Reasoning

Sam

ple

Do Not

Dup

licat

e

Page 30: by GF Educators, Inc. Seventh Grade Reading Book … Sample Books/7th Reading Practice...Literary Text/Drama: Full length passage #2 ..... 54-58 Sample Reporting Category 2 Answer

26Seventh Grade Reading © 2016 GF Educators, Inc.

9 Which sentence best summarizes the excerpt?

A At the Bunting house, a robber makes a mess while trying to steal their money.

Wrong AnsWer: Nothing is mentioned about the house being a mess.

B When the Buntings hear a noise in their house, they realize a robber is stealing from them, but they are unable to see him.

right AnsWer: This summary mentions the main idea and supporting details.

C When she hears a noise, Mrs. Bunting decides not to wake her husband and instead tries to find the source on her own.

Wrong AnsWer: Mrs. Bunting does not investigate on her own.

D A robber decides to steal food at the Bunting house and has an allergic reaction.

Wrong AnsWer: Although the robber sneezes, we do not know that he has an allergic reaction.

Exercise Reasoning

10 The darkness of the night contributes to the story plot by —

A creating a sense of fear in the characters right AnsWer: The time of day adds to the mood of the story and the plot because it makes the main characters fearful.

B making the burglar's actions invisible Wrong AnsWer: Darkness does not have the ability to make things invisible.

C providing the burglar a place to hide in the garden

Wrong AnsWer: By then it was dawn, and there was faint light in the garden.

D leading Mrs. Bunting to plead with Mr. Bunting not to go outside

Wrong AnsWer: She does not plead with him not to go outside.

11 A theme expressed in this story centers on —

A two heads being better than one Wrong AnsWer: Although the Buntings worked together, that is not the theme.

B never giving up Wrong AnsWer: The Buntings did continue looking for the robber, but this is not the theme.

C thinking things through before you take action

right AnsWer: The Buntings should have thought things through before they went after a burglar.

D appreciating what you have Wrong AnsWer: The Buntings did appreciate what they had.

Sam

ple

Do Not

Dup

licat

e

Page 31: by GF Educators, Inc. Seventh Grade Reading Book … Sample Books/7th Reading Practice...Literary Text/Drama: Full length passage #2 ..... 54-58 Sample Reporting Category 2 Answer

27 © 2016 GF Educators, Inc.Seventh Grade Reading

Literary Text/Fiction: Full-Length Passage #3 Mixed Practice

Read the selection and choose the best answer to each question.

Toads and DiamondsBy Andrew Lange

1 Once upon a time there lived a widow with two daughters. The elder was often mistaken for her mother, so like her was she both in nature and in looks; parent and child being so disagreeable and arrogant that no one could live with them.

2 The younger girl, who took after her father in the gentleness and sweetness of her disposition, was also one of the prettiest girls imaginable. The mother doted on the elder daughter naturally enough, since she resembled her so closely—and disliked the younger one as intensely. She made the latter live in the kitchen and work hard from morning till night.

3 One of the poor child’s many duties was to go twice a day and draw water from a spring a good half-mile away, bringing it back in a large pitcher. One day when she was at the spring an old woman came up and begged for a drink.

4 “Why, certainly, good mother,” the pretty lass replied. Rinsing her pitcher, she drew some water from the cleanest part of the spring and handed it to the dame, lifting up the jug so that she might drink the more easily.

5 Now this old woman was a fairy, who had taken the form of a poor village dame to see just how far the girl’s good nature would go. “You are so pretty,” she said, when she had finished drinking, “and so polite, that I am determined to bestow a gift upon you. This is the boon I grant you: with every word that you utter there shall fall from your mouth either a flower or a precious stone."

6 When the girl reached home she was scolded by her mother for being so long in coming back from the spring.

Question 1This information helps the reader understand that the word arrogant in paragraph 1 means —

D self-centered

Knowing that a person who "claims for oneself" is arrogant along with substituting the answer choices helps the reader conclude that self-centered is the best choice for this sentence.

Question 3The mother’s interaction with the younger daughter contributes to the plot by—

C exposing how far the mother will go to hurt the younger girl

The mother was willing to do anything to hurt the girl and this is necessary to move the plot along.

Question 6The young girl’s actions in this excerpt reveal that she —

A desires to help people in need

She knew the woman was old and could not do this on her own, and she wanted to help her. There is no evidence for choices B or C, and the girl had no idea the woman was a fairy.Sa

mpl

e

Do Not

Dup

licat

e

Page 32: by GF Educators, Inc. Seventh Grade Reading Book … Sample Books/7th Reading Practice...Literary Text/Drama: Full length passage #2 ..... 54-58 Sample Reporting Category 2 Answer

28Seventh Grade Reading © 2016 GF Educators, Inc.

7 “I am sorry to have been so long, Mother,” said the poor child.

8 As she spoke these words there fell from her mouth three roses, three pearls, and three diamonds.

9 “What’s this?” cried her mother; “did I see pearls and diamonds dropping out of your mouth? What does this mean, dear daughter?” (This was the first time she had ever addressed her daughter affectionately.)

10 The poor child told a simple tale of what had happened, and in speaking scattered diamonds right and left.

11 “Really,” said her mother, “I must send my own child there. Come here, Fanchon; look what comes out of your sister’s mouth whenever she speaks! Wouldn’t you like to be able to do the same? All you have to do is to go and draw some water at the spring, and when a poor woman asks you for a drink, give it her very nicely.”

12 ‘‘Oh, indeed!” replied the ill-mannered girl; ‘‘don’t you wish you may see me going there!”

13 “I tell you that you are to go,” said her mother, “and to go this instant.”

14 Very sulkily the girl went out taking with her the best silver flagon in the house. No sooner had she reached the spring than she saw a lady, magnificently attired, who came towards her from the forest, and asked for a drink. This was the same fairy who had appeared to her sister, masquerading now as a princess in order to see how far this girl’s ill nature would carry her.

15 “Do you think I have come here just to get you a drink?” said the loutish damsel, arrogantly. “I suppose you think I brought a silver flagon here specially for that purpose it’s so likely, isn’t it? Drink from the spring, if you want to!”

16 “You are not very polite,” said the fairy, displaying no sign of anger. “Well, in return for your lack of courtesy I decree that for every word you utter a snake or a toad shall drop out of your mouth.”

17 The moment her mother caught sight of her coming back she cried out, “Well, daughter?”

Question 7The younger daughter’s response in paragraph 7 reveals that she treats her mother with—

B respect

By apologizing for being late, she is showing respect.

Question 2What does masquerading mean in paragraph 14?

A Pretending

The fairy was pretending to be princess.

Sam

ple

Do Not

Dup

licat

e

Page 33: by GF Educators, Inc. Seventh Grade Reading Book … Sample Books/7th Reading Practice...Literary Text/Drama: Full length passage #2 ..... 54-58 Sample Reporting Category 2 Answer

29 © 2016 GF Educators, Inc.Seventh Grade Reading

18 “Well, mother?” replied the rude girl. As she spoke a viper and a toad were spat out of her mouth.

19 “Gracious heavens!” cried her mother, “what do I see? Her sister is the cause of this, and I will make her pay for it!” Off she ran to thrash the poor child, but the latter fled away and hid in the forest nearby. The king’s son met her on his way home from hunting, and noticing how pretty she was inquired what she was doing all alone, and what she was weeping about.

20 “Alas, sir,” she cried, “my mother has driven me from home!”

21 As she spoke the prince saw four or five pearls and as many diamonds fall from her mouth. He begged her to tell him how this came about, and she told him the whole story.

22 The king’s son fell in love with her, and reflecting that such a gift as had been bestowed upon her was worth more than any dowry, which another maiden might bring him, he took her to the palace of his royal father, and there married her.

23 As for the sister, she made herself so hateful that even her mother drove her out of the house. Nowhere could the wretched girl find anyone who would take her in, and at last she lay down in the forest and died.

Source: The Blue Fairy Book by Andrew Lange, London: Longmans, Green, 1889.

4 How does the setting influence the plot of the story?

A The incidents at the spring highlighted aspects of the girls’ characters.

right AnsWer: When the younger girl goes to the spring, she helps an old woman (really a fairy), revealing that she is a kind, helpful girl. When the older sister goes to the spring, she meets the same woman (fairy) who is now a princess. She refuses to help the princess, showing her nasty attitude.

B The circumstances in the forest contribute to the conflict the young girl experiences.

Wrong AnsWer: By the time the younger daughter enters the forest, the conflict was already in place.

C The prince’s appearance in the forest changes the story from a moral to a romantic tale.

Wrong AnsWer: While the prince's appearance introduces an element of love, it does not change the story from a moral to a love story.

D The older sister’s attitude would have been different if she had remained at home.

Wrong AnsWer: The older daughter's attitude was bad all the time. It was not influenced by the setting.

Exercise Reasoning

Sam

ple

Do Not

Dup

licat

e

Page 34: by GF Educators, Inc. Seventh Grade Reading Book … Sample Books/7th Reading Practice...Literary Text/Drama: Full length passage #2 ..... 54-58 Sample Reporting Category 2 Answer

30Seventh Grade Reading © 2016 GF Educators, Inc.

5 The point of view from which the story is told gives the reader insight into —

A the mother’s sadness at the death of her husband

Wrong AnsWer: The mother was never sad over her husband’s death.

B the reasons the older sister despises her younger sister

Wrong AnsWer: It is never stated that the older girl hated the younger girl.

C the differences between the two daughters right AnsWer: The third person POV allows the reader to see how the two girls are different.

D the prince’s gratitude to the fairy Wrong AnsWer: The prince never knew about the fairy.

Exercise Reasoning

8 What is the best summary of the story?

A A fairy casts a spell on two daughters, and they use the spell to make diamonds and snakes. The mother wants the younger daughter to run away so the older daughter can marry a prince.

Wrong AnsWer: This does not tell the story in the correct way.

B After a fairy casts a spell on a woman’s younger daughter, she wants her older daughter to have a spell too. The younger daughter runs away so she can marry a prince.

Wrong AnsWer: This is not what happens in the story.

C A mother wants her older daughter to get a spell like her younger daughter, but the older daughter’s spell is bad. The mother blames the younger daughter, who runs away. The young daughter finds and marries a prince, and the older daughter ends up alone.

right AnsWer: This gives the plot correctly and in the right order.

D A young daughter wants a spell, so she has a fairy give her one. Then her older sister decides she wants one. Once the mother finds out that the older daughter receives a bad spell, she goes after the younger daughter. Then a prince asks the younger sister to marry him.

Wrong AnsWer: The younger girl does not have a fairy give her a spell.

9 A theme expressed in this story centers on —

A diamonds being a girl’s best friend Wrong AnsWer: Diamonds were not a main part of this story.

B knowing what goes around comes around right AnsWer: The fact that if you do wrong it will come back to you is a strong message in the story.

C appreciating what you have Wrong AnsWer: The older girl does not learn to appreciate what she has.

D he who has the gold, rules Wrong AnsWer: This was not learned by anyone in the story.

Sam

ple

Do Not

Dup

licat

e

Page 35: by GF Educators, Inc. Seventh Grade Reading Book … Sample Books/7th Reading Practice...Literary Text/Drama: Full length passage #2 ..... 54-58 Sample Reporting Category 2 Answer

31 © 2016 GF Educators, Inc.Seventh Grade Reading

Literary Text/Literary Nonfiction Supporting 7.7A Understanding the SkillThere are two types of nonfiction writing.

Informational nonfiction (Expository) is writing with the purpose of describing

or expressing facts.

Literary nonfiction uses literary styles and techniques to create factually

accurate narratives about people, places, and events in the real world.

Essay

Letter

Speeches

Memoirs

Diary

Journal

Autobiography

Biography

Travel writing

Nature writing

Science writing

Sports writing

In 7th grade, the TEKS for literary nonfiction focus on the autobiography, the

diary, and fictional adaptations of either of these.

Autobiography Diary Fictional AdaptationGenre Literary Nonfiction Literary Nonfiction Literary Nonfiction

Subject Matter

Story of a person’s life told by a person who is reflecting back on significant life events

A daily journal that is kept in which the writer speaks freely and without judgement; not intended for an audience

Events in a person’s life are the basis of a story

Scope

Can be about the author’s entire life or just a portion of it

Personal and emotional in nature; focuses on thoughts and feelings

• Events are altered to add drama or make it more interesting

Perspective Author’s reflective Author’s immediate Observer’s secondhandPoint of View 1st person 1st person Usually 3rd person

Time

Written many years after the events have happened; a review of life from a particular moment in time

Written as events occur in the writer’s life; moves through a series of moments in time

Written during the subject’s lifetime or after his or her deathSa

mpl

e

Do Not

Dup

licat

e

Page 36: by GF Educators, Inc. Seventh Grade Reading Book … Sample Books/7th Reading Practice...Literary Text/Drama: Full length passage #2 ..... 54-58 Sample Reporting Category 2 Answer

32Seventh Grade Reading © 2016 GF Educators, Inc.

Literary Text/Literary Nonfiction Full-Length Passage #1 Mixed Practice Read the selection and choose the best answer to each

question.

Reagan’s Reply 1 In 1984, Andy, a seventh-grader from South Carolina

wrote a letter to then President Ronald Reagan. In his letter,

Andy wrote, “Today my mother declared my bedroom a

disaster area. I would like to request federal funds to hire a

crew to clean up my room.”

2 Here is a copy of Reagan’s reply:

3 Andy Smith Irmo, South Carolina May 11, 1984

4 Dear Andy:

5 I'm sorry to be so late in answering your letter, but as you know, I've been in China and found your letter here upon my return.

6 Your application for disaster relief has been duly noted, but I must point out one technical problem: the authority declaring the disaster is supposed to make the request. In this case your mother.

7 However, setting that aside, I'll have to point out the larger problem of available funds. This has been a year of disasters: 539 hurricanes as of May 4th and several more since, numerous floods, forest fires, drought in Texas, and a number of earthquakes. What I'm getting at is that funds are dangerously low.

8 May I make a suggestion? This administration, believing that government has done many things that could better be done by volunteers at the local level, has sponsored a Private Sector Initiative program, calling upon people to practice volunteerism in the solving of a number of local problems.

9 Your situation appears to be a natural. I'm sure your mother was fully justified in proclaiming your room a disaster. Therefore, you are in an excellent position to launch another volunteer program to go along with the more than 3,000 already underway in our nation — congratulations.

10 Give my best regards to your mother.

Sincerely, Ronald Reagan

Question 7The details in paragraph 1 help the reader infer that –

D Andy has a sense of humor

Andy does have a sense of humor as a 7th grader to send this letter. There is no evidence to support answer choices A, B, or C.

Question 4Reagan includes the question in paragraph 8 most likely to –

D suggest a change to Andy’s idea

Reagan is trying to move Andy towards starting a volunteer program that might fit his situation - becoming a help to the nation, instead of a taker.

Question 2Ronald Reagan uses this sentence to explain how his disaster relief fund –

C only allowed him to help real disasters

If funds are low, he could not have helped a boy trying to be funny.

Question 1In paragraph 6, the word relief means –

A assistance

Applying indicates that you need assistance.

Question 6What is ironic about Andy requesting disaster relief?

A Andy realizes that the one who declared his room a disaster had to request help.

Andy went over his mother’s head to get help, but she has to apply for the help.

Sam

ple

Do Not

Dup

licat

e

Page 37: by GF Educators, Inc. Seventh Grade Reading Book … Sample Books/7th Reading Practice...Literary Text/Drama: Full length passage #2 ..... 54-58 Sample Reporting Category 2 Answer

33 © 2016 GF Educators, Inc.Seventh Grade Reading

3 What can the reader infer about Andy’s mother?

A She thought that telling her son about his room would encourage him to clean it.

right AnsWer: Telling Andy his room was a disaster should have encouraged him to clean.

B She gave her son an ultimatum to get his room cleaned or she would have to rope it off.

Wrong AnsWer: He was not given an ultimatum.

C She concentrated on a plan to get him help from the community to start a new volunteer program.

Wrong AnsWer: She did not get him set up with community help. This was a suggestion from the President to encourage Andy to do something for the community.

D She came to the conclusion that her son must have had a natural occurrence in his room.

Wrong AnsWer: She knew it was not something in nature that caused the room’s mess.

Exercise Reasoning

8 Which sentence from the passage best shows that Reagan most likely agrees with Andy’s mother about the condition of his room?

A I'm sorry to be so late in answering your letter, but as you know, I've been in China and found your letter here upon my return.

Wrong AnsWer: This just indicates that he was late in responding.

B … the authority declaring the disaster is supposed to make the request. In this case your mother.

Wrong AnsWer: This just lets the reader know that his mother can declare the disaster.

C I'm sure your mother was fully justified in proclaiming your room a disaster.

right AnsWer: Saying his mother was justified reveals that he agrees with Andy's mother.

D Give my best regards to your mother. Wrong AnsWer: Reagan is telling Andy to acknowledge Andy's mother for him.

9 The tone of President Reagan’s letter is –

A scolding Wrong AnsWer: No words used indicate a scolding.

B hopeful Wrong AnsWer: He may be hopeful that Andy cleans his room, but the tone is not hopeful.

C light-hearted right AnsWer: The words used indicate light-heartedness: “the authority declaring the disaster is supposed to make the request. In this case your mother.

D dismissive Wrong AnsWer: Reagan does not dismiss Andy's problem.

5 What lesson does Andy learn from reading the President’s letter?

A Respect other people’s property. Wrong AnsWer: He was not disrespecting someone else’s property.

B Money doesn’t buy happiness. Wrong AnsWer: Andy wasn’t looking for money for happiness.

C Sometimes you should listen to other’s advice.

right AnsWer: Andy needs to listen to the President’s advice.

D Be thankful for the things your parents give you.

Wrong AnsWer: Andy did not receive anything, so he isn't focusing on himself.

Sam

ple

Do Not

Dup

licat

e

Page 38: by GF Educators, Inc. Seventh Grade Reading Book … Sample Books/7th Reading Practice...Literary Text/Drama: Full length passage #2 ..... 54-58 Sample Reporting Category 2 Answer

34Seventh Grade Reading © 2016 GF Educators, Inc.

Literary Text/Literary Nonfiction Full-Length Passage #2 Mixed Practice

Apples AppealJanuary 1980

1 I’m ready to move out of the hustle of Houston life

and into the quiet Hill Country. I won’t miss the stress

of my life as a petroleum geologist. Besides, it seems

that the oil business is starting to drop off. I think

ranching might be a good change of pace for our family.

Next weekend we will begin the search for a good piece

of land in the Texas Hill Country.

June 1980

2 After months of searching, we’ve finally found the

right place. The land is in Bandera County, about ten

miles west of the small town of Medina. The property

is a little bit neglected and run down but has some

beautiful bottomland and lots of water from the creek

that runs through it. There is a beautiful spot for a

home nestled in a grove of trees on top of a hill. We

camped there last night, and I woke to a majestic

sunrise. The land has never been used as cropland,

and there seems to be an abundant supply of grass and

water. I think cattle will do well here.

August 1980

3 This has been a dry summer. There’s been so little

rain and the few cows that we own have eaten every bit

of the grass. There’s hardly a blade of grass left

on the ranch. I need to sell the cattle since there

Read the selection and choose the best answer to each question.

In 1980, Baxter and Carol Adams established Love Creek Orchards, Texas’ first successful commercial apple orchard. Read the following entries from Baxter’s journal to learn more about these modern-day agricultural pioneers.

Question 1What is the main point of the August 1980 journal entry?

A The dry summer showed the Adams that their land was not well-suited for raising cattle.

The Adams decided to sell their cows and figure something else to do on their land.Sa

mpl

e

Do Not

Dup

licat

e

Page 39: by GF Educators, Inc. Seventh Grade Reading Book … Sample Books/7th Reading Practice...Literary Text/Drama: Full length passage #2 ..... 54-58 Sample Reporting Category 2 Answer

35 © 2016 GF Educators, Inc.Seventh Grade Reading

isn’t anything for them to eat. Next, I’m going to

do some research. There must be a better way

to use this land, and I plan to find it. We’ve begun

building our home on the hilltop, and we’ve fallen in

love with this part of Texas. The last thing I want to do

is give up this dream.

October 1980

4 I’ve done lots of reading, and I’ve met with a

horticulturist from the extension service. He suggested

that I might want to experiment with planting a small

orchard of dwarf apple trees. Traditionally, apple

growing in the United States has been limited to the

Northwest and New England. No one in Texas has ever

succeeded at growing apples commercially. It seems

that the Texas soil doesn’t have enough acid in it. If

I treat the soil with large amounts of sulphur before I

plant the trees, I can make it more acidic. This would

give the apple trees a much better chance to grow and

produce a good crop. I know that I’m taking a big risk,

but my family is supporting me in my latest adventure.

December 1980

5 Love Creek Orchards begins! We’ve planted about

a thousand dwarf apple trees on five acres of our

bottomland. There are several varieties including

Jonathan, Granny Smith, Golden Delicious, and Red

Delicious. The trees should grow to be seven or eight

feet tall, and, if all goes well, will each produce two

bushels of fruit each year.

March 1981

6 The trees are growing well, and we are supporting

their growth with a system of wire trellises. As the

limbs grow, we fasten them to a trellis wire and shape

them so they will produce more fruit. We installed an

irrigation system to provide water to the trees. The

water comes from the creek on our property.

Sam

ple

Do Not

Dup

licat

e

Page 40: by GF Educators, Inc. Seventh Grade Reading Book … Sample Books/7th Reading Practice...Literary Text/Drama: Full length passage #2 ..... 54-58 Sample Reporting Category 2 Answer

36Seventh Grade Reading © 2016 GF Educators, Inc.

December 1981

7 While we wait for the trees to mature, we are

learning all that we can about the business of growing

and marketing apples. We’ve traveled to other

orchards around the country, and we read every apple

journal we can find. We’ve decided to use all natural

growing and pest control methods. We certainly don’t

want to damage our water supply with pesticides.

July 1982

8 Our first crop! The trees have produced beautiful

fruit that is absolutely delicious. We let the fruit ripen

on the trees, so our apples have up to forty percent

more sugar than apples imported to Texas. It looks

like our orchard is going to be a success. We are

making plans to install a new, larger orchard farther

down the creek, which will be home to three to four

thousand trees. We are also looking at another piece

of property as a possible orchard site.

June 1987

9 Our apple business is growing. We now have

approximately 10,000 apple trees. We’ve brought

in members of our family to act as sales managers

and production managers for our orchards. We are

experimenting with several exotic apple varieties, a

vineyard, and maple seedlings. Every Saturday we

conduct tours of our orchards. We have a small

shop in a nearby town where we sell our apples,

fresh cider, apple products, and handcrafted

goods. The shop also has an assortment of

handmade knives, quilts, leather goods, T-shirts,

photographs and paintings by artists from the

area.

Question 2Baxter’s July 1982 entry illustrates that he felt –

C hopeful

He is hopeful because their first crop came in well, and they are planning ahead.

Question 3The main reason the author included the June 1987 journal entry is to show that –

B the Adams sell other products besides apples

The entry lets the reader know that other things are sold at the shop besides apples.

Sam

ple

Do Not

Dup

licat

e

Page 41: by GF Educators, Inc. Seventh Grade Reading Book … Sample Books/7th Reading Practice...Literary Text/Drama: Full length passage #2 ..... 54-58 Sample Reporting Category 2 Answer

37 © 2016 GF Educators, Inc.Seventh Grade Reading

July 1987

10 The citizens of our community organized and held

the first Texas International Apple Festival to promote

the Texas Hill Country apple industry. It was a great

success. Plans have already begun for next year’s

celebration.

July 1988

11 The second annual Apple Festival begins tonight

in downtown Medina. Medina is now known as the

“Apple Capital of Texas.” The pageants will be held

tonight. Tomorrow, there will be arts and crafts and

food booths, tours of the orchard, a variety of dance

groups, and a parade. The festival will end with an old-

fashioned barn dance. Proceeds from the festival will go

toward building a town park in Medina.

July 2015

12 This marks the thirty-fifth year that our family

has produced apples in the Texas Hill Country. We

are amazed at how successful our business has been.

Little did I realize how well this venture would turn out.

Texans in many other parts of the state have planted

orchards. Fresh-off-the-tree apples and apple products

made from Texas apples are appearing throughout

the area. It is exciting to know we played a part in

establishing a new industry for the great state of Texas.

Sam

ple

Do Not

Dup

licat

e

Page 42: by GF Educators, Inc. Seventh Grade Reading Book … Sample Books/7th Reading Practice...Literary Text/Drama: Full length passage #2 ..... 54-58 Sample Reporting Category 2 Answer

38Seventh Grade Reading © 2016 GF Educators, Inc.

4 Which sentence from the June 1987 journal entry best supports the idea that Love Creek Orchards has expanded beyond the orchards?

A We now have approximately 10,000 apple trees.

Wrong AnsWer: We do not know if this expanded the orchards.

B We’ve brought in members of our family to act as sales managers and production managers for our orchards.

Wrong AnsWer: Bringing in family members for the orchards is not expanding beyond the orchards.

C We are experimenting with several exotic apple varieties, a vineyard, and maple seedlings.

Wrong AnsWer: Experimenting with different varieties of apples has to do with the orchards.

D We have a small shop in a nearby town where we sell our apples, fresh cider, apple products, and handcrafted goods.

right AnsWer: Adding a shop that sells more products shows an expansion.

Exercise Reasoning

5 How does the setting influence the plot of the story?

A The decline in the oil industry contributes to Baxter Adam’s conflict.

Wrong AnsWer: The oil industry does not have anything to do with his conflict.

B The condition of the land in Medina inspires Baxter to try new things.

right AnsWer: The fact that the land could not be used to raise cows inspired Baxter to try something new, which caused the diary entries to evolve in a new direction.

C The citizens become influential characters in Baxter’s journal.

Wrong AnsWer: The citizens were not influential.

D Baxter’s family encourages him to work harder to make more money

Wrong AnsWer: The family was not motivated by money.

6 What lesson does Baxter Adams learn while spending time in his orchards?

A Weather usually has complex effects on nature.

Wrong AnsWer: Weather was not a problem with the orchards.

B If you do many things, you will have better results.

Wrong AnsWer: They did not do many things in the orchards. They only grew apples.

C Being persistent with something helps it to be successful.

right AnsWer: They had to be persistent with learning how to grow the apples, what needed to be done on the land, what kind of apples to grow, etc. All of this helped them to be successful.

D Things happen in their own time frame. Wrong AnsWer: This was not something learned while being in the orchards.

Sam

ple

Do Not

Dup

licat

e

Page 43: by GF Educators, Inc. Seventh Grade Reading Book … Sample Books/7th Reading Practice...Literary Text/Drama: Full length passage #2 ..... 54-58 Sample Reporting Category 2 Answer

39 © 2016 GF Educators, Inc.Seventh Grade Reading

7 From this passage, you can tell that Baxter Adams –

A knows all there is to know about growing apples

Wrong AnsWer: He does not since he states that he needed to research.

B is concerned about protecting the environment

right AnsWer: They are careful about their pest control which shows concern about the environment.

C would be successful no matter what he tried Wrong AnsWer: He may be, but this is not something that can be inferred in the passage.

D was an oil millionaire from Houston Wrong AnsWer: It never said he was a millionaire.

Exercise Reasoning

8 The point of view from which the story is told gives the reader insight into –

A the long-term commitment required to establish a successful apple orchard

right AnsWer: Having Baxter’s POV lets you know his commitment is great because of everything he did.

B reasons why the Texas Hill Country is not suitable for raising cattle

Wrong AnsWer: We do not gain insight into this because of the POV.

C the support of the people of Medina, Texas for the Baxter’s apple business

Wrong AnsWer: It is not from their point of view, so we have no insight into their support.

D why people move from crowded cities to rural areas

Wrong AnsWer: This is only one person, so we do not get a general idea of why people move.

9 Which of the following best completes the summary of the passage?

A He and his family established successful apple orchards because of their dedication and hard work.

right AnsWer: This provides the rest of the information to complete the summary.

B The Adams family was ready to move from Houston to the Hill Country.

Wrong AnsWer: That was mentioned in the first sentence.

C Once there, he established an orchard in Texas.

Wrong AnsWer: That is not enough information.

D Growing apples in Texas will be a successful venture for other farmers.

Wrong AnsWer: This is not a part of the story.Sa

mpl

e

Do Not

Dup

licat

e

Page 44: by GF Educators, Inc. Seventh Grade Reading Book … Sample Books/7th Reading Practice...Literary Text/Drama: Full length passage #2 ..... 54-58 Sample Reporting Category 2 Answer

40Seventh Grade Reading © 2016 GF Educators, Inc.

Literary Text/Literary Nonfiction Full Length Passage #3 Mixed Practice

Read the selection and choose the best answer to each question.

Meatloaf1 The orange light of sunset shown in the window as

I worked on my math homework. Math is my favorite

subject. My pencil moved like a flash as I labored

through first the section on fractions, then the

page of long division. Not expecting the phone to

ring, I was startled to hear the sound. The caller ID said

it was Dad.

2 “Hey, Adam!” Dad tried being upbeat, but I could tell

something was up. “I hate to ask this, but I’ve had an

unexpected delay in my project. Seems like first it’s one

thing, then another, and there’s always a snag with that

other company! I need to catch the contractor between

now and five o’clock. If I stay here until five, the traffic

will be so bad that there’s no way I’ll be home in time to

get supper on the table at a decent hour. Do you think

you could help me by fixing dinner tonight? It’s

pretty straightforward.”

3 Puzzled, I decided to see if I could at least find what

needed to be cooked. Walking to the refrigerator, I saw

the recipe card: meatloaf. “Is it this meatloaf card that’s

on the refrigerator?”

Question 3The author includes this sentence most likely to show–

A how fast Adam was completing his math work

If your pencil moves like a flash, it is going fast. Think of a flash of lightning – it is quick.

Question 1Which meaning of decent is used in paragraph 2 of the selection?

D Definition 4 (suitable, appropriate)

The words suitable/appropriate are the best substitution for the word decent. Supper at a suitable time.

Question 4What can the reader infer about the author’s father?

D He believed that his son was capable of stepping in to help out.

If the father did not believe his son could make the meal, he would not have called to ask him. He walked him through where everything was for the meal and was confident that he could do it.Sa

mpl

e

Do Not

Dup

licat

e

Page 45: by GF Educators, Inc. Seventh Grade Reading Book … Sample Books/7th Reading Practice...Literary Text/Drama: Full length passage #2 ..... 54-58 Sample Reporting Category 2 Answer

41 © 2016 GF Educators, Inc.Seventh Grade Reading

Meatloaf

1 ½ pound ground beef 1 T Worcestershire sauce

1 egg ¼ cup chili sauce

1 cup milk ½ teaspoon salt or less

½ cup bread crumbs 4-6 medium potatoes

1. Preheat oven to 350 degrees.

2. Using a fork, mix the first seven ingredients together well.

3. Put into loaf pan and bake one hour.

4. Wash the potatoes and pierce each one several times with a fork. Place the potatoes on the oven rack to bake while the meatloaf cooks. They will take about one hour.

5. Remove potatoes and pan of meatloaf using potholders.

6. Serves 4-6 people, depending on how hungry they are.

4 “If that’s where Mom put the recipe, yes. Read the

ingredients to me.” After I read the list, Dad said, “Go to

the pantry and verify that we have that much bread.”

5 I was perturbed that so much was being asked: not

only the cooking, but also the hunting of ingredients.

Couldn’t they at least have the ingredients on

hand? Instead I said nothing and went to the pantry. I

picked up a wad of orange-brown plastic to spy only the

heel. “Nope.”

6 “I think your mom has used oatmeal when she

doesn’t have bread. Get the oatmeal, then, and see how

full it is,” Dad continued. I reported that the seal of the

oatmeal carton was intact. “Great — that’ll work, then.

Use up that heel since no one wants it anyway, and

we can hide it in the meatloaf. Fill up the remainder of

that half-cup portion with the oats. Okay — everything

else, we should have. I’ve got to get back to calling this

contractor. I’m sorry for the trouble, but I know you

Question 9The directions in this story are about how to –

B make meatloaf and baked potatoes

Question 2What does the word perturbed mean in paragraph 5 of the selection?

B Bothered

The author is bothered by what is being asked of him.

Question 6The author includes the question in paragraph 5 most likely to –

D imply that Adam is frustrated with what is being asked of him

Adam was frustrated because he didn't want to search for ingredients. He thought they should have everything on hand for a meal that needed to be made.Sa

mpl

e

Do Not

Dup

licat

e

Page 46: by GF Educators, Inc. Seventh Grade Reading Book … Sample Books/7th Reading Practice...Literary Text/Drama: Full length passage #2 ..... 54-58 Sample Reporting Category 2 Answer

42Seventh Grade Reading © 2016 GF Educators, Inc.

don’t want Mom disappointed any more than I do.”

7 “Yes, sir,” I replied. Hanging up the phone, I peered

at the clock—3:45. I decided to get to work on the

preparations for dinner.

8 “I’ll pull together my ingredients and measuring tools

now, so that all I have to factor in is the time to put the

things together.” I got a stool to reach the chili sauce,

and then located the measuring cups along with Mom’s

collection of measuring spoons. The other ingredients

were easily found. “There!” I proclaimed with a level of

satisfaction. “Everything’s out and on the counter except

the cold items. I’ll do homework until 4:30, wash my

hands, and begin cooking!”

9 I couldn’t believe how quickly that 45 minutes went

by! The recipe card required the oven to be on 350°.

Since everything was ready on the counter, it didn’t take

me any time at all to mix the meatloaf and put it into

the pan. Once that part was done, I found the potatoes,

washed them, pierced them, put them and the meatloaf

into the oven. I set the timer for one hour, and I went

back to work on my homework.

10 When Mom got home, she was shocked that dinner

was ready and Dad wasn’t home yet. “What on earth!”

she exclaimed.

11 “It was me, Mom,” I asserted with my chest

out and a smile on my face.

Question 5What lesson does Adam learn while spending time in the kitchen?

B Sometimes we can surprise ourselves with things we can accomplish.

Adam seemed concerned about his ability, but he did do it. He has his chest out and a smile on his face when he tells his mom he did it.

Sam

ple

Do Not

Dup

licat

e

Page 47: by GF Educators, Inc. Seventh Grade Reading Book … Sample Books/7th Reading Practice...Literary Text/Drama: Full length passage #2 ..... 54-58 Sample Reporting Category 2 Answer

43 © 2016 GF Educators, Inc.Seventh Grade Reading

Exercise Reasoning

7 Read the following sentences from the selection.

“There!” I proclaimed with a level of satisfaction. (paragraph 8)

“It was me, Mom,” I asserted with my chest out and a smile on my face. (paragraph 11)

What idea does Adam communicate with these sentences?

A He didn’t really want to cook the meatloaf. Wrong AnsWer: While this is indicated in the passage, Adam not wanting to make the meatloaf is not communicated by these two sentences.

B He was proud of what he had accomplished. right AnsWer: Being satisfied and sticking his chest out with a smile illustrate that Adam was proud.

C He would rather cook than do his homework. Wrong AnsWer: The idea communicated by the sentences is not Adam's attitude toward his homework.

D He was concerned about being asked to make the meatloaf.

Wrong AnsWer: Adam was concerned, but that is not the idea communicated in these two sentences.

8 What information is included in the selection, but missing from the recipe?

A Piercing the potatoes Wrong AnsWer: Piercing the potatoes is in paragraph 9 of the selection and Step 4 of the recipe.

B Preheating the oven Wrong AnsWer: Preheating the oven is mentioned in paragraph 9 of the selection and Step 1 of the recipe.

C Using potholders Wrong AnsWer: Potholders are mentioned in Step 5 of the recipe, but not in the selection.

D Using oatmeal right AnsWer: The oatmeal is mentioned in the selection, but not in the recipe. The recipe refers to breadcrumbs, not oatmeal.

Sam

ple

Do Not

Dup

licat

e

Page 48: by GF Educators, Inc. Seventh Grade Reading Book … Sample Books/7th Reading Practice...Literary Text/Drama: Full length passage #2 ..... 54-58 Sample Reporting Category 2 Answer

44Seventh Grade Reading © 2016 GF Educators, Inc.

Literary Text/Poetry Full-Length Passage #1 Mixed PracticeRead the poem and choose the best answer to each question.

Stopping by Woods on a Snowy EveningBy Robert Frost

Whose woods these are I think I know.

His house is in the village, though;

He will not see me stopping here

To watch his woods fill up with snow.

5 My little horse must think it queer

To stop without a farmhouse near

Between the woods and frozen lake

The darkest evening of the year.

He gives his harness bells a shake

10 To ask if there is some mistake.

The only other sounds the sweep

Of easy wind and downy flake.

The woods are lovely, dark, and deep,

But I have promises to keep,

15 And miles to go before I sleep,

And miles to go before I sleep.

Source: Poem Hunter (www.poemhunter.com/poem)

Question 2What is the effect of the repetition of the last two lines of the poem?

A Reveals a literal and metaphorical interpretation to the lines

The lines refer to the fact that he literally has miles to go before he sleeps for the night, and metaphorically speaking, he has miles to go to complete the work he has left to do in life before he sleeps in death.

Question 1Why does the speaker stop by the woods?

C He is drawn to the quietly falling snow and is inspired to watch.

Lines 3 and 4 are evidence as to why he stops.

Question 3Which line in the poem reminds the speaker that he cannot linger in the woods?

D But I have promises to keep

If he has promises to keep, he cannot linger in the woods.

Sam

ple

Do Not

Dup

licat

e

Page 49: by GF Educators, Inc. Seventh Grade Reading Book … Sample Books/7th Reading Practice...Literary Text/Drama: Full length passage #2 ..... 54-58 Sample Reporting Category 2 Answer

45 © 2016 GF Educators, Inc.Seventh Grade Reading

4 In the poem, the poet is comparing –

A the house and the village Wrong AnsWer: The house is located in the village.

B nature and civilization right AnsWer: The beauty of the woods is being compared to the life he has to lead with work and all it entails.

C man and horses Wrong AnsWer: The man and the horse travel together, but they are not compared.

D nature and man Wrong AnsWer: Nature is what the man is admiring and longing for.

Exercise Reasoning

5 Read these two lines from the poem.

The only other sounds the sweep

Of easy wind and downy flake.

The imagery in these lines helps the reader understand – A the speaker’s solitude Wrong AnsWer:

This language appeals to the sense of sound, not to the speaker's feelings.

B the quietness of the woods right AnsWer: The only sounds were the soft sounds of a sweeping, easy wind and gentle snowfall, so it was very quiet.

C why the speaker stopped Wrong AnsWer: These lines refer to the sounds in the forest, not the reason the speaker stopped.

D what the speaker was searching for Wrong AnsWer: These lines are not a visual image of what the speaker is seeking.

6 This poem is most likely about a man who –

A stops to experience some peacefulness but has obligations to honor

right AnsWer: The promises that the man has to keep are the man's job obligations that are ever present, even as he enjoys his ride through the woods.

B is searching for an alternate route to traveling in the woods

Wrong AnsWer: He is not looking for another route; he is enjoying the woods.

C tries to get his horse to move faster through the dark woods

Wrong AnsWer: He does not try to move the horse along faster.

D goes through the woods to find the owner of the property

Wrong AnsWer: The speaker mentions the owner, but he is not looking for the owner.

Sam

ple

Do Not

Dup

licat

e

Page 50: by GF Educators, Inc. Seventh Grade Reading Book … Sample Books/7th Reading Practice...Literary Text/Drama: Full length passage #2 ..... 54-58 Sample Reporting Category 2 Answer

46Seventh Grade Reading © 2016 GF Educators, Inc.

Literary Text/Poetry Full-Length Passage #2 Mixed PracticeRead the poem and choose the best answer to each question.

The FlowerBy Alfred, Lord Tennyson

ONCE in a golden hour

I cast to earth a seed.

Up there came a flower,

The people said, a weed.

5 To and fro they went

Thro' my garden bower,

And muttering discontent

Cursed me and my flower.

Then it grew so tall

10 It wore a crown of light,

But thieves from o'er the wall

Stole the seed by night.

Sow'd it far and wide

By every town and tower,

15 Till all the people cried,

"Splendid is the flower!"

Read my little fable:

He that runs may read.

Most can raise the flowers now,

20 For all have got the seed.

And some are pretty enough,

And some are poor indeed;

And now again the people

Call it but a weed.

Question 6The poet includes these lines most likely to suggest that the speaker –

D does not receive approval from others

If the people curse him, he is not receiving their approval.

Question 2The poet places the words “Splendid is the flower!” on a line by themselves most likely because the words –

A reveal a shift in people’s opinion of the flower

The same people who cursed the flower in line 9 are now calling it splendid -- this reveals a shift in their opinion.

Question 4In the final two lines of the poem, the speaker points out that –

A people don’t appreciate the flower since it is now common

Because the flower is so common and not always pretty, people once again call it a weed.Sa

mpl

e

Do Not

Dup

licat

e

Page 51: by GF Educators, Inc. Seventh Grade Reading Book … Sample Books/7th Reading Practice...Literary Text/Drama: Full length passage #2 ..... 54-58 Sample Reporting Category 2 Answer

47 © 2016 GF Educators, Inc.Seventh Grade Reading

1 The reader can conclude that the speaker’s flower –

A was only stolen for its beauty Wrong AnsWer: The reason it was stolen is not given.

B grew in beautiful gardens Wrong AnsWer: There is no mention of beautiful gardens.

C was a weed and no one liked it Wrong AnsWer: It was called a weed, but after the speaker planted it and it was later sown far and wide, people called it splendid and they liked it.

D offered something new to people right AnsWer: When the speaker planted the flower, it grew to be beautiful, and the people liked the new plant.

Exercise Reasoning

3 This poem is mostly about someone who –

A decides to follow what everyone else does and calls the flower a weed

Wrong AnsWer: The speaker does not call the flower a weed, nor does he follow everyone else.

B has a new idea that people cannot accept at first, but after time they decide it is good

right AnsWer: The speaker saw the beauty in the flower even though no one else did, so he grew the flower, and people came to like it. The flower is a metaphor for something new that people had a hard time accepting.

C goes into the neighbor's yard, steals his idea, and gives it to everyone

Wrong AnsWer: This is not what happened. Others came into the speaker's property and stole the seed.

D tries the same method for creating a flower and is not successful because people call it a weed

Wrong AnsWer: He didn't create the flower. The flower was already there, and he planted the seed and grew beautiful flowers in his garden.

5 Which lines from the poem best suggest that the speaker’s idea is accepted?

A Once in a golden hour/I cast to earth a seed Wrong AnsWer: This lets us know he planted the flower.

B Sow'd it far and wide/By every town and tower,

Wrong AnsWer: This lets us know that the people who stole the seeds planted them over a wide area.

C Most can raise the flowers now,/For all have got the seed.

right AnsWer: The people took the seeds and grew the flower if they wanted to. The flower is now common and accepted.

D And some are pretty enough,/And some are poor indeed;

Wrong AnsWer: This explains what happened when they planted the flower. Not all of the plants look alike.

Sam

ple

Do Not

Dup

licat

e

Page 52: by GF Educators, Inc. Seventh Grade Reading Book … Sample Books/7th Reading Practice...Literary Text/Drama: Full length passage #2 ..... 54-58 Sample Reporting Category 2 Answer

48Seventh Grade Reading © 2016 GF Educators, Inc.

Literary Text/Poetry Full-Length Passage #3 Mixed PracticeRead the poem and choose the best answer to each question.

I Wandered Lonely as a Cloud by William Wordsworth

I wandered lonely as a cloud

That floats on high o'er vales and hills,

When all at once I saw a crowd,

A host, of golden daffodils;

5 Beside the lake, beneath the trees,

Fluttering and dancing in the breeze.

Continuous as the stars that shine

And twinkle on the Milky Way,

They stretched in never-ending line

10 Along the margin of a bay:

Ten thousand saw I at a glance,

Tossing their heads in sprightly dance.

The waves beside them danced; but they

Out-did the sparkling waves in glee:

15 A poet could not but be gay,

In such a jocund company:

I gazed — and gazed — but little thought

What wealth the show to me had brought:

For oft, when on my couch I lie

20 In vacant or in pensive mood,

They flash upon that inward eye

Which is the bliss of solitude;

And then my heart with pleasure fills,

And dances with the daffodils.

Question 1The imagery in lines 5 through 12 helps the reader understand

D the speaker's adoration of the daffodils

The words used speak of an admiration of the beauty of the flowers.

Question 2The poet includes these lines most likely to –

A highlight that the daffodils were more breathtaking than the waves

By out-doing the waves, the daffodils surpass them.

Question 3The daffodils are personified as –

B humans

The flowers are portrayed as being in a crowd, dancing, and that they are jocund, or cheerful company. These are human characteristics.

Question 4What is the most likely reason the poet uses the hyphens in line 17?

D To show the persistence of the speaker’s gaze

The gaze is persistent - the hyphens create a pause and show that he continues the gaze.

Question 6In the last stanza of the poem, the speaker –

C fondly remembers the day by the lake

Line 21 reveals the speaker is remembering.

Question 5Which lines best confirms the message “Nature’s beauty uplifts the human spirit"?

D And then my heart with pleasure fills,

Remembering the beauty he witnessed in nature, the speaker's heart fills with pleasure.

Sam

ple

Do Not

Dup

licat

e

Page 53: by GF Educators, Inc. Seventh Grade Reading Book … Sample Books/7th Reading Practice...Literary Text/Drama: Full length passage #2 ..... 54-58 Sample Reporting Category 2 Answer

49 © 2016 GF Educators, Inc.Seventh Grade Reading

Literary Text/Drama Supporting 7.5 Understanding the Skill

Drama is written to be performed by actors. When reading a play, your goal is to

imagine as much as possible about the performance of the play. Try to see it in

your “mind’s eye.”

The actual text of a drama is known as a script. The script includes a list of the

cast of characters, a description of the setting, stage directions, and dialogue.

Dialogue and stage directions are used by playwrights to create drama. They use

words and actions to define characters and reveal plot.

Dialogue — spoken words exchanged between the characters

• In drama, the entire story is presented in dialogue

• Used to establish the play’s problem and to set up the conflict

• Reveals who the characters are as well as their motivations

• Advances the plot

Stage directions — indicate how characters should speak their lines

• Tell how the characters speak (loudly, sarcastically, in a whisper)

• Tell how the characters move (places hands on hips, makes a half turn,

exits stage left)

• Tell how the characters look (eyes open in surprise, boasts with pride,

crouching in fear)

• Include notes about set design, lighting, and music

• Help the reader visualize the setting

Drama requires the reader to constantly make inferences about the characters in

a play.

As you read through a drama —

• Pay attention to what the characters say and do.

• Notice how the characters feel about each other.

• Analyze their actions to determine why they act the way they do.Sam

ple

Do Not

Dup

licat

e

Page 54: by GF Educators, Inc. Seventh Grade Reading Book … Sample Books/7th Reading Practice...Literary Text/Drama: Full length passage #2 ..... 54-58 Sample Reporting Category 2 Answer

50Seventh Grade Reading © 2016 GF Educators, Inc.

Literary Text/Drama Full-Length Passage #1 Mixed PracticeRead the selection and choose the best answer to each question.

A Lesson in MannersAdapted from a story by the same name in Fifty Famous People, by James Baldwin, New York: American Book Company, 1912

Characters:

James, Mr. Boyle’s delivery man

Annie, a servant girl

Dean Swift, the owner of the home

Scene 1

1 A large entry hall in Dean Swift’s home. Window

flanking each side of the door look out on Dean Swift’s

manicured gardens. In the adjoining drawing room

is a fireplace with fire burning. On the walls are four

bracketed gas lamps. The room contains a large writing

table, a sofa and two overstuffed chairs. Annie is

dusting the furniture in the drawing room.

2 At the rise of curtain, a knocking is heard on the

front door. Annie swiftly sets aside her cleaning and

moves to answer the door.

3 Annie: (opens the door) May I help you, sir?

4 James: (steps in from outside and hands Annie a

recently killed fine duck) Here’s a present for the Dean.

It’s from Mr. Boyle.” (He turns and walks away.)

Scene 2

5 A few days later in Dean Swift’s entry hall. There is

an insistent knocking on the door.

6 Annie: (opens the door) Oh, Mr. James, I wasn’t

expecting you. What can I do for you today?

7 James: Well, you know Mr. Boyle spends most of

his time hunting. He thinks highly of Dean Swift, so he’s

Question 1Why does the playwright include the stage directions at the beginning of Scene 1?

D To provide information about the setting of the story

This introduction allows the reader to see where the play is about to take place -- the entry hall, its appearance, how it is furnished, and what is happening as the curtain rises.

Question 2What does the word insistent mean in paragraph 5?

A Repeated

This definition best fits that the door is being knocked on continuously. The person is insisting that the door be opened by knocking repeatedly.

Sam

ple

Do Not

Dup

licat

e

Page 55: by GF Educators, Inc. Seventh Grade Reading Book … Sample Books/7th Reading Practice...Literary Text/Drama: Full length passage #2 ..... 54-58 Sample Reporting Category 2 Answer

51 © 2016 GF Educators, Inc.Seventh Grade Reading

sending him another bird. (He pulls a partridge from a

game bag and thrusts it in Annie’s face.) Here! (James

turns and strides away)

Scene 3

8 The following week in Dean Swift’s entry hall. Loud

knocking can be heard at the front door.

9 Annie: (startled, swings open the door) Mr. James,

are you hurt? Is Mr. Boyle ill? Whatever is wrong?

10 James: (speaking roughly) Here’s something else

for the Dean. (He tosses a dead quail into Annie’s arms.)

11 Dean Swift enters the drawing room, adjusts one of

the lamps by the table, sits down and picks up a pen

and paper.

12 Annie: (shutting the door firmly and handling the

quail gingerly mutters) That man! (She shakes her

head.)

13 Dean Swift: Annie, did you say something to me?

(notices Annie’s agitated state) What seems to be the

problem here?

14 Annie: That fellow has no manners!

15 Dean Swift: Let me know the next time he comes.

I will go to the door.

Scene 4

16 Two days later in Dean Swift’s entry hall. Pounding

fists repeatedly hit the door. Dean Swift strides to the

door.

17 James: (without looking up, extends his hand,

holding a bag) Here’s a rabbit from Mr. Broyle.

18 Dean Swift: (in a stern voice) See here; that is not

the way to deliver a message here. Just step inside and

make believe that you are Dean Swift. I will go out and

pretend that I am delivering him a gift. I’ll show you

how a messenger ought to behave.

19 James: (amused) I’ll agree to that. (He steps

Sam

ple

Do Not

Dup

licat

e

Page 56: by GF Educators, Inc. Seventh Grade Reading Book … Sample Books/7th Reading Practice...Literary Text/Drama: Full length passage #2 ..... 54-58 Sample Reporting Category 2 Answer

52Seventh Grade Reading © 2016 GF Educators, Inc.

inside.)

20 Dean Swift takes the rabbit and goes out of the

house. After a bit, he knocks gently at the door. James

opens the door.

21 Dean Swift: (bows gracefully) If you please, sir, Mr.

Broyle’s compliments, and he wishes you to accept this

fine rabbit.

22 James: (very politely) Oh, thank you. (Pulling out

his coin purse, he offers the Dean a shilling.) And here

is something for your trouble.

23 Dean Swift: (shaking James’ hand) See, there,

young man. Don’t forget this lesson in manners. The

next time your bear gifts to anyone’s home, be sure to

be polite.

24 James: Yes, sir, I will. (He turns and steps out.

Smiling, Dean Swift closes the door behind him.)

25 The lights dim and the curtain closes.

Question 6James’ response in Scene 4 is important because it shows –

D how he handles being taught a lesson

James accepts what Dean has taught him very well by responding to him with “Yes, sir.”

Sam

ple

Do Not

Dup

licat

e

Page 57: by GF Educators, Inc. Seventh Grade Reading Book … Sample Books/7th Reading Practice...Literary Text/Drama: Full length passage #2 ..... 54-58 Sample Reporting Category 2 Answer

53 © 2016 GF Educators, Inc.Seventh Grade Reading

3 In Scene 3, the dialogue between Annie and James reveals that Annie –

A does not appreciate James’s actions right AnsWer: Annie saying “That man!” and shaking her head shows she does not like what James is doing.

B enjoys talking to James Wrong AnsWer: She shut the door firmly, which shows she does not like to talk to James.

C refuses to clean the quail Wrong AnsWer: Nothing points to the fact that she will not clean the quail.

D is outspoken around Dean Swift Wrong AnsWer: Dean Swift doesn't criticize James. He only tells Annie that he will speak to him the next time he comes.

Exercise Reasoning

4 Which of the following lines does the playwright include to show that Dean Swift is supportive?

A Dean Swift: (shaking James’ hand) See, there, young man. Don’t forget this lesson in manners. The next time your bear gifts to anyone’s home, be sure to be polite.

Wrong AnsWer: Dean is not being supportive to James; just giving him a word of notice on how to behave.

B Dean Swift: Annie, did you say something to me? (notices Annie’s agitated state) What seems to be the problem here?

Wrong AnsWer: Asking Annie what is going on does not show support.

C Dean Swift: (bows gracefully) If you please, sir, Mr. Broyle’s compliments, and he wishes you to accept this fine rabbit.

Wrong AnsWer: This is Dean play-acting with James, not showing support.

D Dean Swift: Let me know the next time he comes. I will go to the door.

right AnsWer: Letting Annie know that he will go to the door shows that Dean is supportive of Annie’s plight.

5 The playwright uses the sound of pounding throughout the play mainly to –

A illustrate to the audience about the thickness of the door

Wrong AnsWer: This is not showing that the door is thick; that would have no bearing on the play.

B give the Dean a reason to answer the door Wrong AnsWer: The pounding does not make Dean answer the door; he only answers after he sees that Annie is annoyed.

C help the audience understand James’s attitude

right AnsWer: By pounding on the door, James is showing his annoying attitude; he could just knock, not pound.

D indicate how Annie knocks on the door Wrong AnsWer: Annie is not knocking on the door.

Sam

ple

Do Not

Dup

licat

e

Page 58: by GF Educators, Inc. Seventh Grade Reading Book … Sample Books/7th Reading Practice...Literary Text/Drama: Full length passage #2 ..... 54-58 Sample Reporting Category 2 Answer

54Seventh Grade Reading © 2016 GF Educators, Inc.

Literary Text/Drama Full-Length Passage #2 Mixed PracticeThe Fire

1 Deanna and Amanda stand stage right outside a

recently burned local grocery store that served their

small town. Other residents gather to watch as firemen

hose down remaining debris. Spotlight on Deanna and

Amanda; fade out towards storefront.

2 Amanda: Reckon we’ll have to shop in San Marcos

now.

3 Deanna: (thoughtfully) Yeah.

4 Amanda: Hate that store. So many customers, and

it seems like it’s always full of college students.

5 Deanna: (remaining unmoved, staring off) Yeah.

6 (Amanda shifts to quarter position left, hands on

hips)

7 Amanda: Deanna, what are you thinking about?

You seem lost in time.

8 Deanna: (peering up towards top of burned

building) Old Mr. Nord did the masonry up there.

(gesturing tellingly) I’m wondering how many buildings

that have his work are still left in this town. Now one

more is down.

9 Amanda: (opens mouth as she squints upward

studying the wall) Well I don’t know that they need

to tear this building down just because the inside’s

charred. Do you think they would do that?

10 Deanna: (defensively) It’s what they did to the old

library! (Hands up to hips, quarter turn left towards

Amanda) Don’t you remember how upset I was about

that?

Question 1Why does the playwright include the stage directions at the beginning of the play (paragraph 1)?

D To provide background information about what just happened

The audience needs to know what has just occurred to understand the play.

Sam

ple

Do Not

Dup

licat

e

Page 59: by GF Educators, Inc. Seventh Grade Reading Book … Sample Books/7th Reading Practice...Literary Text/Drama: Full length passage #2 ..... 54-58 Sample Reporting Category 2 Answer

55 © 2016 GF Educators, Inc.Seventh Grade Reading

11 Amanda stares to the side, communicating to the

audience that she does not remember.

12 Amanda: (slowly) Yeah… (She lies, shifting eyes

down. Pensive pause). I don’t think that’ll happen this

time. (Amanda waves hands at building and turns three-

fourths right). You comin’ to the fish fry tonight? Keith

and Molly called.

13 (Deanna, ignoring the intentional change of subject,

peers down Center Street, facing full back.)

14 Deanna: And there’s the Methodist Church that

got refurbished. And my Aunt Sessy’s quilt shop.

They condemned that, but I don’t blame ‘em. (faces

Amanda full profile) Once mildew’s in a building that’s

it. (Deanna turns three-quarters back, walks to upstage

center; gestures upstage) All that was downtown then.

15 (Amanda does not follow Deanna; she is

uninterested in town history.)

16 Amanda: I better get to the San Marcos store.

Can’t have fried fish without coleslaw. (waiting for

Deanna’s response; none comes and Amanda crosses

stage right hoping Deanna will follow)

17 Deanna remains peering upstage; a single hand

drooped lightly on hip. Amanda stops walking, quarter

turns towards her.

18 Deanna: (shakes head, voice trembling) I can’t

believe it, yet! (quiet sobbing starts)

19 Amanda: Well, Deanna—what did it matter? The

new one was already built. When they offered the old

library to the elementary school across the street, the

district said its air conditioning and roofing were too

costly to fix. So they leveled it for parking.

Question 4The Latin word audentia, meaning "a hearing, listening," helps the reader understand that audience in paragraph 11 means –

C a group of people who gather together to listen to something

The audience is the group of people watching the play. While watching, the audience is listening.

Question 2Amanda’s dialogue in paragraphs 9 – 12 suggests that she –

D does not understand Deanna’s feelings

Looking at the audience to show she does not remember indicates that she cannot understand.

Sam

ple

Do Not

Dup

licat

e

Page 60: by GF Educators, Inc. Seventh Grade Reading Book … Sample Books/7th Reading Practice...Literary Text/Drama: Full length passage #2 ..... 54-58 Sample Reporting Category 2 Answer

56Seventh Grade Reading © 2016 GF Educators, Inc.

20 Deanna: (still weeping) Parking. Leveled for

parking.

21 Amanda stands three quarters downstage, facing

Deanna in recognition of her sorrow. Deanna drops

her head, picks her fingernail, and sniffs.

22 Deanna: We used to meet there, me and Freddy.

(Deanna lifts her head, still weeping). Especially

on game days—he ALWAYS wanted to see me on

game days! (Deanna quarter turns stage left to face

Amanda, smiling through tears)

23 Amanda moves upstage two steps, leans towards

Deanna

24 Amanda: Freddy—the boy you were engaged to

before the war?

25 Deanna nods silently and crosses arms. She steps

stage left and Amanda counters by slightly moving

stage right to provide balance

26 Amanda: I’ve not heard you mention him in

ages.

27 Deanna: (abruptly guffaws, sniffs and shakes

shoulders) How could I??

28 Amanda: (carefully) Well I don’t know--I just

don’t hear it mentioned much. You and Joey got

married, and—

29 Deanna: (sternly interrupting) THAT’S RIGHT

Joey and I got married and THAT’s supposed to

make everything FINE. (Deanna faces Amanda, both

hands thrown down, crying harder).

30 Pause. Amanda is motionless, taking in the

intensity of Deanna’s reaction.

Question 5Paragraph 22 is important to the play because it shows that Deanna –

A has feelings for an old boyfriend

The tears and reminiscing show that Deanna misses Freddy.

Question 6In paragraph 27, the word guffaws means –

B laughs loudly

Looking at how Deanna is behaving shows that she abruptly laughs loudly – abruptly helps with the definition.

Sam

ple

Do Not

Dup

licat

e

Page 61: by GF Educators, Inc. Seventh Grade Reading Book … Sample Books/7th Reading Practice...Literary Text/Drama: Full length passage #2 ..... 54-58 Sample Reporting Category 2 Answer

57 © 2016 GF Educators, Inc.Seventh Grade Reading

31 Deanna: They were wrong. They were

wrong when they told me that time would

make me forget. And the library—that library

they tore down for PARKING, was the last

place. It was the last place, that Freddy ever

kissed me. And he told me he’d write. And he

told me he’d come home.

32 (Amanda takes two steps towards Deanna.)

33 Amanda: (speaking softly) And he never did.

34 Deanna quivers in sorrow and shakes her head.

Amanda moves upstage next to Deanna, links arms,

and carefully escorts her down center stage.

35 Amanda: But you’ve had an all right life.

(Friends halt together, Deanna stares down, still

sobbing, Amanda quarter turned towards Deanna)

You and Joey got married. Maybe---you might not

want me saying it but—(Amanda shakes her head

in sympathy) maybe you don’t love Joey, the way

you did Freddy. But—you’re here. We love you.

Your kids played with mine. (Pause. Deanna ceases

physical shaking)

36 Long pause

37 Amanda: I can’t imagine this town without you.

Freddy would’ve carted you who knows where! You’re

my friend, Deanna. And I’m sorry for your pain. I

never knew you cared so much about that part of

your life.

38 Deanna wipes last of tears, gazes at Amanda.

39 Deanna: I’ll go with you to San Marcos; you

make terrible coleslaw. (friends exchange smiles and

exit stage left, arm in arm)

Question 3The playwright includes these lines to suggest that —

C Deanna is distressed

Deanna is upset that the city is tearing down the places that reminded her of Freddy.

Question 7Amanda’s response in paragraph 35 is important because it shows –

A that she feels sympathy for Deanna

She shakes her head in sympathy, which means she has sympathy for her.

Sam

ple

Do Not

Dup

licat

e

Page 62: by GF Educators, Inc. Seventh Grade Reading Book … Sample Books/7th Reading Practice...Literary Text/Drama: Full length passage #2 ..... 54-58 Sample Reporting Category 2 Answer

58Seventh Grade Reading © 2016 GF Educators, Inc.

8 Which line does the playwright include to show that Amanda is surprised?

A Amanda: Freddy—the boy you were engaged to before the war?

right AnsWer: The fact that she asks this as a question shows her surprise about Freddy.

B Amanda: Deanna, what are you thinking about? You seem lost in time.

Wrong AnsWer: This shows that Amanda is concerned about her friend.

C Amanda: (speaking softly) And he never did.

Wrong AnsWer: This does not have enough detail to reveal anything about Amanda.

D Amanda: Reckon we’ll have to shop in San Marcos now.

Wrong AnsWer: This just shows that Amanda and Deanna have to find a new place to shop.

Exercise Reasoning

9 Deanna: (sternly interrupting) THAT’S RIGHT Joey and I got married and THAT’s supposed to make everything FINE. (Deanna faces Amanda, both hands thrown down, crying harder).

What is ironic about Deanna’s statement about marrying Joey?

A Deanna does not want to reveal that she really did not want to marry Freddy.

Wrong AnsWer: This does not indicate that she did not want to marry Freddy. It only reveals that marrying Joey was supposed to solve her problems.

B Deanna lies about her feelings to Amanda. Wrong AnsWer: She is not lying at this time.

C Deanna realizes that marrying Joey may not have been what she wanted.

right AnsWer: The bold capitals of THAT’s and FINE shows that Deanna may have been wrong. And the fact that she cries.

D Deanna has Joey and their children even though she doesn’t have Freddy.

Wrong AnsWer: This fact isn't mentioned in this statement.

Sam

ple

Do Not

Dup

licat

e

Page 63: by GF Educators, Inc. Seventh Grade Reading Book … Sample Books/7th Reading Practice...Literary Text/Drama: Full length passage #2 ..... 54-58 Sample Reporting Category 2 Answer

59 © 2016 GF Educators, Inc.Seventh Grade Reading

Seventh Grade Reading Answer Key

Literary Text/Multiple Themes Page 271 C2 Paragraph 1 supports the first theme, and paragraph 2 supports the second theme.

Page 281 C2 A3 Change of power: She at least acknowledged that others would now have to share the meal preparation: she hadn’t room to dictate the entire menu. Change vs. tradition: This meant instead of the standards, Health Kick Aunt Ouida brought kale, which is not as bad as you think, and there was homemade vanilla ice cream for dessert because Sister Netty couldn’t shut up about her new ice-cream maker.

Literary Text/Conventions in Myths and Epic TalesPage 301 B2 B3 D4 A5 C

Page 311 C2 D3 B

Literary Text/Influence of Place and Time on ThemePage 321 D2 B3 A

Page 331 C2 D3 A

Page 341 B2 D3 A4 B

Page 351 B2 D3 The physical act of the curtains rising affected Crystal, but her preparation kept her calm and collected.

Literary Text/Sensory LanguagePage 361 C2 D

Page 371 A2 C3 A4 C

Page 381 B2 C3 D

Page 391 A2 C3 A

Page 401 D2 A3 D4 C

Page 411 Each game a player is involved in is new. There are never two exactly the same. So he means that everything Tex was about to do was something he had never done before.2 C3 B

Literary Text/Sensory Language: Full-Length Passage #1Pages 43-441 B2 A3 A4 D5 A6 C

Literary Text/Media Literacy Page 471 A2 D3 A4 D5 B

Page 491 D2 A3 C4 B5 A6 C

Literary Text/Media Literacy: Full-Length Passage #1Pages 50-511 C2 B3 B4 C5 D6 A

Literary Text/Fiction: SettingPage 521 A2 B3 D

Page 531 D2 C

Literary Text/Fiction: Setting Page 541 B2 B3 The poster gave Rachel something on which to focus.

*All page numbers refer to the Student Edition

Reporting Category 2 Warm-Ups and Selections

Sam

ple

Do Not

Dup

licat

e

Page 64: by GF Educators, Inc. Seventh Grade Reading Book … Sample Books/7th Reading Practice...Literary Text/Drama: Full length passage #2 ..... 54-58 Sample Reporting Category 2 Answer

60Seventh Grade Reading © 2016 GF Educators, Inc.

Seventh Grade Reading Answer Key

Page 551 C2 B3 A4 Several times Marty calls to Caitlin to get started on the chores. Because Caitlin ignores him, they do not finish their chores. Because of this, they have sandwiches and not pizza for dinner.5 Because Caitlin was older, Marty chose to not “tell” her to start the chores. If he did, the conflict would have been resolved, and the chores would have been done.6 A

Literary Text/Fiction: Point of ViewPage 581 C2 A3 D

Page 591 A2 The reader gets the full effect of LeAnn’s anguish because she is the one relating what she is feeling.3 C4 D

Literary Text/Fiction: Full-Length Passage #1Pages 60-611 A2 A3 C4 D5 C6 C

Literary Text/Fiction: Full-Length Passage #2Pages 63-651 A2 B3 C4 D5 B6 A7 A8 D9 B10 A11 CLiterary Text/Fiction: Full-Length Passage #3Pages 68-691 D2 A3 C4 A5 C6 A7 B8 C9 B

Literary Nonfiction: Autobiography & Fictionalized AccountPage 721 B2 D3 A4 D

Page 741 C2 A3 D4 “Dear Diary," and personal account5 The author of the diary is a student on the first day of school - “This is not a good way to start the first day of school.” The author of the fictionalized account wrote it from the POV of a dog - Sometimes I wonder about my owners. Like today, the girl who’s supposed to feed me—didn’t even remember.

Literary Text/Literary Nonfiction Full-Length Passage #1Pages 76-771 A2 C3 A4 D5 C6 A7 D8 C9 C

Literary Text/Literary Nonfiction Full-Length Passage #2Pages 81-821 A2 C3 B4 D5 B6 C7 B8 A9 A

Literary Text/Literary Nonfiction Full-Length Passage #3Pages 85-861 D2 B3 A4 D5 B6 D7 B8 D9 B

Literary Text/Poetry: Graphical ElementsPage 871 B2 C3 A

Page 881 D2 B

*All page numbers refer to the Student Edition

Reporting Category 2 Warm-Ups and Selections

Sam

ple

Do Not

Dup

licat

e

Page 65: by GF Educators, Inc. Seventh Grade Reading Book … Sample Books/7th Reading Practice...Literary Text/Drama: Full length passage #2 ..... 54-58 Sample Reporting Category 2 Answer

61 © 2016 GF Educators, Inc.Seventh Grade Reading

Seventh Grade Reading Answer Key

Literary Text/Poetry Full-Length Passage #1Page 901 C2 A3 D4 B5 B6 A

Literary Text/Poetry Full-Length Passage #2Pages 91-921 D2 A3 B4 A5 C6 D

Literary Text/Poetry Full-Length Passage #3Page 941 D2 A3 B4 D5 D6 C

Literary Text/Drama: Dialogue & Stage DirectionsPage 961 B2 A3 D

Page 971 A2 B3 C

Literary Text/Drama Full-Length Passage #1Page 1001 D2 A3 A4 D5 C6 D

Literary Text/Drama Full-Length Passage #2Pages 104-1051 D2 D3 C4 C5 A6 B7 A8 A9 C

*All page numbers refer to the Student Edition.

Reporting Category 2 Warm-Ups and Selections

Sam

ple

Do Not

Dup

licat

e

Page 66: by GF Educators, Inc. Seventh Grade Reading Book … Sample Books/7th Reading Practice...Literary Text/Drama: Full length passage #2 ..... 54-58 Sample Reporting Category 2 Answer

Reporting Category 3

Understanding and Analysis of

Informational Texts

Sam

ple

Do Not

Dup

licat

e

Page 67: by GF Educators, Inc. Seventh Grade Reading Book … Sample Books/7th Reading Practice...Literary Text/Drama: Full length passage #2 ..... 54-58 Sample Reporting Category 2 Answer

63 © 2016 GF Educators, Inc.Seventh Grade Reading

MARYLAND TRAVEL BLOGAlarms and Alerts Spring/Summer, 2016

1 A boom in the black bear population has forced rural homeowners and farmers in western Maryland to take aim at the problem themselves. Longtime residents say the potential for tragedy is high, especially with city dwellers headed for vacation hot spots like Deep Creek Lake where bear sightings are up dramatically.

2 “These bears will kill you over a candy bar because they want food,” says area resident Janna Barnes. “I’m worried that someone might get killed, and it will most likely be some kid walking around with a sandwich in his hand.”

3 Thanks to the state’s ban on hunting bears, the bear population is now about 400, compared with less than 200 a decade ago and up from an estimated low of 20 in 1953. The bears have no natural predators in Maryland, except cars and trucks. Western Maryland residents say the state needs to resurrect a bear-hunting season, something Maryland hasn’t had since 1953.

Informational Text/Media Literacy Full-Length Passage #1 Mixed Practice

Question 1Which synonym would best replace the word boom as it is used in paragraph 1?

C Number 3 (growth)

Substitution shows that growth fits the sentence – growth in the black bear population.

Question 3The reader can conclude that -

A food is the main attractor of bears

This sentence is evidence that the bears want food.

Question 2 The image is included in the selection most likely to –

B illustrate that many bears are coming into rural areas

The picture shows bears out of the woods and out in the open area. There are no people in the picture.Sa

mpl

e

Do Not

Dup

licat

e

Page 68: by GF Educators, Inc. Seventh Grade Reading Book … Sample Books/7th Reading Practice...Literary Text/Drama: Full length passage #2 ..... 54-58 Sample Reporting Category 2 Answer

64Seventh Grade Reading © 2016 GF Educators, Inc.

4 Which emotion is the photograph most likely intended to evoke in the reader?

A Awe at how many bears there are Wrong AnsWer: The emotion is not "wonder" at the number of bears.

B Worry for the residents of Maryland right AnsWer: Three bears, especially a mother and her cubs, should cause the people in the area to be worried about what may happen.

C Excitement about seeing black bears Wrong AnsWer: People should not be excited for bears to be living in their area.

D Disappointment in the friendliness of the bears

Wrong AnsWer: The bears are not friendly.

Exercise Reasoning

5 The homeowners took things into their hands to –

A search for bears in their neighborhood Wrong AnsWer: They did not search for the bears.

B find the bears at Deep Creek Lake Wrong AnsWer: They were not trying to find bears at Deep Creek Lake.

C stop attacks before vacationers come right AnsWer: The residents are trying to get the state to end the hunting ban on the bears.

D leave the area before the bears take over Wrong AnsWer: They did not want to leave - they wanted the bears gone before tourists arrived.

6 The author included the Janna Barnes’ comments in this post to –

A explain why black bears are dangerous right AnsWer: Janna’s statement reinforces the fact that the bears are a problem.

B imply that the state does not know what it is doing

Wrong AnsWer: Her comments did not imply this idea.

C show that children are the most vulnerable Wrong AnsWer: Any person, no matter what age, is vulnerable if they have food.

D suggest that the black bears belong in the area

Wrong AnsWer: She does not mention that black bears belonged there.Sam

ple

Do Not

Dup

licat

e

Page 69: by GF Educators, Inc. Seventh Grade Reading Book … Sample Books/7th Reading Practice...Literary Text/Drama: Full length passage #2 ..... 54-58 Sample Reporting Category 2 Answer

65 © 2016 GF Educators, Inc.Seventh Grade Reading

Informational Text/Media Literacy Full-Length Passage #2 Mixed PracticeRead the selections and choose the best answer to each question.

School Board Accepts Resignation of Travis Junior High Principal

1 School officials will be looking for a new junior

high school principal once Sabrina Rosson leaves

Feb. 5. Rosson’s letter of resignation was accepted

Monday by the district’s board of education. In her

letter, she expressed mixed feelings about leaving.

She has been hired by the Texas Education Agency

as Texas’ associate state director for STAAR testing.

Rosson is the fourth TJHS staff member to leave to

work for TEA in the past 12 months.

2 Superintendent James Hesson wished Rosson

well in her new endeavor. Rosson said she is looking

forward to gaining a new perspective on Texas

schools.

3 “I’m very excited,” Rosson said about her

new job. She will help schools develop STAAR

remediation programs, and so may visit Travis

Junior High periodically in that capacity.

Travis Junior High Principal Sabrina Rosson has resigned. Read the two passages below to see how the local newspaper reported the resignation and one citizen’s reaction to Rosson’s resignation.

Question 1The Latin root resignare, meaning "to check off, annul, cancel, give back, give up," helps the reader understand that the word resignation in paragraph 1 in the article means –

B departure

A letter of resignation is a departure from one’s job. The root definition "give up" helps the reader to infer this as the meaning.

Question 4What is most likely the reason that the author included paragraph 3 in the newspaper article?

D To give readers information about how Ms. Rosson feels

The paragraph lets the reader know that Rosson is “excited” about her new job.

Question 9 What can the reader conclude about Ms. Rosson?

B She may work at TJHS with TEA .

She could go work at TJHS as part of her job with TEA.

Question 2Which meaning of capacity is used in paragraph 3 of the article?

B Definition 2 (official role)

Ms. Rosson may be able to go to TJHS in her official role as associate state director for STAAR testing.

Sam

ple

Do Not

Dup

licat

e

Page 70: by GF Educators, Inc. Seventh Grade Reading Book … Sample Books/7th Reading Practice...Literary Text/Drama: Full length passage #2 ..... 54-58 Sample Reporting Category 2 Answer

66Seventh Grade Reading © 2016 GF Educators, Inc.

Letter to the Editor Dear Editor,

1 Once again, Travis Junior High loses a top-quality

administrator to the Texas Education Agency. I

am referring to the recent resignation of Principal

Sabrina Rosson.

2 Just last year, the TEA lured three department

heads from Travis. They were offered positions

developing the STAAR test in core subject areas.

Like Mrs. Rosson, these teachers resigned in the

winter months, leaving a huge gap in the faculty

and requiring members of the Travis staff to carry

extra responsibilities. It took months before

permanent replacements could be found,

and TJHS students did not receive the quality

education they deserve.

3 While I wish Ms. Rosson well in her new position,

the timing of her departure discourages me.

-Jane Glover, TJHS PTA President

Question 3Which synonym would best replace the word lured as it is used in paragraph 2 in the opinion?

A Number 1 (attracted)

TEA did attract the employees to leave.

Question 6 Which sentence supports the idea that the needs of TJHS students were not considered during this time?

D It took months before permanent replacements could be found, and TJHS students did not receive the quality education they deserve.

If the students did not receive a quality education, much thought did not go to them and their circumstances.

Sam

ple

Do Not

Dup

licat

e

Page 71: by GF Educators, Inc. Seventh Grade Reading Book … Sample Books/7th Reading Practice...Literary Text/Drama: Full length passage #2 ..... 54-58 Sample Reporting Category 2 Answer

67 © 2016 GF Educators, Inc.Seventh Grade Reading

Exercise Reasoning5 The reader can conclude that –

A Superintendent Hesson is upset about Ms. Rosson leaving

Wrong AnsWer: There is no information in the article or opinion that lets the reader conclude that he is upset.

B students will protest Ms. Rosson leaving Wrong AnsWer: Students protesting is not mentioned in the article or the opinion.

C Ms. Rosson feels conflicted about leaving the district

right AnsWer: The article mentions that Rosson had mixed feelings about leaving.

D Ms. Glover hopes Ms. Rosson will fail and return

Wrong AnsWer: Mrs. Glover wishes Ms. Rosson well in her new position.

7 Both Superintendent Hesson and Jane Glover –

A accepted Ms. Rosson’s resignation Wrong AnsWer: Only the Board accepted her resignation.

B hope for the best for Ms. Rosson right AnsWer: Both Hesson and Glover said they wished her well.

C are tired of teachers leaving TJHS Wrong AnsWer: Hesson never mentioned being tired of teachers leaving.

D worry about their STAAR scores Wrong AnsWer: STAAR scores were not mentioned.

8 How do the authors' purposes differ in the two news articles?

A The article provides information about the STAAR scores, and the opinion scrutinizes the scores.

Wrong AnsWer: Neither the article nor the opinion discuss STAAR scores.

B The article is written to sway people’s thoughts, and the opinion discusses loss to students.

Wrong AnsWer: The article is not persuasive.

C The article is from the superintendent’s point of view, and the opinion is from Jane Glover’s point of view.

Wrong AnsWer: The article is not from the superintendent's POV.

D The article provides information about the resignation, and the opinion shares an observation about the resignation.

right AnsWer: These are the reasons the article and the opinion were written.

Sam

ple

Do Not

Dup

licat

e

Page 72: by GF Educators, Inc. Seventh Grade Reading Book … Sample Books/7th Reading Practice...Literary Text/Drama: Full length passage #2 ..... 54-58 Sample Reporting Category 2 Answer

68Seventh Grade Reading © 2016 GF Educators, Inc.

Informational Text/Procedural Full-Length Passage #1 Mixed PracticeRead the selection and choose the best answer to each question.

Texting through the Night

1 Look around you almost any place you go. You are

probably surrounded by people who are using their

phones to talk, check their social media, and text. While

cellular phones have given us flexibility in many areas,

there are also harmful aspects to their use. If it’s not

bad enough that teens are constantly on their phones

during the day, a more alarming issue is that many

of them are texting throughout the night and causing

themselves harm by doing so. This trend is deeply

disturbing.

2 According to a 2015 report by the Pew Research Center,

88% of teens in the United States have access to a

mobile phone, and 73% of teens have a smartphone.

The study also revealed that teenagers use text

messaging more than any other form of communication.

The average number of text messages teens send

is more than 3400 per month. That’s up from an

average of 1500 messages per month in 2009. And that

number doesn’t even include the number of messages

sent through messaging services such as Facebook

Messenger™, WhatsApp™, and Snap Chat™.

Question 5Which emotion is the photograph most likely intended to evoke in the reader?

C Curiosity about the content of text messages

Seeing the girl under her covers, reading a text, and smiling makes the reader feel curious about what she is reading. Choices A and B are too broad, and choice D cannot be inferred from the article.

Question 6The image included with the article helps the reader understand –

C what teens are actually doing at night

It does show what the girl is doing at night. None of the other choices are things that can be determined from the photograph.

Sam

ple

Do Not

Dup

licat

e

Page 73: by GF Educators, Inc. Seventh Grade Reading Book … Sample Books/7th Reading Practice...Literary Text/Drama: Full length passage #2 ..... 54-58 Sample Reporting Category 2 Answer

69 © 2016 GF Educators, Inc.Seventh Grade Reading

3 All of this message sending takes time. Some people

fire off messages late at night, which not only keeps

the sender from sleeping but often wakes the recipient.

Sleep-texting is a rising phenomenon among teenagers.

Many teens send and receive text messages during the

night and then have no recollection of them when they

awaken in the morning. Is it not surprising that more

than 50% of teens sleep less than 7 hours each night,

even though it is commonly recognized that they should

be sleeping at least 9 hours?

4 Researchers are beginning to study the effects of late

night texting on teens’ sleep habits and their rates

of daytime sleepiness. A 2012 study conducted

by researchers in Belgium found that late night

texting affected the sleep cycles of nearly 45% of that

country’s teens. Nocturnal phone use has been linked

to sleep problems ranging from night-time restlessness,

insomnia, and leg pain to daytime anxiety, depression,

and learning difficulties. Similar findings were confirmed

in a recent study in Japan. And lest you think the

teens affected are already insomniacs, research reveals

that the perpetrators are more often teens keeping

other teens awake than it is poor sleepers who are

sending texts rather than counting sheep.

5 So, why should you put an end to late-night texting?

Simply put, you need better sleep. To be healthy, you

actually need more sleep than elementary school

children. Healthy, uninterrupted sleep increases

alertness and creativity, as well as helps you be more

productive and improves your general well-being. If

you suspect that your texting habits are getting in the

way of you getting a good night’s sleep, do something

proactive to improve your situation. Begin by removing

your phone from your room at night. Put it in your

backpack or let your parents put it in a safe place. Be

Question 1In paragraph 3, the word phenomenon means –

C occurrence

Sleep texting is becoming something that is occurring more often with the teens.

Question 2The Latin word nocturnus, meaning “of the night,” helps the reader understand that the word nocturnal in paragraph 4 means –

A night-time

Sleep problems, along with the meaning of the Latin root, helps the reader to infer that nocturnal means night-time.

Question 7Paragraph 4 focuses primarily on –

B research concerning teens and texting at night

This is the focus of the paragraph. The other choices only mention details in the paragraph, not the primary focus.

Sam

ple

Do Not

Dup

licat

e

Page 74: by GF Educators, Inc. Seventh Grade Reading Book … Sample Books/7th Reading Practice...Literary Text/Drama: Full length passage #2 ..... 54-58 Sample Reporting Category 2 Answer

70Seventh Grade Reading © 2016 GF Educators, Inc.

mature and set limits on yourself instead of waiting for

your parents to do so. By doing this, you will take a step

in the right direction toward becoming a responsible

decision maker.

Nine Steps to Breaking FreeFrom Your Phone’s Nocturnal Grasp

1. Set realistic limits for the amount of time you spend on your phone.

2. Don’t send emails or post on social media for at least an hour before you plan to go to bed.

3. Resist the urge to text after 10 p.m. Most things can wait until morning.

4. Turn off your phone when it is time to sleep.5. Remove the phone from your bedroom.6. Use an actual alarm clock rather than using your

phone’s alarm function.7. If you insist on having your phone at your

bedside, turn off the notifications.8. Learn how to use the “Do Not Disturb” mode on

your phone.9. Prepare yourself for symptoms of withdrawal.

This won’t be easy!

Question 3According to the information in “Nine Steps to Breaking Free From Your Phone’s Nocturnal Grasp,” when is the best time to stop sending emails?

D An hour before you go to bed

This is in Step 2.

Question 10The author organizes the selection by –

D stating a problem and then suggesting ways to resolve the problem

The article begins by describing night-time texting as the problem, and ends with a chart suggesting ways to stop the problem.

Sam

ple

Do Not

Dup

licat

e

Page 75: by GF Educators, Inc. Seventh Grade Reading Book … Sample Books/7th Reading Practice...Literary Text/Drama: Full length passage #2 ..... 54-58 Sample Reporting Category 2 Answer

71 © 2016 GF Educators, Inc.Seventh Grade Reading

Exercise Reasoning4 The author wrote this selection most likely to –

A describe the steps to stop texting at night Wrong AnsWer: The steps are just one part of the selection.

B explain why texting at night is harmful right AnsWer: The author explains the research and why texting at night is harmful.

C present research about late night texting Wrong AnsWer: Research is only one part of the selection.

D show that teens should never text at night Wrong AnsWer: This is not correct according to the passage.

8 Which sentence from the selection states an opinion?

A This trend is deeply disturbing. right AnsWer: This is only how someone feels, maybe not everyone believes this is deeply concerning.

B All of this message sending takes time. Wrong AnsWer: This can be proven.

C A 2012 study conducted by researchers in Belgium found that late night texting affected the sleep cycles of nearly 45% of that country’s teens.

Wrong AnsWer: These are the objective findings of a research study and can be proven.

D Healthy, uninterrupted sleep increases alertness and creativity, as well as helping you be more productive and improving your general well being.

Wrong AnsWer: This can be proven.

9 Read these sentences from the selection.

If it’s not bad enough that teens are constantly on their phones during the day, a more alarming issue is that many of them are texting throughout the night and causing themselves harm by doing so. (paragraph 1)

Researchers are beginning to study the effects of late night texting on teens’ sleep habits and their rates of daytime sleepiness. (paragraph 4)

What idea does the author communicate in these sentences?

A Texting at night is harming teens. right AnsWer: Both sentences have information about the harmfulness of night texting.

B Phones are constantly affecting teens. Wrong AnsWer: This topic is not addressed in these sentences.

C Daytime sleepiness is a problem for adults. Wrong AnsWer: The sentences mention daytime sleepiness, but not specifically for adults.

D Phones increase the level of harmful texting. Wrong AnsWer: The phones don't increase the levels of texting. It is how teens choose to use their phones that affects the level of harmful texting.

Sam

ple

Do Not

Dup

licat

e

Page 76: by GF Educators, Inc. Seventh Grade Reading Book … Sample Books/7th Reading Practice...Literary Text/Drama: Full length passage #2 ..... 54-58 Sample Reporting Category 2 Answer

72Seventh Grade Reading © 2016 GF Educators, Inc.

Informational Text/Procedural Full-Length Passage #2 Mixed PracticeRead the selection and choose the best answer to each question.

Designing Ideas

1 Go to any discount or department store and you will find display cases filled with jewelry. While the glitz and the glitter are certainly appealing, it all begins to look the same after a while. Did you ever long to have some accessories that are uniquely yours? If so, maybe you should consider designing your own jewelry. It’s really not as hard as you might think.

Search for Inspiration2 As in any situation where a person is designing

something new, you should begin by gathering ideas. Look at your own collection of jewelry. What things appeal to you? Are you drawn to a certain style of jewelry? You can take elements of pieces you already own and combine them to create a one-of-kind ornamental piece. Do you prefer bracelets to pendants, or are you particularly drawn to earrings? As you examine your assortment of jewelry, determine if there is a certain type of jewelry you might need. Perhaps you have several pairs of hoop earrings, but you need something different for more formal occasions. It’s always easier to justify the time and expense you’ll expend creating a new ornament when you know you really need it.

3 Don’t limit yourself to your jewelry. Study the jewelry of your family members and friends. Do they have pieces you particularly like? Look on television, in magazines and catalogs, and on the Internet. Visit sites like Pinterest and Etsy to see what other people are making and selling. Go to stores to see what’s trending this season. Decide if you like vintage, classic, or trendy

Question 1Which synonym would best replace the word accessories as it is used in paragraph 1?

C Number 3 (adornments)

Jewelry is an adornment for a person. So this is the best word to replace accessories.

Question 2Which meaning of ornamental is used in paragraph 2 of the selection?

B Definition 2 (decorative)

Jewelry is decorative, so that is the best definition for the word as it is used in the sentence.

Question 6The author includes questions in paragraph 2 most likely to –

D prompt the reader to think about what to make

The questions are trying to guide readers to decide what their area of interest may be in order to craft something in which they have an interest.

Question 4What emotion is the photograph most likely to evoke in the reader?

C Excitement about starting a new business

The picture paired with the selection indicates an excitement at starting a new business.

Sam

ple

Do Not

Dup

licat

e

Page 77: by GF Educators, Inc. Seventh Grade Reading Book … Sample Books/7th Reading Practice...Literary Text/Drama: Full length passage #2 ..... 54-58 Sample Reporting Category 2 Answer

73 © 2016 GF Educators, Inc.Seventh Grade Reading

styles. Armed with all of this information, you’ll be ready to proceed to the next step of jewelry design.

Select Your Materials4 Most jewelry is made from a combination of metals

such as clasps, chains, and wires, which form the foundation for the piece of jewelry and hold the materials together. You might want to use beads or gemstones. This is very common when making rings and pendant necklaces. Depending on the style of the jewelry you are making, you may choose to use other elements such as twine, natural fibers, wood, seedpods, or other natural materials.

5 While you will probably have access to a greater variety of materials and you might even save a little money if you search websites for jewelry supplies, it is usually best for a novice designer to visit local craft stores or specialty shops. In larger cities, you can find entire stores that are exclusively devoted to the sale of beads and beading supplies and accessories. One of the benefits of patronizing a brick-and-mortar store is that you can get expert advice from the store’s staff. They will steer you to the best materials for your products and help you avoid costly mistakes. Specialty stores often offer classes from beginner levels all the way to advanced where you can work on projects with the guidance of an expert. An additional benefit is meeting other people with whom you have a common interest.

Make It Your Own6 As you learn new techniques and methods, you will

probably become more comfortable in creating your own designs. Keep pencil and paper close at hand so you can sketch out your designs based on your inspirations. Use graph paper and pencil to be sure your creations are balanced and to help you in determining the materials you need for your project. You will follow these designs as you fabricate your finished product. With any luck, you’ll create unique jewelry that will be

the envy of all your friends.

Question 8Paragraph 6 is mainly about the –

D reasons to make your own designs

Being creative and comfortable, using your own inspiration, and creating something unique are all reasons to make your own design.

Choices A & B are details, not the main idea. Choice C is incorrect because Pinterest and Etsy are not mentioned in paragraph 6.Sa

mpl

e

Do Not

Dup

licat

e

Page 78: by GF Educators, Inc. Seventh Grade Reading Book … Sample Books/7th Reading Practice...Literary Text/Drama: Full length passage #2 ..... 54-58 Sample Reporting Category 2 Answer

74Seventh Grade Reading © 2016 GF Educators, Inc.

How to Make Beaded Necklaces1. Choose a length for your

necklace and purchase that much beading thread, plus a few extra inches.

2. Make sure you have enough beads, including spacer beads (the small beads that go between the larger beads). You will also need a clasp and two bead tips, which are used to secure the ends of your necklace so the clasp can be attached.

3. Lay your beads out on a flat surface atop a towel to keep them from rolling away. Determine the design of your necklace and in what order the beads will be strung.

4. Knot a thread end and use scissors to trim the excess thread. The knot should be at the very end of the thread, but not so close that it runs the risk of unknotting.

5. Place a small amount of craft glue on the knot to keep it from coming undone and allow it to dry.

6. Push the unknotted end of your thread through one open end of the bead tip and thread it through the hole. Pull the thread all the way through so that the knot you made rests firmly against the hole.

7. String your beads and then thread on the other bead tip so that the open end of the tip faces away from the strung beads.

8. Tie a loose knot in the thread end and use a long, straight pin to push the knot down into the open end of the bead tip. The knot should rest securely against the hole. Trim the excess thread and use pliers to close the bead tip.

9. Attach one clasp piece onto one bead tip hook so that the loops at the end of each are joined. Use the pliers to close one ring over the other. Repeat with the remaining clasp piece at the other end of the necklace.

Question 3What is the most likely reason the author has included “How to Make Beaded Necklaces” in the selection?

A To instruct readers about how to make a necklace

HINT: Look at the title!

The author provided this information for anyone wanting to start creating necklaces.

There are no bracelet directions, and describing beads is only a part of the selection. For choice D to be correct, this part of the selection would have to be a narrative. Instead, this is a procedural text.

Question 5The reader can conclude that the people completing the project in “How to Make a Beaded Necklace” would learn how to make it at –

B an arts and crafts store

Arts and Crafts stores sell beads and wire and provide classes on jewelry making.

The other 3 choices are not places where one can buy materials for or take classes in jewelry making.Sa

mpl

e

Do Not

Dup

licat

e

Page 79: by GF Educators, Inc. Seventh Grade Reading Book … Sample Books/7th Reading Practice...Literary Text/Drama: Full length passage #2 ..... 54-58 Sample Reporting Category 2 Answer

75 © 2016 GF Educators, Inc.Seventh Grade Reading

Exercise Reasoning7 Which sentence from the selection suggests that the author thinks you should research

before you start making jewelry? A Go to any discount or department store and

you will find display cases filled with jewelry.Wrong AnsWer: Looking at cases does not help you see what people are interested in, just what the store sells. Not only that, these are mass produced items.

B Visit sites like Pinterest and Etsy to see what other people are making and selling.

right AnsWer: When you are looking at Etsy and Pinterest, you are researching what items are currently being made.

C Decide if you like vintage, classic, or trendy styles.

Wrong AnsWer: This does not entail research. You only need to answer a question.

D An additional benefit is meeting other people with whom you have a common interest.

Wrong AnsWer: Meeting people does not necessarily mean you are researching them.

9 Based on the information in the selection, which of these places would the author most likely visit?

A Superstore Wrong AnsWer: While a superstore carries both jewelry and craft supplies, it is unlikely that it will have a great collection of jewelry making supplies.

B Department store Wrong AnsWer: Department stores usually sell jewelry, but they don't sell craft supplies.

C Jewelers’ supply store right AnsWer: A jewelry supply store would be the best place to visit since it would have the best selection of jewelry making supplies, as well as having employees who can give helpful advice.

D Bookstore Wrong AnsWer: A bookstore might sell books on designing and making jewelry, but the store won't have crafting supplies.

10 The selection is organized by —

A describing the process of starting a crafting business

right AnsWer: The selection relates the process of what needs to be accomplished to start a craft business - specifically a jewelry business.

B stating general information about crafting businesses

Wrong AnsWer: The selection contains specific, not general, information.

C outlining the development of a business from start to finish

Wrong AnsWer: There is no outline on the development of a business. There is no business plan, how to get funding, etc.

D comparing how to start a craft business to how to start an Etsy store

Wrong AnsWer: There is no comparison of a craft business to an Etsy store.

Sam

ple

Do Not

Dup

licat

e

Page 80: by GF Educators, Inc. Seventh Grade Reading Book … Sample Books/7th Reading Practice...Literary Text/Drama: Full length passage #2 ..... 54-58 Sample Reporting Category 2 Answer

76Seventh Grade Reading © 2016 GF Educators, Inc.

Informational Text/Procedural Full-Length Passage #3 Mixed PracticeRead the selection and choose the best answer to each question.

Communication Skills

1 Communication is the giving or exchanging of

information between two people. People communicate

their thoughts, feelings, and opinions through sending

and receiving messages. The sender is the person who

sends the messages to someone else. For example,

Mary Beth wants to borrow some of her sister’s clothes,

so she asks her. The message is the information, which

is sent. In this case, the message is that Mary Beth

is asking Claire if

she can borrow

her jacket. The

receiver is the

person who gets

the message. In

this situation, Claire

is the receiver. The

answer the receiver

gives is known as the feedback. Here, Claire may tell

Mary Beth yes or no to her request to borrow a jacket.

Regardless of what the answer is, the response is still

known as feedback. If any of these four elements is

missing, communication is blocked. If the sender does

not send the message clearly, then the receiver will

not understand it. If the receiver is not listening

carefully, then the feedback may not be complete

or correct.

2 There are two main types of communication. Verbal

communication is when you use words to express

your ideas or feelings. Verbal communication can be

spoken, or it can be written. The words that are used

and the tone of voice can affect the way a message is

C GF Educators, Inc.

Figure 1

THE PROCESS OF COMMUNICATIONWHAT MARY

BETH SAYSSENDER

(MARY BETH)

RECEIVER(CLAIRE)

CLAIRE’SRESPONSE

Question 2From the information in the “Process of Communication” diagram, you can determine that –

A Mary Beth sends a message to Claire

The arrows show that Mary Beth sends a message to Claire.

Question 1The word feedback in paragraph 1 means –

B response

When you answer a question or message, you are responding.

Question 4According to the author, feedback may not be complete or correct because—

D the receiver is not listening carefully

This statement is the evidence supporting Choice D.

Question 6The primary purpose of the "Process of Communication" diagram is to —

A illustrate the communication process

Always look at the title of a diagram. This diagram shows how two people communicate.

Sam

ple

Do Not

Dup

licat

e

Page 81: by GF Educators, Inc. Seventh Grade Reading Book … Sample Books/7th Reading Practice...Literary Text/Drama: Full length passage #2 ..... 54-58 Sample Reporting Category 2 Answer

77 © 2016 GF Educators, Inc.Seventh Grade Reading

received. If words are spoken in a loud, angry manner,

you communicate your message forcefully. It is often

more difficult to convey your emotions in written form

because then you cannot change your tone or the

volume of your voice. When you use your body to

send a message, this is non-verbal communication.

Your hand gestures, facial expressions, posture, and

movements can express non-verbal communication. All

of these show your attitude and feelings. When you

fold your arms across your chest and cross your legs

during a conversation, your actions indicate that you are

not accepting what you are being told, or that you are

closed to the message. Actions such as these can show

your attitude and feelings. Non-verbal messages can

sometimes be clearer than verbal messages. Without

your ever having to say a word, someone can know you

are happy by looking at the smile on your face and the

sparkle in your eyes. People usually communicate using

both verbal and non-verbal communication at the same

time.

3 If you want to develop good communication skills,

it will take practice. You must send your message

clearly and ask questions to be sure the receiver

understands your message. You must also learn to

listen well when someone is talking and ask questions

when you do not understand something.

Question 8The reader can conclude that good communicators –

C express themselves clearly and are good listeners

These statements from the passage support this conclusion.

Sam

ple

Do Not

Dup

licat

e

Page 82: by GF Educators, Inc. Seventh Grade Reading Book … Sample Books/7th Reading Practice...Literary Text/Drama: Full length passage #2 ..... 54-58 Sample Reporting Category 2 Answer

78Seventh Grade Reading © 2016 GF Educators, Inc.

Steps to Improve Your Communication Skills

1 Show that you respect others by listening to them and talking in a polite manner. Do not look around at other things in the room or occupy yourself with other activities while you are involved in a conversation.

2 Never shout or use improper language. Cursing and vulgar language should not be a part of your conversations.

3 Carefully listen to what the other person is saying. Do not interrupt or decide what the person is saying before he or she has finished talking. Give the other person the opportunity to fully explain her position or feelings.

4 If you do not understand, ask questions. Remember, asking questions indicates that you are interested in what the other person is saying.

5 Do not dominate the conversation by taking it over. Let the other person have plenty of opportunities to respond to what you say.

6 Avoid criticism that is negative and general. Never attack the person. Instead, discuss the action. For example, rather than saying, “You ruined everything!” try to give a specific example of what is wrong and then say, “What do you think we can do to solve this problem?”

7 Accept responsibility for the things that you have said and done. Do not blame others for your actions.

8 Set a goal of enjoying, understanding, and trusting others. Then work toward achieving that goal.

4 How well you get along with others around you is greatly

influenced by the manner in which you communicate. It is

important for you to learn to express yourself clearly and

be a good listener. It takes attention and effort to develop

good relationships with others. Setting a goal of becoming

a good communicator is the first step toward developing

good relationships. If you make the effort to improve your

communication skills while you are young, you will find

that you will benefit from your efforts in all of your adult

relationships.

Question 5The most likely reason that the author has included "Steps to Improve Your Communication Skills" in the selection is to –

C give readers information on how to communicate effectively

Following these steps will help the reader be a better communicator.

This section is not about the study of communication, nor does it compare two forms of communication. Although there are some poor communication characteristics in the "Steps," that is not why this section is included.

Question 3Which meaning of dominate is used in Step 5 of "Steps to Improve Your Communication Skills"?

A Definition 1 (to rule over; govern; control)

If you are dominating the conversation, you are trying to control it.

Sam

ple

Do Not

Dup

licat

e

Page 83: by GF Educators, Inc. Seventh Grade Reading Book … Sample Books/7th Reading Practice...Literary Text/Drama: Full length passage #2 ..... 54-58 Sample Reporting Category 2 Answer

79 © 2016 GF Educators, Inc.Seventh Grade Reading

Exercise Reasoning7 Which sentence from the selection states an opinion?

A People communicate their thoughts, feelings, and opinions through sending and receiving messages.

Wrong AnsWer: This is a statement of fact concerning the ways people communicate.

B Verbal communication is when you use words to express your ideas or feelings.

Wrong AnsWer: This statement defines verbal communication. A definition is a statement of fact.

C When you use your body to send a message, this is non-verbal communication.

Wrong AnsWer: This statement defines non-verbal communication. A definition is a statement of fact.

D How well you get along with others around you is greatly influenced by the manner in which you communicate.

right AnsWer: The word "greatly" indicates that this is an opinion.

9 The main idea of this selection is –

A you should ask questions if you do not understand

Wrong AnsWer: This is a detail from the selection. It is not the main idea of the entire selection.

B good communication is important in building good relationships

right AnsWer: The whole selection is about being able to communicate and that to have good relationships, you must have the ability to communicate well.

C you must accept responsibility for the things you say

Wrong AnsWer: This is a detail from the selection. It is not the main idea of the entire selection.

D communication can be both verbal and non-verbal

Wrong AnsWer: This is a detail from the selection. It is not the main idea of the entire selection.

10 The reader can conclude that the author –

A believes in showing respect when communicating

right AnsWer: Everything that the author discusses has to do with respect. Listening carefully, proper language, being careful of non-verbal language, and tone are several of the things mentioned.

B thinks taking over conversations is important

Wrong AnsWer: This would be the exact opposite of what the author believes. See step 5 in the graphic.

C agrees with using only verbal communication Wrong AnsWer: The author writes about properly using both verbal and non-verbal communication.

D recommends shouting over someone to be heard

Wrong AnsWer: Step 2 of the graphic shows that the author would not agree with this statement.

Sam

ple

Do Not

Dup

licat

e

Page 84: by GF Educators, Inc. Seventh Grade Reading Book … Sample Books/7th Reading Practice...Literary Text/Drama: Full length passage #2 ..... 54-58 Sample Reporting Category 2 Answer

80Seventh Grade Reading © 2016 GF Educators, Inc.

Informational Text/Expository Full-Length Passage #1 Mixed PracticeRead the selection and choose the best answer to each question.

The Evolution of the Hair Dryer

1 In the 1890s, French hairdresser Alexandre Godfrey invented the first hair dryer for use in his hair salon. His large, seated hair dryer consisted of a cap that was attached to the chimney pipe of a gas stove. It was not portable or handheld, but instead could only be used by having the person sit underneath it.

2 The idea of blow-drying hair at home originated in vacuum cleaner advertisements. An early 1900s advertisement for the Pneumatic Vacuum Cleaner showed a woman seated at her bathroom counter, drying her hair with a hose connected to the vacuum’s exhaust. The caption of the ad assured readers that while the front end of the vacuum sucked up and safely trapped dirt, the back end generated a “current of pure, fresh air from exhaust.” This ad planted the idea of blow-drying hair in the minds of the American public. A 1924 issue of Popular Mechanics instructed women how to make a hair dryer from a vacuum cleaner and an electric toaster.

3 The modern electric hair dryer is the offspring of two unrelated appliances — the vacuum cleaner

Question 1Which synonym would best replace the word cap as it is used in paragraph 1 of the passage?

C Number 3 (bonnet)

Bonnet is the best substitution for cap in the sentence.

Question 6The primary purpose of the diagram next to paragraph 2 is to -

A illustrate how a vacuum and toaster were used to create a blow dryer

Examine the diagram carefully. This picture is used to show how to make a hair dryer with a toaster and a vacuum.The type of vacuum cleaner is not listed, nor does the diagram mention different types of toasters. Choice D is incorrect since there are no steps listed.

Sam

ple

Do Not

Dup

licat

e

Page 85: by GF Educators, Inc. Seventh Grade Reading Book … Sample Books/7th Reading Practice...Literary Text/Drama: Full length passage #2 ..... 54-58 Sample Reporting Category 2 Answer

81 © 2016 GF Educators, Inc.Seventh Grade Reading

and the blender. The development of a hand-held electric hair dryer was delayed by the absence of a small, efficient, low-powered motor. In 1910, this motor was developed for small appliances such as a blender. Thus, the hot-air exhaust of the vacuum cleaner was combined with the compact motor of the blender to produce the modern hair dryer. The first hair dryers were difficult to handle, heavy, and frequently overheated. However, it was more convenient for styling hair than the vacuum cleaner.

4 In the 1930s and 1940s, the hair dryer was greatly improved. The main focus for improving hair dryers was improving the wattage and the exterior materials. Since early hair dryers only operated on 100 watts of power, it took a long time to dry the hair. Modern hair dryer motors use up to 2000 watts of power and are much more efficient. Newer models also allowed for different temperature settings and speeds. Lightweight plastic models appeared on the market in the mid-twentieth century. These lightweight models were the best ones so far. Hair dryers were popular with women from their introduction, but it wasn’t until the late 1960s, when men began to experience the difficulty of drying and styling long hair, that the market for hair dryers rapidly expanded. As improvements have

been made, hair dryers have changed from noisy, bulky, heavy contraptions to the powerful and streamlined plastic models that people use today.

Question 2What does convenient mean in paragraph 3?

C useful

By substitution, the word useful is the best choice for a synonym.

Question 4Which of the following belongs in the empty box?

A A motor was developed for small appliances

Paragraph 4 states that this occurred in 1910. Godefrey developed his hair dryer in the 1890s. The referenced Popular Mechanics came out in 1924.Choices B and D occurred after the 1924 Popular Mechanics article.Choice C contains incorrect information.

Sam

ple

Do Not

Dup

licat

e

Page 86: by GF Educators, Inc. Seventh Grade Reading Book … Sample Books/7th Reading Practice...Literary Text/Drama: Full length passage #2 ..... 54-58 Sample Reporting Category 2 Answer

82Seventh Grade Reading © 2016 GF Educators, Inc.

Exercise Reasoning3 Which sentence from the selection states an opinion?

A It was not portable or handheld, but instead could only be used by having the person sit underneath it.

Wrong AnsWer: This statement is a fact that can be proven.

B This ad planted the idea of blow-drying hair in the minds of the American public.

Wrong AnsWer: This statement is a fact that can be proven.

C The modern electric hair dryer is the offspring of two unrelated appliances — the vacuum cleaner and the blender.

Wrong AnsWer: This statement is a fact that can be proven.

D These lightweight models were the best ones so far.

right AnsWer: Being the “best” model would be an opinion. Everyone may not believe this is a fact.

5 The main idea of the selection is –

A most modern appliances were developed in the twentieth century

Wrong AnsWer: This selection is only about one appliance – the hair dryer.

B no one was interested in hair dryers until they were made of plastic

Wrong AnsWer: Plastic hair dryers are mentioned in the selection, but they are not the focus of the selection. This is a detail.

C men demanded hair dryers because they are vainer than women

Wrong AnsWer: This is not mentioned in the selection.

D when appliances are improved, they gain in popularity

right AnsWer: Use of the hair dryer began in the early 1900s and continually improved. As a result, they became more popular and useful.

7 The author organizes this selection by –

A comparing the vacuum cleaner to the blow dryer

Wrong AnsWer: The two appliances are not being compared.

B describing how a toaster and vacuum cleaner make a blow dryer

Wrong AnsWer: This was never actually explained in the selection.

C detailing the achievements Alexandre Godefrey

Wrong AnsWer: Alexandre Godefrey was only mentioned in the first paragraph.

D relating the history of the blow dryer in chronological order

right AnsWer: The selection starts with Alexandre Godefrey in the 1890s and continues on to the improvements to the hair dryer through the 1960s.

Sam

ple

Do Not

Dup

licat

e

Page 87: by GF Educators, Inc. Seventh Grade Reading Book … Sample Books/7th Reading Practice...Literary Text/Drama: Full length passage #2 ..... 54-58 Sample Reporting Category 2 Answer

83 © 2016 GF Educators, Inc.Seventh Grade Reading

Exercise Reasoning8 The images included in the article help the reader understand –

A how a blow dryer works Wrong AnsWer: Only one image shows how a blow dryer works.

B why the invention of the blow dryer is important

Wrong AnsWer: Neither image shows why the invention of the blow dryer was important.

C how the blow dryer has changed over time right AnsWer: From the first to the second image the reader can see how the blow dryer has changed over time.

D the materials used to make a blow dryer Wrong AnsWer: The second image does not give a good idea of the material used to make the blow dryer.

9 Which of these is the best summary of the article?

A In 1890, Alexandre Godefrey invented the first blow dryer. In the decades that followed improvements were made to the original design. Today the blow dryer is streamlined and lightweight.

right AnsWer: This summary has the pertinent information from the selection.

B Popular Mechanics had the first do-it-yourself instructions on how to make a blow dryer. Since then, women have tinkered with this design until the new streamlined blow dryer of today was created.

Wrong AnsWer: Women have not tinkered with the design of the blow dryer.

C Alexandre Godefrey created a bonnet attached to a stovepipe and called it a blow dryer. Popular Mechanics made improvements to this model and sold it through its magazine.

Wrong AnsWer: This is missing a lot of information about changes after the Popular Mechanics article.

D Men finally decide that they needed a machine to style their long hair, so Alexandre Godefrey created a stovepipe blow dryer using a vacuum cleaner. Today the blow dryer is lightweight and streamlined.

Wrong AnsWer: The fact that men decided that they needed to style their hair came much later and is only a small detail.

Sam

ple

Do Not

Dup

licat

e

Page 88: by GF Educators, Inc. Seventh Grade Reading Book … Sample Books/7th Reading Practice...Literary Text/Drama: Full length passage #2 ..... 54-58 Sample Reporting Category 2 Answer

84Seventh Grade Reading © 2016 GF Educators, Inc.

Boys’ Clothing Styles

1 Clothing nonverbally communicates societal expectations and norms. What is acceptable, attractive, and reasonable is seen through children’s clothes. Clothing enables or inhibits movement. The style of clothing dictates how children are to act and look.

2 In the 1700s, young children’s clothing lacked gender distinction. In fact, even European men and women wore some gown or tunic lacking gender inclination. But when males divided their breeches, meaning their pants, everything changed. Male and female clothing became more distinctive. The breeches remained reserved for men and older boys, while women and younger boys continued wearing skirted items. Such gowns were considered appropriate for young boys.

3 Another influence on male styles was the ancient practice of swaddling babies. Originally people wrapped their infants in linen strips of cloth to immobilize them because they believed the practice was necessary to force babies’ limbs to grow straight. When medical concerns overruled and showed that this immobilization weakened the arms and legs, people stopped swaddling and placed the young children in loose-fitting, lightweight clothing that allowed freedom of movement. This became the most popular clothing style for the children. John Locke’s 1693 influential publication Some Thoughts Concerning Education highly influenced this movement. Locke’s ideas became popular and were so widely spread by other philosophers that by 1800 most English and American parents had ceased swaddling. The boys then began wearing male breeches about age five.

Informational Text/Expository Full-Length Passage #2 Mixed PracticeRead the selection and choose the best answer to each question.

Question 1Which word from paragraph 1 helps the reader understand the meaning of the word inhibits in the paragraph?

B enables

The word or lets us know that the two words are opposite. That will help the reader understand that inhibits means to not enable.

Question 2What does swaddling mean in paragraph 3?

A Wrapping

The sentence after the word swaddling is used lets the reader know that it means wrapping.

Question 3In paragraph 3, the information about swaddling suggests that it –

D damaged the babies’ limbs

The swaddling weakened the babies' arms and legs.

Question 7The ideas in John Locke’s publication, Some Thoughts Concerning Education, were –

C embraced by other philosophers

The sentence says it was "widely spread" by other philosophers, which means they embraced it.

Sam

ple

Do Not

Dup

licat

e

Page 89: by GF Educators, Inc. Seventh Grade Reading Book … Sample Books/7th Reading Practice...Literary Text/Drama: Full length passage #2 ..... 54-58 Sample Reporting Category 2 Answer

85 © 2016 GF Educators, Inc.Seventh Grade Reading

The Development of Boys’ Clothing Styles, 1500-1900

Babies and young children wore dresses. The dresses for both boys and girls were almost identical. By 5 to 7 years of age boys were put into small replicas of the styles worn by their fathers. There was little in the way of styles designed specifically with children in mind.

The fashion of the day was knee breeches for both men and boys. For the first time, some English parents at mid-century began dressing their sons in sailor suits. For the first time, some parents began to think of special styles designed specifically for children. In the latter part of the century new juvenile fashions developed, involving long trousers and frilly or ruffled open necked blouses.

Throughout the century, babies and toddlers were kept in dresses-- little different from those worn by their sisters. Both boys and girls wore white frilled lacy pantalets under their frocks which covered their legs to their ankles. Sometime between the ages of 3 to 6 years, boys were put into various styles of smocks, tunics or suits. Older boys were allowed to wear various boyish styles of suits, such as sailor suits.

Babies and toddlers were still kept in similar dresses. After they graduated from their toddler dresses at about 5 or 6 years of age, English boys were put into various styles of fancy suits, especially kilts, tunics with matching bloomers, and sailor suits. English styles greatly influence upper-class American dress and middle class Americans followed the styles adopted by the upper class.

A generation of American and British boys were introduced to fancy velvet suits, often with elaborate lace collars and trim. Some mothers added long hair and curls to complete the effect. While mothers often adored this style, it was generally despised by even the youngest boys.

Clothing styles for boys became somewhat less formal. Little boys emerged from dresses at earlier ages. Rompers for toddlers and little boys became popular nursery attire.

1500s – 1600s

1700s

Early 1800s

Mid 1800s

Late 1800s

1900

Sam

ple

Do Not

Dup

licat

e

Page 90: by GF Educators, Inc. Seventh Grade Reading Book … Sample Books/7th Reading Practice...Literary Text/Drama: Full length passage #2 ..... 54-58 Sample Reporting Category 2 Answer

86Seventh Grade Reading © 2016 GF Educators, Inc.

Exercise Reasoning4 The timeline titled “The Development of Boys’ Clothing Styles, 1500 – 1900” has been

included in the selection most likely to —

A explain the differences between boys’ and girls’ clothes

Wrong AnsWer: The table’s function is not to explain the clothing differences. It is only part of the table information.

B support the idea that boys and girls should dress the same

Wrong AnsWer: The chart does not support that the boys and girls should dress alike.

C highlight how boys’ clothing has changed throughout the centuries

right AnsWer: The dates each give specific information of how boys’ clothing changed during that time period.

D illustrate how babies and toddlers dressed Wrong AnsWer: The information in the chart is not only about babies and toddlers.

5 Which sentence from the selection states an opinion?

A Clothing nonverbally communicates societal expectations and norms.

Wrong AnsWer: This is a fact that can be proven.

B In the 1700s, young children’s clothing lacked gender distinction.

Wrong AnsWer: This is a fact that can be proven.

C This became the most popular clothing style for the children.

right AnsWer: The phrase most popular indicates this sentence is an opinion.

D Such gowns were considered appropriate for young boys

Wrong AnsWer: This is a fact that can be proven.

6 Paragraphs 1-3 are organized by –

A describing how boys’ clothing changed during the 18th century

right AnsWer: The paragraphs are about how boys’ clothing changed during the 18th century.

B comparing boys’ clothes to girls’ clothes Wrong AnsWer: No comparison is made.

C detailing the dangers of swaddling children Wrong AnsWer: Only paragraph 3 is about swaddling.

D relating the history of baby clothing from 1500-1900

Wrong AnsWer: The chart is organized in this manner, not paragraphs 1-3.

Sam

ple

Do Not

Dup

licat

e

Page 91: by GF Educators, Inc. Seventh Grade Reading Book … Sample Books/7th Reading Practice...Literary Text/Drama: Full length passage #2 ..... 54-58 Sample Reporting Category 2 Answer

87 © 2016 GF Educators, Inc.Seventh Grade Reading

Exercise Reasoning8 Which sentence expresses the main idea of the selection?

A Boys’ clothing was gender-free until time and other influences caused it to change.

right AnsWer: Boys and girls wore the same clothing until the bifurcated breeches. This and the swaddling led to the clothing changes.

B Boys’ clothing during the 1700s consisted of smocks and gowns and hasn’t changed much today.

Wrong AnsWer: Boys’ clothing has changed quite a bit. The passage includes this information in paragraph 2.

C Boys’ clothing consists of breeches and shirts that are long like gowns.

Wrong AnsWer: Boys' clothing didn’t always consist of breeches and shirts. They also wore gowns.

D With the exception of breeches, boys’ clothing has evolved little since the 1700s.

Wrong AnsWer: By looking at current clothing styles, the reader knows boys' clothing has gone through many changes since the 1700s.

9 Which of these is the best summary of the article?

A When boys split their gowns and created breeches, everything about boys’ clothing changed. John Locke encouraged this by saying swaddling was bad.

Wrong AnsWer: There are important details missing from this summary.

B Children’s clothing indicated norms and expectations, but for many years boys’ and girls’ clothing were similar. Once pants were created, this changed for men and older boys. When swaddling was discredited, younger boys started wearing breeches.

right AnsWer: This summary includes the introduction, how clothes changed for men and older boys, and the changes for younger boys.

C John Locke wrote a publication called Some Thoughts Concerning Education, which encouraged young boys to wear breeches or pants. This led to a distinction between boys’ and girls’ clothing.

Wrong AnsWer: There are important details missing from this summary.

D In the 1700s, children wore clothing that was undistinguishable between boys and girls. Then pants were created, and that changed the styles for only older boys. Mothers started dressing younger boys in velvet clothes and made them grow long hair with curls.

Wrong AnsWer: This summary includes information from the timeline.

10 Which idea is emphasized in the selection and the timeline?

A As time passed, gender-free clothing changed for boys.

right AnsWer: The timeline and the selection indicate that boys changed from wearing gowns to breeches or pants.

B Smocks and dresses remained the mainstay in boys’ clothing.

Wrong AnsWer: These did not remain mainstays. Breeches or pants were worn by boys.

C Boys’ clothing became more formal over time.

Wrong AnsWer: This information is only in the chart.

D Breeches were the only things different in boys’ clothing.

Wrong AnsWer: The chart lists other things that changed – lace collars, velvet suits.

Sam

ple

Do Not

Dup

licat

e

Page 92: by GF Educators, Inc. Seventh Grade Reading Book … Sample Books/7th Reading Practice...Literary Text/Drama: Full length passage #2 ..... 54-58 Sample Reporting Category 2 Answer

88Seventh Grade Reading © 2016 GF Educators, Inc.

Amos Stagg – A Coach for 71 Years

1 “There is no way a person could be a coach

for 71 years!” Despite my proclamation, my son,

Kenneth, assured me that most certainly this was

true. Kenneth was an avid sports fan of almost

every sport, but especially football. By age seven,

he learned many sports facts as he studied and

collected baseball and football cards. But 71 years

of coaching? He proved me wrong!

2 Amos Stagg coached for 71 years of his life

before retiring at the age of 98. Known as the

“grand old man of college football,” Stagg served

as a head coach for 54 years, spending 41 of

those years at the University of Chicago. While at

the University, he led the team to a national

championship in 1905. Chicago won the Big

Ten title in 1899, 1905, 1907, 1908, 1913, and

1924, and tied for the conference championship

in 1922. Stagg was chosen “Coach of the

Year” at age 81 and was elected to both the

College Football Hall of Fame and the College

Basketball Hall of Fame.

3 Amos Alonzo Stagg, “Lon”, was born in West

Orange, New Jersey, August 16, 1862, the fifth

of eight children. His father was a shoemaker. An

excellent baseball player, Lon was offered a full

scholarship to Dartmouth College. However, he

chose to attend Yale, where he planned to study for

the ministry. During his time at Yale, Lon had little

money and ate little more than crackers at times.

Once he was even hospitalized for malnutrition. In

Informational Text/Expository Full-Length Passage #3 Mixed PracticeRead the selection and choose the best answer to each question.

Question 1What does avid mean in paragraph 1?

A Enthusiastic

Enthusiastic is the best replacement in the passage for avid to describe Kenneth.

Question 2Paragraph 2 focuses primarily on Amos Stagg’s –

C accomplishments for his 71 years as a coach

All of these are accomplishments of Amos Staggs.

Question 3The main difference between Stagg’s college baseball career and football career is that baseball –

D provided him with a scholarship

Stagg was malnourished because he declined his scholarship and had very little money. Football was his favorite sport to coach, and he never coached baseball.

Sam

ple

Do Not

Dup

licat

e

Page 93: by GF Educators, Inc. Seventh Grade Reading Book … Sample Books/7th Reading Practice...Literary Text/Drama: Full length passage #2 ..... 54-58 Sample Reporting Category 2 Answer

89 © 2016 GF Educators, Inc.Seventh Grade Reading

spite of these challenges, Stagg pitched the Yale

baseball team to five championships. In 1888,

he and his team beat the Major League’s Boston

Braves 2-1 in an exhibition game. In his junior

year, Amos Stagg decided to play football and

became a star player. In 1889, Stagg was named to

the first All-American football team.

4 After leaving Yale, Stagg taught at the

International Young Men’s Christian Association

Training School in Springfield, Massachusetts.

It was there that he played his first game of

basketball. He also coached the YMCA College’s

football team.

5 In 1892, the University of Chicago hired Mr.

Stagg to coach its football team. He remained

there until 1932. During these 41 years, Stagg’s

leadership led the team to five undefeated seasons.

They also won numerous championships and titles.

Overall, Stagg’s teams won 314 games, lost 199,

and finished in 35 ties.

6 Coach Stagg wasn’t only known for winning.

He also pioneered the forward pass and helped

develop such mainstays of football as the huddle,

the shift play, and the use of tackling dummies

in practice. Coach Stagg didn’t limit himself to

coaching football. In 1896, he coached Chicago’s

basketball team to a victory over Iowa in a historic

first: the first college basketball game played with

five players on each side.

7 In 1932, the University of Chicago asked

70-year old Stagg to retire. However, he still

wanted to work, so he accepted a coaching

job at the University of the Pacific in

Stockton, California. In 1943, the 81-year-old

Question 7Which sentence supports the idea that Stagg felt confident in his ability to continue coaching?

D However, he still wanted to work, so he accepted a coaching job at the University of the Pacific in Stockton, California.

The fact that Staggs still wanted to coach when he was 70 years old shows that he was confident he had the ability to be a good coach.

Sam

ple

Do Not

Dup

licat

e

Page 94: by GF Educators, Inc. Seventh Grade Reading Book … Sample Books/7th Reading Practice...Literary Text/Drama: Full length passage #2 ..... 54-58 Sample Reporting Category 2 Answer

90Seventh Grade Reading © 2016 GF Educators, Inc.

Stagg led the school to a 7-2 record. He was

named college football’s “Coach of the Year” and

“Football Man of the Year.” Three years later,

the University asked him to retire and accept

an advisory position. Still wanting to coach,

Stagg joined his son, Amos Jr., the head coach at

Susquehanna College in Pennsylvania. He served as

the offensive coordinator and coaching advisor until

he was 90.

8 In 1953, when Stagg’s wife was unable to travel

anymore, he accepted an advisory position at

Stockton Junior College. In 1960, he wrote, “For

the past 70 years, I have been a coach. At 98

years of age, it seems like a good time to stop.”

Stagg retired after 71 years of coaching college

football.

Sam

ple

Do Not

Dup

licat

e

Page 95: by GF Educators, Inc. Seventh Grade Reading Book … Sample Books/7th Reading Practice...Literary Text/Drama: Full length passage #2 ..... 54-58 Sample Reporting Category 2 Answer

91 © 2016 GF Educators, Inc.Seventh Grade Reading

Exercise Reasoning4 The author wrote this selection most likely to –

A recount the history of Amos Stagg’s career right AnsWer: The selection tells of Stagg’s college career and his coaching career.

B describe the role of the Halls of Fame Wrong AnsWer: The Hall of Fame is mentioned in the selection, but not the role they play.

C compare Stagg’s time in football, baseball, and basketball

Wrong AnsWer: Although Stagg played all three sports, the selection doesn't focus on comparing his time in these sports.

D explain how someone is nominated to a collegiate Hall of Fame

Wrong AnsWer: The nomination process is not explained in the selection.

5 Which sentence from the selection is an opinion?

A “There is no way a person could be a coach for 71 years!”

right AnsWer: The tone of the statement and the phrase "there is no way" make this an opinion.

B During his time at Yale, Lon had little money and ate little more than crackers at times.

Wrong AnsWer: This is a statement of fact that can be proven.

C Three years later, the university asked him to retire and accept an advisory position.

Wrong AnsWer: This is a statement of fact that can be proven.

D “For the past 70 years, I have been a coach.”

Wrong AnsWer: This is a statement of fact that can be proven.

6 Which of these is reinforced throughout the selection?

A It took Stagg many years to become a good coach and win championships.

Wrong AnsWer: The selection does not show that it took him long to become a good coach.

B The University of Chicago gave Stagg the best opportunity.

Wrong AnsWer: Although it may have been the best opportunity, there are other schools mentioned in the selection.

C Stagg’s coaching years were productive, and he was honored for them.

right AnsWer: The selection states that Stagg coached at a variety of colleges, his teams did well, and he was honored.

D Kenneth exaggerated about Stagg and was unable to prove his claims.

Wrong AnsWer: Kenneth did not exaggerate; he proved that Stagg was a coach for 71 years.

Sam

ple

Do Not

Dup

licat

e

Page 96: by GF Educators, Inc. Seventh Grade Reading Book … Sample Books/7th Reading Practice...Literary Text/Drama: Full length passage #2 ..... 54-58 Sample Reporting Category 2 Answer

92Seventh Grade Reading © 2016 GF Educators, Inc.

8 Read these sentences from the selection.

In 1889, Stagg was named to the first All-American football team. (paragraph 3)He was named college football’s “Coach of the Year” and “Football Man of the Year.(paragraph 7)

What idea does the author communicate with these sentences?

A The NFL thought that Stagg was an excellent player.

Wrong AnsWer: The NFL is not mentioned in this selection.

B Stagg was rewarded as both a player and coach.

right AnsWer: The first sentence shows Stagg was rewarded as a player, and the second sentence shows he was rewarded as a coach.

C The College Board rewarded Stagg as a coach.

Wrong AnsWer: This sentence does not address the information in the first sentence.

D Stagg has become a legend in sports history.

Wrong AnsWer: Even though the reader can infer from the passage that Stagg was a legend, this is not the information communicated by the two sentences.

9 The reader can conclude that Amos Stagg’s first coaching job –

A was his only coaching position Wrong AnsWer: Stagg also coached at the University of the Pacific, Susquehanna College, and Stockton Junior College.

B provided him with a great retirement Wrong AnsWer: The quality of Stagg's retirement is not mentioned.

C was successful for 41 years, but was not his only coaching job

right AnsWer: Stagg had a 41-year career at the University of Chicago and then went on to coach at other schools.

D offered him the opportunity to coach the sport for which he received a scholarship

Wrong AnsWer: In Chicago, Stagg coached basketball and football. His scholarship was for baseball.

10 The author organizes this selection by –

A relating Stagg’s career from a college student to coaching to retirement

right AnsWer: This selection is a chronology of Stagg's career from college until his final retirement.

B describing the awards Stagg earned, how he earned them, and where he earned them

Wrong AnsWer: This is included in the selection, but it is not the organizational pattern.

C comparing Stagg’s career and its influence on his son’s career

Wrong AnsWer: While Stagg's son did become a coach, the passage does not mention Stagg's influence on his son's career.

D providing general information about Stagg and then relating specifics about each place he coached

Wrong AnsWer: The passage is written in time order, not in a general to specific organization.

Exercise Reasoning

Sam

ple

Do Not

Dup

licat

e

Page 97: by GF Educators, Inc. Seventh Grade Reading Book … Sample Books/7th Reading Practice...Literary Text/Drama: Full length passage #2 ..... 54-58 Sample Reporting Category 2 Answer

93 © 2016 GF Educators, Inc.Seventh Grade Reading

Training Like an Astronaut

1 Astronaut Mike Hopkins

diligently trained for

his 2013-2014 mission

aboard the International

Space Station. As a NASA

astronaut, his goal was to

maintain his health and

fitness continually. He and

his fellow Expedition 37

crew members prepared

their bodies for months

before their spaceflight which launched in

September of 2013. For six months, the crew

lived and worked in space at the International

Space Station. Their preparatory training included

spacecraft simulations and spacewalking. Much

exercise and healthy eating helped ensure

that the crew members were healthy and in

shape for the duration of their mission.

2 Mike grew up on a farm near Richard, Missouri.

There he kept active by playing in the creeks

and nearby woods. He played basketball in

2nd grade and began playing football in 4th

grade. Hopkins continued playing football

throughout college. Hobbies such as skiing,

backpacking, running, and CrossFit helped

to keep him active and fit. The best activity to

keep him fit was playing basketball.

3 When the Expedition 37 crew members were

in space, they exercised about two hours per day.

Informational Text/Expository Full-Length Passage #4 Mixed PracticeRead the selection and choose the best answer to each question.

Question 1The Latin word praeparat, meaning “made ready beforehand,” helps the reader understand that the word preparatory in paragraph 1 means –

B done in preparation

The training the astronauts completed was done in preparation for travel into space. The Latin root's meaning of "making ready beforehand" qualifies this as the answer.

Question 2Paragraph 2 focuses primarily on –

A Hopkins' activities as a child

The paragraph lets the reader know the activities Hopkins participated in when he was younger.Sa

mpl

e

Do Not

Dup

licat

e

Page 98: by GF Educators, Inc. Seventh Grade Reading Book … Sample Books/7th Reading Practice...Literary Text/Drama: Full length passage #2 ..... 54-58 Sample Reporting Category 2 Answer

94Seventh Grade Reading © 2016 GF Educators, Inc.

Some of the workout items on the space station

included a specially designed treadmill that used a

harness to hold the astronaut in place, an exercise

bicycle, and a one-of-a-kind exercise device.

4 Why was exercise so important to the

astronauts? While the astronauts were in space,

exercises helped limit changes to muscles, bones,

and the cardiovascular system. They participated

in studies conducted to show how the human body

reacts to the space environment. The research

also helped with medical problems here on Earth.

Astronauts better adjust to gravity when they

return to Earth if they maintain their strength

while on their missions.

5 Astronaut Mike Hopkins desired to help

young people and adults be healthy and fit.

While he was in space, he helped create a fitness

community to show how anyone can train like an

astronaut. He and his “Train Like an Astronaut”

support team posted regular updates on the

Internet. These included workout videos from

space and exercise tips from NASA trainers. The

program was open to people of all ages. Programs

like this prove that working out and staying healthy

are very important in the life of an astronaut.

Adapted from Train Like an Astronaut with Mike Hopkins, www.NASA.gov, 2013

Question 3In paragraph 4, the word adjust means –

A adapt

Analyzing the choices given in the question, adapt would be the meaning of adjust in the sentence. The astronauts were able to adjust to the gravity of the Earth when they returned.

Question 6Hopkins hopes that his training as an astronaut will –

B inspire people to be healthier

Paragraph 5 states that Hopkins' desire is to help people be healthy.

Choices A & D are not mentioned in the selection.

Although fitness videos are mentioned, they were created in space and do not indicate the possibilty of a fitness video career.

Question 4Which of these conclusions about Mike Hopkins is supported by paragraph 5?

C He was a role model for people to become healthier.

The paragraph reveals that Mike did many things to help others be healthier: fitness community, Internet updates, open to everyone. These great things shows that he is being a good role model, not just with his online presence, but as an example for others to follow.Sa

mpl

e

Do Not

Dup

licat

e

Page 99: by GF Educators, Inc. Seventh Grade Reading Book … Sample Books/7th Reading Practice...Literary Text/Drama: Full length passage #2 ..... 54-58 Sample Reporting Category 2 Answer

95 © 2016 GF Educators, Inc.Seventh Grade Reading

Exercise Reasoning

7 Read these sentences from the selection.Much exercise and healthy eating helped ensure that the crew members were healthy and in shape for the duration of their mission. (paragraph 1)Programs like this prove that working out and staying healthy are very important in the life of an astronaut. (paragraph 5)What idea does the author communicate with these sentences?

A Astronauts must be fit and healthy to travel in space.

right AnsWer: Both sentences emphasize the importance of fitness and health in the life of an astronaut.

B Exercise and good nutrition are part of the requirements to be an astronaut.

Wrong AnsWer: Exercise and nutrition are important, but the sentences don't address the requirements for being an astronaut.

C The crew members had to compete with each other during their missions.

Wrong AnsWer: There is no mention of competition among the crew members.

D The world’s best fitness experts created the exercise program used by astronauts.

Wrong AnsWer: There is no mention of the world’s best fitness experts creating the astronauts' program.

8 The author organizes this selection by –

A comparing different astronauts’ fitness programs and then telling which is the best

Wrong AnsWer: No particular programs are mentioned, just workout items.

B describing the equipment that astronauts use and then providing information about the equipment’s role in space

Wrong AnsWer: Only one paragraph mentions their equipment.

C stating facts about NASA’s space program and then discussing how it has impacted fitness programs on Earth

Wrong AnsWer: Facts about the space program are not in the selection – just health and fitness.

D explaining astronauts’ need to be healthy and fit and then using Hopkins as an example

right AnsWer: The overriding idea of health and fitness is throughout the selection, and Hopkins is shown as an example.

5 Which sentence from the selection states an opinion?

A As a NASA astronaut, his goal was to maintain his health and fitness continually.

Wrong AnsWer: This is a fact and can be proven.

B He played basketball in 2nd grade and began playing football in 4th grade.

Wrong AnsWer: This is a fact and can be proven.

C The best activity to keep him fit was playing basketball.

right AnsWer: Whether basketball was the best activity for fitness is a matter of opinion. Key in on the words "the best."

D When the Expedition 37 crew members were in space, they exercised about two hours per day.

Wrong AnsWer: This is a fact and can be proven.

Sam

ple

Do Not

Dup

licat

e

Page 100: by GF Educators, Inc. Seventh Grade Reading Book … Sample Books/7th Reading Practice...Literary Text/Drama: Full length passage #2 ..... 54-58 Sample Reporting Category 2 Answer

96Seventh Grade Reading © 2016 GF Educators, Inc.

Exercise Reasoning9 Which of these ideas is reinforced throughout the selection?

A An astronaut’s background has changed drastically over the years.

Wrong AnsWer: The background of astronauts is not discussed in the selection.

B It takes astronauts time and effort to be prepared physically to travel in space.

right AnsWer: The astronauts had to prepare themselves with healthy eating and exercise. This takes both time and effort.

C NASA’s duties to the astronauts include ensuring that the Space Station is safe.

Wrong AnsWer: The safety of the Space Station is not mentioned.

D Astronauts are required to provide a fitness community for young people and adults.

Wrong AnsWer: Nothing states that they had to create a fitness community.

10 Which sentence from the selection helps explain how astronauts help in the medical field?

A Some of the workout items on the space station include a specially designed treadmill that used a harness to hold the astronaut in place, an exercise bicycle, and a one-of-a-kind exercise device.

Wrong AnsWer: This is about the equipment on the space station.

B While the astronauts were in space, exercises helped limit changes to muscles, bones, and the cardiovascular system.

Wrong AnsWer: This is about how the astronauts’ exercise helps keep the body from too many changes.

C The research also helped with medical problems here on Earth.

right AnsWer: This lets the reader know that research helps in solving medical problems, and the astronauts are involved in the research.

D Astronaut Mike Hopkins desired to help young people and adults be healthy and fit.

Wrong AnsWer: This tells how they helped young people and adults.

11 The author wrote this selection most likely to –

A recount how Hopkins became physically fit Wrong AnsWer: This is one aspect of the selection, but it is not the primary purpose for writing it.

B describe how the astronauts created a fitness community

Wrong AnsWer: The fitness community is only a small part of the selection.

C compare Hopkins to the other astronauts Wrong AnsWer: No comparisons are made in the selection.

D explain why it is important for astronauts to be fit and healthy

right AnsWer: The selection emphasizes the need to be fit and healthy as an astronaut.

Sam

ple

Do Not

Dup

licat

e

Page 101: by GF Educators, Inc. Seventh Grade Reading Book … Sample Books/7th Reading Practice...Literary Text/Drama: Full length passage #2 ..... 54-58 Sample Reporting Category 2 Answer

97 © 2016 GF Educators, Inc.Seventh Grade Reading

Informational Text/Persuasive Supporting 7.11A Understanding the Skill

Central Arguments in Persuasive Text

In a speech, the speaker will present information that they want you to believe.

The CENTRAL ARGUMENT is the way in which a speaker presents or organizes

their argument.

The purpose of a speech is to PERSUADE others to:

• think a certain way

• believe a certain thing

• feel something

• do something

When you listen to or read a speech, ask yourself:

• What is the speaker trying to get me to think, believe, feel or do?

• What claim does the speaker make?

• By what means is the speaker trying to influence me?

Central Argument StructuresStructure Evidence Key Words Example

Cause and EffectExplaining the reasons for an argument and describing the possible outcomes

What made it happen?

Why did it happen?

• Since• Because• As a result• Caused by• Due to the fact

We should be able to have cell phones in class because they are excellent educational tools.

AnalogyComparing ideas or situations

The main topic of the speech is usually the thing that is “new and different”

• Like• So on• As if

Terrorism is like cancer.

AuthorityUsing his own authority or the authority of others to support his argument

Expert witnesses

Research findings

• Research• Studies show…• Statistics

Researchers have found that less than 50% of Americans get enough exercise. Sa

mpl

e

Do Not

Dup

licat

e

Page 102: by GF Educators, Inc. Seventh Grade Reading Book … Sample Books/7th Reading Practice...Literary Text/Drama: Full length passage #2 ..... 54-58 Sample Reporting Category 2 Answer

98Seventh Grade Reading © 2016 GF Educators, Inc.

Informational Text/Persuasive Supporting 7.11B Understanding the Skill

Rhetorical Fallacies

Rhetorical fallacies are false arguments which are based on an error

in reasoning. They are often found in persuasive writing, political speeches,

advertising, and in various forms of media.

Rhetorical Fallacies

Type Defining Characteristics Process Effect Example

Ad Hominem

Personal Attack Directed against a person and not the position they are maintaining

Casts doubt on a person’s character in order to discredit him or her.

How can you support animal rights? Right now, you’re wearing leather shoes!

Exaggeration

Stretching the Truth

A point is made by saying something that would be true, but the truth has been distorted in some way

Makes something seem better or worse than it really is

Mrs. Turner has been teaching here since the Stone Age.

Nothing can stop our team from winning the state tournament.

Stereotyping

Preconceived Notions

A group of people, a place, or an idea are lumped together

Oversimplified and generally incorrect

All Texans wear cowboy boots, own horses, and live on ranches.

Categorical Claims

Study of Relationships

Claims any given X belongs in Group Y

Telling how two things or actions relate to each other

Cheerleading isn’t a true sport.

All men are created equal.Sa

mpl

e

Do Not

Dup

licat

e

Page 103: by GF Educators, Inc. Seventh Grade Reading Book … Sample Books/7th Reading Practice...Literary Text/Drama: Full length passage #2 ..... 54-58 Sample Reporting Category 2 Answer

99 © 2016 GF Educators, Inc.Seventh Grade Reading

Josiah gave this speech for his health class. Read the speech and answer the questions that follow.

Stop Static

1 It’s summer in Texas, and the temperature is

over 100 degrees. You pull up to the gas pump to

refill your car. Rather than stand outside in the

heat, you decide to get back in your car and enjoy

the air conditioning while the pump is pumping.

What did you do wrong?

2 Even though you should turn off your engine

while refueling, the greater danger comes from

getting back in and out of your car.

3 At the gas station, I’ve seen several people do

this, and I’ve done it myself. Teenagers are the

worst at getting back in to check their phones or

GPS. After researching this topic, I realize how

dangerous it can be and want to convince you to

stay outside of your vehicle while getting gas.

4 I’m going to tell what the danger is, what causes

it, and how you can prevent it. Out of almost 17

billion fuelings a year in the United States, most

are safe events that pose no danger to consumers.

But, according to Robert Renkes, vice-president of

Informational Text/Persuasive Full-Length Passage #1 Mixed PracticeRead the selection and choose the best answer to each question.

Question 1In paragraph 2 of the selection, the word refueling means to –

A fuel again

The prefix re- means "again," so refueling is to fuel again.

Question 2Which synonym would best replace the word consumers as it is used in paragraph 4?

C Number 3 (customers)

Consumers are the customers who buy the fuel.

Sam

ple

Do Not

Dup

licat

e

Page 104: by GF Educators, Inc. Seventh Grade Reading Book … Sample Books/7th Reading Practice...Literary Text/Drama: Full length passage #2 ..... 54-58 Sample Reporting Category 2 Answer

100Seventh Grade Reading © 2016 GF Educators, Inc.

the Petroleum Equipment Institute, all motorists

need to understand that re-entering their cars will

create static electricity that can cause a fire. The

Institute has documented more than 150 incidents

of static ignition at the fuel pump in the United

States. More than half of them have been reported

since 1999. The PEI estimates that there are

hundreds of unreported incidents each year.

5 Static electricity is most often generated

when motorists get back into their vehicles

while refueling. Renkes said, “While filling up their

vehicles with gasoline, many motorists will return

to their cars to stay warm, make a phone call, or

retrieve their purse or wallet. When they slide out

of their car, static charge is generated. Then,

when they touch the nozzle, a spark can ignite

fuel vapors around the nozzle.”

6 By following safe refueling procedures, you can

lower the risk of these kinds of accidents. The

most important thing you can do is to remain

outside your vehicle when pumping gas.

7 There are many reasons you might be tempted

to get back in your car.

• You need to get your wallet or credit card. • The weather is hot or cold or rainy. • Your cell phone is ringing. • You want to hear something on the radio or

talk to a passenger in your car.

8 The average fill-up takes only two minutes.

Staying outside the vehicle lowers the risk of

any build-up of static electricity that could

be discharged at the nozzle. If you can’t avoid

getting back in your car, before you touch the

nozzle a second time, you should always use your

Question 4What evidence does the author use to support the claim that you should stay outside your car when you are fueling?

D It lowers the risk of static electricity that could cause a fire.

Lowering the risk of static electricity that would start a fire is the evidence needed to not get back into the car.

Question 3The author mentions the Petroleum Equipment Institute in paragraph 4 to –

B suggest that an authority backs up the author’s claim

In order to make a good argument in a persuasive selection, it is appropriate to have an authority to agree with your claim.

Question 6The reader can conclude that static electricity –

C can be created by going in and out of the car

These sentences are the evidence necessary to prove that you can create static electricity by going in and out of a car.

Question 8The author uses this statement to support the claim that –

C eliminating static electricity can prevent fires

Static causes the fire and staying outside the vehicle will help eliminate the static that can cause the fire.

Sam

ple

Do Not

Dup

licat

e

Page 105: by GF Educators, Inc. Seventh Grade Reading Book … Sample Books/7th Reading Practice...Literary Text/Drama: Full length passage #2 ..... 54-58 Sample Reporting Category 2 Answer

101 © 2016 GF Educators, Inc.Seventh Grade Reading

bare hand to touch a metal object that is not close to your fuel pipe. That way, if

there is a spark from the static, it won’t be around the gas fumes.

9 By staying outside your car while you get gas, you can prevent a build-

up of static electricity. If you eliminate static electricity at the pump, then you

can eliminate fires. By eliminating fires at the pump, you make refueling safer for

you, your passengers, and everyone at the gas station. Next time you put gas in

your car, think about how you can do it safely.

1. Before you get out of your car, make sure you have everything you need. 2. Stay outside your car while the gas is pumping. 3. If you have to go back in the car, make sure you touch something metal to

discharge any static electricity before you get close to the gas nozzle. 4. Be safe. Otherwise, you and your pump fire might get posted on YouTube

for the entire world to see!

Exercise Reasoning5 Which of the author’s statements is an example of stereotyping?

A Teenagers are the worst at getting back in to check their phones or GPS.

right AnsWer: This stereotypes teenagers as “the worst.”

B The most important thing you can do is to remain outside your vehicle when pumping gas.

Wrong AnsWer: No one is being portrayed stereotypically in this sentence.

C If you eliminate static electricity at the pump, then you can eliminate fires.

Wrong AnsWer: No one is being portrayed stereotypically in this sentence.

D Next time you put gas in your car, think about how you can do it safely.

Wrong AnsWer: No one is being portrayed stereotypically in this sentence.

7 With which of the following statements would the author agree?

A Station attendants should stop people when they try to get back in their cars.

Wrong AnsWer: This article is about pumping gas yourself. If an attendant pumped your gas, you wouldn’t get out of your car.

B Fill up your car at night to avoid sparks. Wrong AnsWer: Darkness does not stop static sparks.

C Summer temperatures do not affect static. Wrong AnsWer: Temperature is not mentioned as a cause of static. Also, people are more prone to get in and out of their cars on hot days.

D Gas pumps should display signs about static electricity and sparks

right AnsWer: Most people do not have this information, so it would be good to have signs posted. The author would agree.

Sam

ple

Do Not

Dup

licat

e

Page 106: by GF Educators, Inc. Seventh Grade Reading Book … Sample Books/7th Reading Practice...Literary Text/Drama: Full length passage #2 ..... 54-58 Sample Reporting Category 2 Answer

102Seventh Grade Reading © 2016 GF Educators, Inc.

Learn a Foreign Language

1 In the United States, there is a rising concern

that American citizens should be able to speak

and use languages other than English. Learning

a second language is a worthwhile pursuit. All

students should learn a foreign language

because of its educational, professional, and

social benefits.

2 People in the academic world, especially

those who work in colleges, view learning a

second language as beneficial to all students. The

knowledge of a foreign language helps to expand

and build one’s vocabulary. The discipline of having

to think, read, and write in another language helps

to develop analytical and problem-solving skills. If a

high school student is well acquainted with a foreign

language, he has the option of becoming a foreign

exchange student who can spend part of his high

school career studying in a foreign country. Because

the student can already speak the language of the

host country, he can spend the majority of his time

learning about the country’s culture and history

through direct contact with the native people there.

3 College-bound students find that most

universities require them to have at least two years

of coursework in a foreign language. Having fulfilled

this language requirement will help a student gain

entrance to top-ranked universities. Students who

speak a second language fluently also have the

opportunity to attend college abroad. For example,

if a person speaks French fluently, she could

go to college in France. The educational benefits

of learning a foreign language are far-reaching.

Informational Text/Persuasive Full-Length Passage #2 Mixed PracticeRead the selection and choose the best answer to each question.

Question 1Which meaning best fits the way abroad is used in paragraph 3?

A Definition 1 (outside one’s country; to a foreign land)

Being fluent in a language other than one's own would give them the ability to go to college in the country where that language is spoken.

Question 8Learning a second language is a worthwhile pursuit.

The author uses this statement to assert the claim that –

C all students would benefit from learning a foreign language

If you benefit from learning a second language, then it is a worthwhile pursuit.

Just because a school requires something does not make it a worthwhile pursuit.

Sam

ple

Do Not

Dup

licat

e

Page 107: by GF Educators, Inc. Seventh Grade Reading Book … Sample Books/7th Reading Practice...Literary Text/Drama: Full length passage #2 ..... 54-58 Sample Reporting Category 2 Answer

103 © 2016 GF Educators, Inc.Seventh Grade Reading

4 Being well versed in a second language has

professional benefits as well. Businesses need

people at all levels that have language skills other

than English. Companies from fast food restaurants

to high-powered international corporations and

everywhere in between, need employees who

have the ability to speak a foreign language

fluently. The ever-growing immigrant populations

in the United States need people who can speak

their native languages as well as help them learn

English. With so many companies working at an

international level, knowing a second language

can often provide an edge when applying for

a job. Foreign businesspeople are impressed

by the commitment shown by people who care

enough to learn their language. Many business

deals have been won or lost through language. The

ability to communicate in another language is a

definite asset in many fields of business.

5 Not only does knowing a foreign language have

educational and professional benefits, but it also

can be a social asset. Most social relationships are

expressed through language. A person who can

deal with a language other than English can watch

television, listen to the radio, and speak to teachers,

students, and people on the street in that language.

This person will be better equipped to travel abroad

because she can converse with others, especially in

restaurants and other public places.

6 Another rationale for learning a foreign language

is the respect it communicates for the other culture.

When Americans travel outside of the United States,

it is said we are identified by our loud clothes,

our loud mouths, and our inability to speak any

language but our own. People in other countries

are understandably offended when we expect them

to accommodate our ignorance and speak to us

Question 9The author would agree that –

C knowledge of a foreign language may influence an international business to hire someone

Knowing a foreign language would give you an edge when applying for those foreign jobs, so the author would agree that a foreign language would help you in the international business world.

Question 6A professional benefit of speaking a second language is–

C having the respect of foreign businesspeople

If you work at an international company, knowing the language would add respect to the partnership with the foreign business. Knowing a foreign language would give you an edge when applying for those foreign jobs, so the author would agree that a foreign language would help you in the international business world.

Question 2In paragraph 4, the word asset means –

C advantage

If you can speak a second language, businesses will think it is an advantage.Sa

mpl

e

Do Not

Dup

licat

e

Page 108: by GF Educators, Inc. Seventh Grade Reading Book … Sample Books/7th Reading Practice...Literary Text/Drama: Full length passage #2 ..... 54-58 Sample Reporting Category 2 Answer

104Seventh Grade Reading © 2016 GF Educators, Inc.

in English, especially if we do not even attempt

to learn their language during our travels. In an

increasingly global economy, younger United States

citizens can work to change the image of Americans

from people who are disinterested and content

to focus on themselves to an image of individuals

who are intent on learning to work with the world

outside the U.S. borders. America cannot prosper

alone any longer. She must learn to link arms

with the world at some level. One of the ways this

linkage is possible is through U.S. citizens learning

to speak other languages.

7 People who put forth the time and effort to

learn a second language most likely will grow as

individuals. They will increase their knowledge in

fun and challenging ways. These people will gain a

broader outlook on life by developing an interest

in another culture. It is a great accomplishment to

learn to speak, write and read a foreign language.

Because of its educational, professional, and

social benefits, learning a second language

can be an asset to individuals in all walks of

life, not just for college-bound students. For

these reasons, all students should be required to

learn a foreign language.

Can you identify the word in the various languages?

лед buz barafu de la glace

Russian Turkish Swahili French

Question 7The reader can conclude that learning a second language –

B is an investment in a person’s future

By learning a second language, you are investing in your future - educational, business and social.

Learning a language requires varying amounts of time for different people and is not a guarantee of success or a requirement for all colleges.

Sam

ple

Do Not

Dup

licat

e

Page 109: by GF Educators, Inc. Seventh Grade Reading Book … Sample Books/7th Reading Practice...Literary Text/Drama: Full length passage #2 ..... 54-58 Sample Reporting Category 2 Answer

105 © 2016 GF Educators, Inc.Seventh Grade Reading

Exercise Reasoning3 What evidence does the author use to support the claim that students should learn a

foreign language?A Universities require students to take a

foreign language.Wrong AnsWer: Universities requiring a foreign language is not the evidence to support the author’s claim.

B Knowing a foreign language will benefit a student educationally, professionally, and socially.

right AnsWer: Knowing a foreign language would help in study abroad, business, and help one converse in other cultures. This evidence supports the value of learning a foreign language.

C Knowledge of a foreign language will only help a student in the educational field.

Wrong AnsWer: It would help in other areas as well – business and speaking to people from other cultures.

D Learning a foreign language requires hard work on the students’ part.

Wrong AnsWer: This doesn't support the claim that a student should learn a foreign language.

4 The author mentions businesses in paragraph 4 to –

A add support to the claim that a foreign language helps beyond the education world

right AnsWer: The business applications of speaking a foreign language are an additional reason to learn one.

B imply that you can only get a job if you know a foreign language

Wrong AnsWer: The text implies that it might be easier to get a job, not that you can ONLY get a job if you know a foreign language.

C imply that knowing English is all that is necessary in the business world

Wrong AnsWer: The passage states that knowing a foreign language is becoming increasingly important in the business world.

D suggest that businesses will help employees learn a foreign language

Wrong AnsWer: There is no suggestion that this would happen.

5 Which sentence from the selection provides a categorical claim about learning a foreign language?

A All students should learn a foreign language because of its educational, professional, and social benefits.

right AnsWer: Saying that all students should learn a foreign language is making a categorical claim.

B College-bound students find that most universities require them to have at least two years of coursework in a foreign language.

Wrong AnsWer: This is not a universal claim about all universities, so it is not a categorical claim.

C Being well versed in a second language has professional benefits as well.

Wrong AnsWer: This is not a universal claim about all people, so it is not a categorical claim.

D A person who puts forth the time and effort to learn a second language will grow as an individual.

Wrong AnsWer: This is not a universal claim about all people, so it is not a categorical claim.

Sam

ple

Do Not

Dup

licat

e

Page 110: by GF Educators, Inc. Seventh Grade Reading Book … Sample Books/7th Reading Practice...Literary Text/Drama: Full length passage #2 ..... 54-58 Sample Reporting Category 2 Answer

106Seventh Grade Reading © 2016 GF Educators, Inc.

Shoot for the Stars

1 Everyone knows people who seem to have

everything going their way. They may be champion

athletes, cheerleaders, star students, student

government leaders or outstanding musicians.

They are the people whom you might envy, but

they often are some of the nicest people you know.

These young superstars share a long list of qualities

that have helped them become successful. Learning

some of their secrets will help you discover how you

too can shoot for the stars!

Be Self-Confident

2 The first step in becoming a winner is to be

confident in yourself and your abilities. In other

words, you must think like a winner if you want

to be a winner. Winners focus on the things they

can do rather than on those things they cannot

do. Everyone has good qualities and strengths.

Take a few minutes to list ten things you like

about yourself. When trying times come, and you

momentarily doubt your value and abilities, remind

yourself of the good qualities you listed. This can

be a real confidence-booster. Prove to yourself and

others that you can achieve the things that you set

out to achieve.

Set Realistic Goals

3 Before you can achieve anything, you must

have an idea of what it is that you want to do. A

predominant characteristic of successful people

is that they have solid goals in mind. The great

majority of these people set goals they can aim

for and then direct their energies toward these

goals. These goals will then help you set your

priorities, deciding which things should hold the

Informational Text/Persuasive Full-Length Passage #3 Mixed PracticeRead the selection and choose the best answer to each question.

Question 1The word confident in paragraph 2 means –

C self-assured

By using substitution, the best synonym to confident is self-assured.

Question 10In which section of the selection does the author fail to present any evidence to support his claims?

C “Be Self-Confident"

In the other sections, the stars, Olympic athletes, Nobel Prize winners and Thomas Edison are evidence to support the author's claim. This section contains a series of claims with no evidence to support them.

Question 2Which words in paragraph 3 help the reader understand what the word priorities means?

B deciding which things should hold the most important place in your life

The comma after the word priorities indicates that what is being presented after it is the meaning of the word priorities – deciding which things should hold the most important place in your life.

Sam

ple

Do Not

Dup

licat

e

Page 111: by GF Educators, Inc. Seventh Grade Reading Book … Sample Books/7th Reading Practice...Literary Text/Drama: Full length passage #2 ..... 54-58 Sample Reporting Category 2 Answer

107 © 2016 GF Educators, Inc.Seventh Grade Reading

most important place in your life. If earning a piano

scholarship for college is your goal, then practicing

the piano should be a priority in your life. Once

you set your goals, you must plan to do the things

necessary to help you achieve those goals. As

Mister Rogers says, “You’ve got to do it—every little

bit.”

Work Hard

4 It takes hours of hard work to make sure your

dreams come true. Wishing for something will never

make it happen. If you ask any Olympic athlete,

professional musician or Nobel Prize winner

what was responsible for his or her success,

you can count on the fact that each will

mention lots of hard work. Sometimes reaching

a goal requires so much hard work that you feel like

you would rather quit than continue trudging on. If

you want to be a winner you must throw the word

“quit” out of your vocabulary. Sometimes things

will come easily, and at other times you will have to

work really hard. The important thing to remember

is that you should not give up if you want to achieve

your goals.

Take Risks

5 There is something far worse than quitting:

not trying. Until you are willing to do something

you know could be difficult, you will never discover

your capabilities. The fear of failure keeps many

people from doing things they might enjoy. Remind

yourself of the saying, “It is better to have tried and

lost than never to have tried at all,” or as Thomas

Edison said, “I have not failed. I've just found

10,000 ways that won't work.”

Be Yourself

6 Winners realize that the best way to be a star

is to make sure you are yourself. Do not be afraid

to go against the crowd - be a leader instead of a

Question 3In paragraph 4, “Work Hard,” the author included examples most likely because he –

D wants to highlight people who have gained much from working hard

The accomplishments of the people in the paragraph who worked hard help show what can be accomplished by working hard.

Question 5What idea is suggested in the section entitled “Take Risks?”

B Not trying is worse than quitting.

You will discover your capabilities when you take risks and try new or difficult things.

Question 4Wishing for something will never make it happen.

The author uses this sentence to support the claim that –

A you will have to work hard to achieve your goals

You cannot just wish for things to happen – you have to work hard.There is nothing wrong with wishing for something to happen. You just have to be willing to work to make your dreams come true.

Sam

ple

Do Not

Dup

licat

e

Page 112: by GF Educators, Inc. Seventh Grade Reading Book … Sample Books/7th Reading Practice...Literary Text/Drama: Full length passage #2 ..... 54-58 Sample Reporting Category 2 Answer

108Seventh Grade Reading © 2016 GF Educators, Inc.

follower. Everyone loves a leader, and they always

stand out in a crowd. If you think, talk, dress, act,

and look like everyone else, how can you possibly

stand out and shine? You have your special star

qualities, but it is up to you to make them shine

through. If you do, your brilliance will not go

unnoticed!

Remember Numerous Stars of the Past Suffered Serious Setbacks

7 Can you name the United States President who

at age seven had to work to support his family and

whose mother died before he was ten? He lost eight

elections, lost his fiancée to the grave, and suffered

a nervous breakdown. He got our country through

the Civil War. That’s right: Mr. Abraham Lincoln.

Although failures serve to redirect our focus, it is

also true that failures cannot cause us to lose our

focus. This paradoxical truth must be accepted.

Only you and those who know you well can tell

when failure should be heeded and when failure

should be ignored. Twenty-seven publishers,

for example, turned down Dr. Seuss, before

someone at long last published The Cat in the

Hat. What if he had given up after twenty-six?

Accept failure as part of the plan and determine

whether you want to continue or alter your course.

Establish Who You Are in Something Besides Your Goal Because Life Is Unpredictable

8 Most successful people set and achieve their

goals, but you should derive who you are in

something besides your goal. For example, you

may make it your ambition to be a particular sort

of athlete. This is good. But what if you become

ill? What if your family moves frequently and you

cannot find and keep a quality coach? Be broad-

minded enough to recognize that with or without

your goals, you will still be you and have yourself,

your love, and your personality to share with the

Question 9What evidence does the author use to support the idea that you may have setbacks?

A Twenty-seven publishers turned down Dr. Seuss before he was published.

Dr. Seuss is a prolific writer. If he was turned down 27 times, we can all have setbacks.

None of the other three answer choices are examples of experiencing setbacks.

Sam

ple

Do Not

Dup

licat

e

Page 113: by GF Educators, Inc. Seventh Grade Reading Book … Sample Books/7th Reading Practice...Literary Text/Drama: Full length passage #2 ..... 54-58 Sample Reporting Category 2 Answer

109 © 2016 GF Educators, Inc.Seventh Grade Reading

meaningful people in your life. This will keep you

balanced through all of those failures we mentioned!

Stop the Comparison Trap

9 There is likely little that is more useless than

comparison. Comparison is akin to living your life

looking in a rearview mirror because it keeps your

focus on others. It is like trying to drive a car

forward while looking at things behind and beside

you. Not effective! Not only can you not control

others, but you also cannot be like others. Your

focus should face forward, on your goals. You

can work on yourself, to be motivated, clean,

straightforward, and trendsetting in your own ways.

Real stars do this every day.

Sam

ple

Do Not

Dup

licat

e

Page 114: by GF Educators, Inc. Seventh Grade Reading Book … Sample Books/7th Reading Practice...Literary Text/Drama: Full length passage #2 ..... 54-58 Sample Reporting Category 2 Answer

110Seventh Grade Reading © 2016 GF Educators, Inc.

Exercise Reasoning6 In this selection, the author claims that –

A people who are superstars have everything going their way

Wrong AnsWer: Superstars work hard to have their way, but this is not the claim being made.

B to make your own star qualities, you must shine through

Wrong AnsWer: You have to work hard to make your star qualities -- not just shine through.

C being a star means believing in yourself, setting goals, and working to achieve them

right AnsWer: This claim has been made throughout the selection. Evidence: be confident in yourself and your abilities, have solid goals in mind, and It takes hours of hard work to make sure your dreams come true.

D the only way to be a winner is to be confident in yourself and your abilities

Wrong AnsWer: This is only one of the things you must do to be a winner.

7 The author of this article is most likely to –

A work hard to reach his or her goals right AnsWer: One section of the passage is entitled "Work Hard." This would indicate the author is likely to work hard because he or she sees the value of hard work.

B set unrealistic goals Wrong AnsWer: The fact the passage has a section called "Set Realistic Goals," would mean the author is unlikely to set unrealistic goals.

C believe life is predictable Wrong AnsWer: There is a section of the passage that points out that life is unpredictable.

D compare himself or herself to others Wrong AnsWer: The last section of the passage points out the danger od comeparing yourself to others. The author is unlikely to do this.

8 Which of the author’s statements is an example of exaggeration?

A A predominant characteristic of successful people is that they have solid goals in mind.

Wrong AnsWer: Nothing in this sentence is exaggerated.

B The fear of failure keeps many people from doing things they might enjoy.

Wrong AnsWer: Nothing in this sentence is exaggerated.

C Everyone loves a leader, and they always stand out in a crowd.

right AnsWer: The words "everyone" and "always" make this sentence an exaggeration.

D Although failures serve to redirect our focus, it is also true that failures cannot cause us to lose our focus.

Wrong AnsWer: Nothing in this sentence is exaggerated.

Sam

ple

Do Not

Dup

licat

e

Page 115: by GF Educators, Inc. Seventh Grade Reading Book … Sample Books/7th Reading Practice...Literary Text/Drama: Full length passage #2 ..... 54-58 Sample Reporting Category 2 Answer

111 © 2016 GF Educators, Inc.Seventh Grade Reading

Seventh Grade Reading Answer KeyInformational Text/Media Literacy Page 1071 C2 A3 B

Page 1081 C2 A3 B4 D

Informational Text/Media Literacy Full-Length Passage #1Pages 109-1101 C2 B3 A4 B5 C6 A

Informational Text/Media Literacy Full-Length Passage #2Pages 111-1131 B2 B3 A4 D5 C6 D7 B8 D9 B

Informational Text/ProceduralPage 1141 C2 A3 D

Pages 115-1161 A2 C3 B4 B

Informational Text/Procedural/Full-Length Passage #1Pages 117-1201 C2 A3 D4 B5 C6 C7 B8 A9 A10D

Informational Text/Procedural/Full-Length Passage #2Pages 121-1241 C2 B3 A4 C5 B6 D7 B8 D9 C10 A

Informational Text/Procedural/Full-Length Passage #3Pages 125-1281 B2 A3 A4 D5 C6 A7 D8 C9 B10 A

Informational Text/Expository: Summary Page 1291 D2 B

3 C

Page 1301 C2 A3 D4 C

Informational Text/Expository: Fact, Opinion Page 1311 C2 D3 A

Page 1321 D2 C3 A

Informational Text/Expository: Organizational PatternsPages 133-1341 D2 B3 B4 D5 A6 B

Pages 135-1361 C2 C3 B4 A

Informational Text/Expository: Logical ConnectionsPages 137-1381 C2 A3 B

*All page numbers refer to the Student Edition.

Reporting Category 3 Warm-Ups and Selections

Sam

ple

Do Not

Dup

licat

e

Page 116: by GF Educators, Inc. Seventh Grade Reading Book … Sample Books/7th Reading Practice...Literary Text/Drama: Full length passage #2 ..... 54-58 Sample Reporting Category 2 Answer

112Seventh Grade Reading © 2016 GF Educators, Inc.

Seventh Grade Reading Answer Key

Pages 1391 A2 D3 B

Informational Text/Expository/Full-Length Passage #1Page 140-1431 C2 C3 D4 A5 D6 A7 D8 C9 A

Informational Text/Expository/Full-Length Passage #2Page 144-1471 B2 A3 D4 C5 C6 A7 C8 A9 B10A Informational Text/Expository/Full-Length Passage #3Page 148-1511 A2 C3 D4 A5 A6 C7 D8 B9 C10 A

Informational Text/Procedural/Full-Length Passage #4Page 152-1551 B2 A3 A4 C5 C6 B7 A8 D9 B10 C11 D Informational Text/PersuasivePage 1571 A2 B3 D

Page 1581 C2 A3 D

Pages 160-1611 B2 A3 D4 A5 C6 CPage 1621 A2 C3 C4 C

Informational Text/Persuasive/Full-Length Passage #1Pages 163-1661 A2 C3 B4 D5 A6 C7 D8 C

Informational Text/Persuasive/Full-Length Passage #2Pages 167-1701 A2 C3 B4 A5 A6 C7 B8 C9 C

Informational Text/Persuasive/Full-Length Passage #3Pages 19-201 C2 B3 D4 A5 B6 C7 A8 C9 A10C

*All page numbers refer to the Student Edition.

Reporting Category 3 Warm-Ups and Selections

Sam

ple

Do Not

Dup

licat

e

Page 117: by GF Educators, Inc. Seventh Grade Reading Book … Sample Books/7th Reading Practice...Literary Text/Drama: Full length passage #2 ..... 54-58 Sample Reporting Category 2 Answer

Reporting Categories 1 - 3

Making Connections Between and Across Texts

Paired Passages

Sam

ple

Do Not

Dup

licat

e

Page 118: by GF Educators, Inc. Seventh Grade Reading Book … Sample Books/7th Reading Practice...Literary Text/Drama: Full length passage #2 ..... 54-58 Sample Reporting Category 2 Answer

114Seventh Grade Reading © 2016 GF Educators, Inc.

Paired Passages TEKS 7.7, 7.9, Figure 19F Understanding the Skill

Paired Passages

Paired passages assess your ability to analyze materials across genres. You

are required to read two passages which are connected in some way, answer

questions about each passage, and then answer questions which compare the

two passages.

Possible Pairings

• Two myths or folktales from different cultures

• A poem and a literary nonfiction passage with similar topics or themes

• An autobiography or a diary and a fictional adaptation of it

• Two expository passages looking at similar topics from different perspectives

• An expository article and a literary nonfiction passage on the same subject

• Two persuasive passages on the same topic – one “for” and one “against.”

Strategies for Paired Passages

• Plan to read each passage more than once.

• Read the first passage carefully and answer its questions.

• Read the second passage carefully and answer its questions.

• Re-read both passages and look for the connection between the two

passages. Ask yourself, “Why are these passages paired together?”

• Look for similarities and differences between the two passages.

• In the third section of questions, you’ll have questions about both passages.

You will need to look for evidence from BOTH passages when you decide

which answer choice is the best.Sam

ple

Do Not

Dup

licat

e

Page 119: by GF Educators, Inc. Seventh Grade Reading Book … Sample Books/7th Reading Practice...Literary Text/Drama: Full length passage #2 ..... 54-58 Sample Reporting Category 2 Answer

115 © 2016 GF Educators, Inc.Seventh Grade Reading

The Three AdmonitionsA Folktale from Italy

1 A man left his country to go to foreign lands and

there entered the service of an abbot. After he had

spent some time in faithful service, he desired to

see his wife and native land. He said to the abbot,

"Sir, I have served you thus long, but now I wish to

return to my country."

2 "Yes, my son," said the abbot, "but before

departing I must give you the three hundred

ounces of gold that I have put together

for you. Will you be satisfied with three

admonitions, or with the three hundred

ounces?"

3 The servant answered, "I will be satisfied

with the three admonitions."

4 "Then listen. First: when you change the old

road for the new, you will find troubles for which

you have not looked. Second: see much and say

little. Third: think over a thing before you do it, for

a thing deliberated is very fine. Take this loaf of

bread and break it when you are truly happy."

5 The good man departed, and on his journey met

other travelers. These said to him, "We are going to

take the by-way. Will you come with us?"

6 But he, remembering the three admonitions

of his master, answered, "No, my friends, I will

keep on this road." When he had gone half way, he

heard some shots. "What was that?" The robbers

had killed his companions. "I have gained the first

hundred ounces!" he said and continued his journey.

Literary Text/Paired Passages Set #1: Two Folktales from Different Cultures Read the selections and choose the best answer to each question.

Question 1Which synonym would best replace the word admonitions as it is used in paragraph 2?

B Number 2 (warnings)

If the alternative is gold, then it is opposite of gold. Further into the story, the admonitions are listed, and they are warnings.

Question 3Why is the gold important to the story?

A It symbolizes the man’s choice to learn and not be greedy.

The man chooses not to take the gold, and instead takes the warnings. This shows he wants to learn and is not greedy.

Sam

ple

Do Not

Dup

licat

e

Page 120: by GF Educators, Inc. Seventh Grade Reading Book … Sample Books/7th Reading Practice...Literary Text/Drama: Full length passage #2 ..... 54-58 Sample Reporting Category 2 Answer

116Seventh Grade Reading © 2016 GF Educators, Inc.

7 On his way, he arrived at an inn as hungry

as a dog and called for something to eat. A

large dish of meat was brought which seemed to be

delicious. He stuck his fork in it and turned it over,

and was frightened out of his wits, for it was human

flesh! He wanted to ask the meaning of such food

and give the innkeeper a lecture, but just then he

thought, "See much and say little;" so he remained

silent. The innkeeper came, he settled his bill and

took leave.

8 But the innkeeper stopped him and said, "You

have saved your life. All those who have questioned

me about my food have been soundly beaten, killed,

and nicely cooked." "I have gained the second

hundred ounces," said the good man, who did not

think his skin was safe until then.

9 When he reached his country, he remembered

his house, saw the door ajar and slipped in. He

looked about and saw no one, only in the middle

of the room was a table, well set with two glasses,

two forks, two seats, service for two." How is this?"

he said. "I left my wife alone, and here I find things

arranged for two. There is some trouble."

10 So he hid under the bed to see what went on.

A moment later, his wife who had gone out a short

time before for a pitcher of water, entered the

room. A little after he saw a sprucely dressed young

priest come in and seat himself at the table.

11 "Ah, is that he?" and he was on the point of

coming forth and giving him a sound beating; but

there came to his mind the final admonition of the

abbot: "Think over a thing before you do it, for a

thing deliberated is very fine;" and he refrained.

12 He saw them both sit down at the table, but

before eating his wife turned to the young priest

Question 2The author uses the comparison to help the reader understand –

D that he is extremely hungry

If you call for something to eat and a large dish of meat is brought to you, then you are very hungry.

Sam

ple

Do Not

Dup

licat

e

Page 121: by GF Educators, Inc. Seventh Grade Reading Book … Sample Books/7th Reading Practice...Literary Text/Drama: Full length passage #2 ..... 54-58 Sample Reporting Category 2 Answer

117 © 2016 GF Educators, Inc.Seventh Grade Reading

and said: "My son, let us say our accustomed

prayers for your father."

13 When he heard this, he came from under the

bed crying and laughing for joy and embraced and

kissed them both so that it was affecting to see

him. Then he remembered the loaf his master had

given him and told him to eat in his happiness. He

broke the loaf and there fell on the table all the

three hundred ounces of gold which the master had

secretly put in the loaf.

Adapted from: Italian Popular Tales by Thomas Frederick Crane (London: Macmillan and Company, 1885)

Sam

ple

Do Not

Dup

licat

e

Page 122: by GF Educators, Inc. Seventh Grade Reading Book … Sample Books/7th Reading Practice...Literary Text/Drama: Full length passage #2 ..... 54-58 Sample Reporting Category 2 Answer

118Seventh Grade Reading © 2016 GF Educators, Inc.

Exercise Reasoning4 The point of view from which the story is told gives the reader insight into –

A the man’s desire to acquire great wealth Wrong AnsWer: Third person POV allows us to see all the characters.

B the relationship between the man and his wife

Wrong AnsWer: Third person POV allows us to see all the characters.

C the man and the people he encounters right AnsWer: Because it is third person POV, we get a look at all the characters’ insights.

D the man’s feelings about the experience Wrong AnsWer: Third person POV allows us to see all the characters.

5 Read theses sentences from the story.

The servant answered, "I will be satisfied with the three admonitions." (paragraph 3)

But he, remembering the three admonitions of his master, answered, "No, my friends, I will keep on this road." (paragraph 6)

Based on these sentences, the reader can infer that the man –

A has not learned anything from the abbot Wrong AnsWer: The man more than learned from the abbot because he took what he learned to heart and made the harder choice.

B is willing to learn even after he left the abbey

right AnsWer: Going through the three admonitions as he travelled home shows that he learned after he left the abbey.

C should make an effort to be a better servant Wrong AnsWer: He was a prince -- the son of a raja. He is not a servant.

D is a disappointment to his wife for not taking the gold

Wrong AnsWer: His wife was not disappointed.

6 Which sentences best shows that the man approaches the admonitions with thoughtfulness?

A He said to the abbot, "Sir, I have served you thus long, but now I wish to return to my country."

Wrong AnsWer: This only lets the reader know the man is ready to go home.

B But he, remembering the three admonitions of his master, answered, "No, my friends, I will keep on this road."

right AnsWer: The man got to a place in the road, thought about what the abbot told him, and kept on the way he knew he was meant to go.

C "I have gained the second hundred ounces," said the good man, who did not think his skin was safe until then.

Wrong AnsWer: This shows that he was earning gold, not thinking about the admonitions.

D "Ah, is that he?" and he was on the point of coming forth and giving him a sound beating;

Wrong AnsWer: This does not show that he thought about the admonitions.

Sam

ple

Do Not

Dup

licat

e

Page 123: by GF Educators, Inc. Seventh Grade Reading Book … Sample Books/7th Reading Practice...Literary Text/Drama: Full length passage #2 ..... 54-58 Sample Reporting Category 2 Answer

119 © 2016 GF Educators, Inc.Seventh Grade Reading

The Prince Who Acquired WisdomA Folktale from India

1 There once was a raja who had an only son. The raja was always urging his son to learn to read and write, so when he came to his kingdom, he might manage well and be able to decide disputes that were brought to him for judgment. But the boy paid no heed to his father’s advice and continued to neglect his lessons. At last, when he was grown up, the prince saw that his father was right, and he resolved to go away to foreign countries to acquire wisdom. So he set off without telling anyone but his wife, and he took with him a purse of money and three pieces of gold. After traveling for a long time, he saw a man plowing in a field, and he asked him if there were any wise men living in that neighborhood. "What do you want with wise men?" asked the plowman.

2 The prince said that he was traveling to get wisdom. The plowman said that he would give him instruction if he were paid. Then the prince promised to give him one gold piece for each piece of wisdom.

3 The plowman agreed and said. "Listen attentively! My first maxim is this. You are the son of a raja. Whenever you go to visit a friend or one of your subjects and they offer you a bedstead, or stool, or mat to sit on, do not sit down at once but move the stool or mat a little to one side. This is one maxim. Give me my gold coin." So the prince paid him.

4 Then the plowman said, "The second maxim is this. You are the son of a raja. Whenever you go to bathe, do not bathe at the common bathing place, but at a place by yourself. Give me my coin," and the prince did so.

5 Then he continued, "My third maxim is this. You are the son of a raja. When men come to you for advice or to have a dispute decided, listen to what the majority of those present say and do not follow your fancy. Now pay me." The prince gave him his

Question 8Which of these best describes the raja’s son’s internal conflict in the story?

C He is unhappy that he did not listen to his father about his lessons.

The son realizes that he did not listen and had to resolve this problem, so he went out to become wise.

Question 7What phrase from paragraph 2 helps the reader understand the meaning of the word maxim in paragraph 3?

D each piece of wisdom

Paragraph 2 says he will pay for each piece of wisdom, and then the plowman proceeds to give the maxims – pieces of wisdom.

Question 9The raja’s son’s interaction with the plowman contributes to the plot by –

A causing the son to pay someone to help him learn

The son wants to learn, so he pays someone to learn. By doing this, he is trying to resolve his problem.Sa

mpl

e

Do Not

Dup

licat

e

Page 124: by GF Educators, Inc. Seventh Grade Reading Book … Sample Books/7th Reading Practice...Literary Text/Drama: Full length passage #2 ..... 54-58 Sample Reporting Category 2 Answer

120Seventh Grade Reading © 2016 GF Educators, Inc.

last gold coin and said that he had no more.

6 "Well," said the plowman, "Your lesson is finished, but still I will give you one more piece of advice free, and it is this. You are the son of a raja restrain your anger; if anything you see or hear makes you angry, still, do not at once take action. Hear the explanation and weigh it well, then if you find the cause, you can give rein to your anger and if not, let the offender off!"

7 After this, the prince set his face homewards since he had spent all his money, and he began to repent of having spent his gold pieces on advice that seemed worthless. However on his way, he turned into a bazaar to buy some food, and the shopkeepers on all sides called out, "Buy, buy." He went to a shop, and the shopkeeper invited him to sit on a rug. He was just about to do so when he remembered the maxim of his instructor and pulled the rug to one side. When he did so he saw that it had been spread over the mouth of a well and that if he had sat on it, he would have been killed, so he began to believe in the wisdom of his teacher.

8 Then he went on his way, and on the road, he turned aside to a tank to bathe. Remembering the maxim of his teacher, he did not bathe at the common place but went to a place apart. Then having eaten his lunch, he continued his journey. He had not gone far when he found that he had left his purse behind, so he turned back and found it lying at the place where he had put down his things when he bathed. Thereupon he applauded the wisdom of his teacher, for if he had bathed at the common bathing place, someone would have seen the purse and have taken it away.

9 When evening came on he turned into a village and asked the headman to let him sleep in his verandah. There was already one other traveler sleeping there, and in the morning it was found that the traveler had died in his sleep. Then the headman consulted the villagers, and they decided that there was nothing to be done but to throw

Question 11The raja’s son’s thoughts in this excerpt reveal that he -

C believes he spent the gold foolishly

Saying that he received advice that seemed worthless reveals he thinks it was a foolish way to spend the money.

Question 10How does the setting contribute to the plot of the story?

B The experiences on the way home contribute to how the son solves his conflict.

Leaving home to find out how to become wise, and then applying those things as he started back home, show how these travels helped the prince solve his problem.

Choices A and C do not refer to the setting, and the event in Choice D never happened.Sa

mpl

e

Do Not

Dup

licat

e

Page 125: by GF Educators, Inc. Seventh Grade Reading Book … Sample Books/7th Reading Practice...Literary Text/Drama: Full length passage #2 ..... 54-58 Sample Reporting Category 2 Answer

121 © 2016 GF Educators, Inc.Seventh Grade Reading

away the body, and since the prince was also a traveler he should do it.

10 At first, he refused to touch the corpse as he was the son of a raja, but the villagers insisted. Then he bethought himself of the maxim that he should not act contrary to the general opinion, so he yielded and dragged away the body, and threw it into a ravine. Before leaving it he remembered that it was proper to remove the clothes, and when he began to do so, he found a roll of coins around the waist of the body. So he took this and was glad that he had followed the advice of his teacher.

11 That evening he reached the boundary of his territory and decided to press on home although it was dark. At midnight he reached the palace and without arousing anyone went to the door of his wife's room. Outside the door, he saw a pair of shoes and a sword. At the sight he became wild with rage and drawing the sword, he called out, "Who is in my room?"

12 As a matter of fact, the prince's wife had asked the prince's little sister to sleep with her, and when the girl heard the prince's voice she got up to leave. But when she opened the door and saw the prince standing with the drawn sword she drew back in fear. She told him who she was and explained that they had put the shoes and sword at the door to prevent anyone else from entering, but in his wrath, the prince would not listen and called to her to come out and be killed. Then she took off her cloth and showed it to him through the crack of the door and at the sight of this he was convinced. Then he reflected on the advice of his teacher and repented because he had nearly killed his sister through not restraining his wrath.

Adapted from: Folklore of the Santal Parganas by Cecil H. Bompas (London: David Nutt Co., 1909)Sa

mpl

e

Do Not

Dup

licat

e

Page 126: by GF Educators, Inc. Seventh Grade Reading Book … Sample Books/7th Reading Practice...Literary Text/Drama: Full length passage #2 ..... 54-58 Sample Reporting Category 2 Answer

122Seventh Grade Reading © 2016 GF Educators, Inc.

Exercise Reasoning12 What theme is found in both selections?

A Don’t take advice from strangers. Wrong AnsWer: The raja's son did take advice from a stranger.

B Gold will make you happy. Wrong AnsWer: This cannot be the theme since the man took the admonitions, and the raja's son gave away his money.

C The third try is always successful. Wrong AnsWer: They were both given three pieces of advice, not three tries.

D One can profit from good advice. right AnsWer: Both men were given good advice, and they did profit from it.

13 Read these lines from “The Three Admonitions.”

"Ah, is that he?" and he was on the point of coming forth and giving him a sound beating; but there came to his mind the final admonition of the abbot: "Think over a thing before you do it, for a thing deliberated is very fine;" and he refrained.

Which sentence from “The Prince Who Acquired Wisdom” best matches the feeling expressed by the author of “The Three Admonitions” in the lines provided?

A Then he continued, "My third maxim is this. You are the son of a raja. When men come to you for advice or to have a dispute decided, listen to what the majority of those present say and do not follow your fancy. Now pay me."

Wrong AnsWer: This is advice being given, not being applied.

B Thereupon he applauded the wisdom of his teacher, for if he had bathed at the common bathing place, someone would have seen the purse and have taken it away.

Wrong AnsWer: The sentence from "The Three Admonitons has no reference to thinking about something before acting.

C That evening he reached the boundary of his territory and decided to press on home although it was dark.

Wrong AnsWer: The sentence from "The Three Admonitons" has no reference to learning or applying what was learned.

D Then he reflected on the advice of his teacher and repented because he had nearly killed his sister through not restraining his wrath .

right AnsWer: The raja's son thought about what he learned and decided against doing something rash, just as the man deliberated before doing something he would regret.

14 How does the man in “The Three Admonitions” differ from the raja’s son in “The Prince Who Acquired Wisdom”?

A The man went out searching for gold. Wrong AnsWer: Neither went out in search of gold.

B The man did not think about harming the person in his home.

Wrong AnsWer: They both thought about hurting a person in their homes.

C The man did not go out searching for wisdom.

right AnsWer: The man was at an abbey serving an abbot; he did not go out to search for wisdom as the raja's son did.

D The man did not get the gold at the end. Wrong AnsWer: The man did get the gold at the end.

Sam

ple

Do Not

Dup

licat

e

Page 127: by GF Educators, Inc. Seventh Grade Reading Book … Sample Books/7th Reading Practice...Literary Text/Drama: Full length passage #2 ..... 54-58 Sample Reporting Category 2 Answer

123 © 2016 GF Educators, Inc.Seventh Grade Reading

A Naughty Little Cometby Ella Wheeler Wilcox

There was a little comet who lived near the Milky Way! She loved to wander out at night and jump about and play.

The mother of the comet was a very good old star; She used to scold her reckless child for venturing out too far.

She told her of the ogre, Sun, who loved on stars to sup,And who asked no better pastime than in gobbling comets up.

But instead of growing cautious and of showing proper fear,The foolish little comet edged up nearer, and more near.

She switched her saucy tail along right where the Sun could see,And flirted with old Mars, and was as bold as bold could be.

She laughed to scorn the quiet stars who never frisked about; She said there was no fun in life unless you ventured out.

She liked to make the planets stare, and wished no better mirthThan just to see the telescopes aimed at her from the Earth.

She wondered how so many stars could mope through nights and days,And let the sickly faced old Moon get all the love and praise. And as she talked and tossed her head and switched her shining trailThe staid old mother star grew sad, her cheek grew wan and pale.

Literary Text/Paired Passages Set #2: Poem & Literary Nonfiction Read the selections and choose the best answer to each question.

Question 1Which line from the poem best supports that the star did not listen to his mother?

A She used to scold her reckless child for venturing out too far

The mother scolded the star for not listening to her and venturing out too far.

Question 3The poet describes the sun in this way to make it seem –

B human

Talking about supper and gobbling up comets makes the sun seems human.

Question 6The imagery in stanzas 5 through 9 helps the reader understand –

C the young star is clearly aware of what it is doing

The star switched its tail at the Sun knowing it was unadvised by its mother. She had warned about the Sun and what it would do.

10

5

15 Sam

ple

Do Not

Dup

licat

e

Page 128: by GF Educators, Inc. Seventh Grade Reading Book … Sample Books/7th Reading Practice...Literary Text/Drama: Full length passage #2 ..... 54-58 Sample Reporting Category 2 Answer

124Seventh Grade Reading © 2016 GF Educators, Inc.

For she had lived there in the skies a million years or more,And she had heard gay comets talk in just this way before.

And by and by there came an end to this gay comet's fun.She went a tiny bit too far -- and vanished in the Sun!

No more she swings her shining trail before the whole world's sight,But quiet stars she laughed to scorn are twinkling every night.

Source: The Beautiful Land of Nod by Ella Wheeler Wilcox (Chicago: W.B. Conkey Co, 1892)

5 This poem is mostly about a young comet who –

A lives in the Milky Way and learns to fly Wrong AnsWer: It is not about the star learning to fly, and the star lived near the Milky Way.

B looks for a new way to travel around the Sun Wrong AnsWer: The star is not looking to find a new way to travel.

C goes to the Moon to make him smile Wrong AnsWer: The Moon is insulted in the poem.

D does not follow her mother’s advice about the Sun

right AnsWer: The star did not follow its mother's advice, and ultimately vanished because of it.

Question 2The reader can conclude that the naughty little comet –

A was destroyed by the heat of the sun

She went a tiny bit too far -- and vanished in the Sun! The Sun destroyed the star.

Question 4What lesson did the young comet learn in the poem?

C It is better to be safe than sorry.

The star's mother warned her to careful, but the star did not listen.

Exercise Reasoning

20

Sam

ple

Do Not

Dup

licat

e

Page 129: by GF Educators, Inc. Seventh Grade Reading Book … Sample Books/7th Reading Practice...Literary Text/Drama: Full length passage #2 ..... 54-58 Sample Reporting Category 2 Answer

125 © 2016 GF Educators, Inc.Seventh Grade Reading

Science in the Middle AgesExcerpted from When Knights were Bold by Eva

March Tappan, 1911

1 The good folk of the Middle Ages were as much interested in the world of nature around them as are the people of today. They wondered what made lightning and thunder, why men died in the water and fish in the air, what would cure their various illnesses, why the moon rises, where the sun goes when it sets, and hundreds of other questions. Most of the studying of the day was carried on in monasteries, as has been said before, and the Venerable Bede and others wrote long treatises on nature, together with some remarkable explanations of its mysteries. In the twelfth century numerous universities were founded; and by the time that they were well established and had become strong and powerful, a fresh supply of knowledge came to them through the Saracens. Long before this, the Saracens had translated into their language, the Arabic, the works of the learned Greeks of centuries earlier, including especially what they knew of stars and planets and comets and eclipses. Many Saracens were now living around the Mediterranean Sea, and through them, manuscripts were brought into Europe and translated from the Arabic into the European tongues.

2 Astronomy was looked upon as an exceedingly practical study because it was by this science that the festivals of the Church were dated. The astronomers of the time knew something of eclipses, and they had tables of stars and planets. They studied the observations made by the wise men of the East for many centuries and learned a great deal. Unfortunately, they made one great mistake. For four hundred years it has been known that the Earth and the other planets revolve around the sun. In the Middle Ages, however, people believed that the sun revolved around the earth. The sky, they thought, was a vast hollow sphere which revolved once a day. It was because of this mistake that when they

Question 7Which definition best fits the meaning of hollow as it is used in paragraph 2?

A Definition 1 (not solid)

By substitution, not solid fits as the meaning of hollow in paragraph 2.Sa

mpl

e

Do Not

Dup

licat

e

Page 130: by GF Educators, Inc. Seventh Grade Reading Book … Sample Books/7th Reading Practice...Literary Text/Drama: Full length passage #2 ..... 54-58 Sample Reporting Category 2 Answer

126Seventh Grade Reading © 2016 GF Educators, Inc.

tried to reason about what they had seen, their conclusions were all wrong, no matter how correct their observations might have been.

3 Now when people believed that the whole starry universe was made for their especial benefit, it was not very unreasonable to take it for granted that the stars, their arrangement in the sky, and their movements had something to do with human affairs. Anything unusual was always alarming. Comets were a source of terror. No one knew whence they came or whither they were going. They were uncanny, and even the educated feared some awful disaster when one of these fiery wanderers appeared in the sky. In the middle of the fifteenth century, a large comet was seen which terrified all Europe. Even before its appearance people were in dread, for the Turks had crossed the Hellespont, and there was reason to believe that they would overrun the Continent. Then came the added horror of the comet, and no one could tell what awful calamity this might portend. It is no wonder that the Pope ordered the church bells to be rung at noon, and prayers to be said three times a day instead of twice. To this prayer was added the petition, "Lord, save us from the Devil, the Turk, and the Comet."

Source: When Knights were Bold by Eva March Tappan (Boston: Houghton Mifflin Co., 1911)

Question 8The author uses this sentence to show how people –

D were afraid of things they didn’t know about

If something is unusual, it can be uncommon and alarming. Therefore the people were afraid because they didn’t know about it.

Question 9In this sentence the author –

C shares the people’s anxiety over comets

There was “horror” over the comets, as well as wondering about the “awful calamity” they might foreshadow.

Sam

ple

Do Not

Dup

licat

e

Page 131: by GF Educators, Inc. Seventh Grade Reading Book … Sample Books/7th Reading Practice...Literary Text/Drama: Full length passage #2 ..... 54-58 Sample Reporting Category 2 Answer

127 © 2016 GF Educators, Inc.Seventh Grade Reading

Exercise Reasoning10 How does the author reveal the people’s fear of the comets?

A By explaining the people’s theory about the sun’s rotation

Wrong AnsWer: The theory of the sun's rotation has nothing to do with their fear.

B By including the prayer they said three times a day

right AnsWer: The Pope ordered an extra time of prayer because the people were afraid of the comet.

C By comparing the university’s ideas to the Church’s ideas

Wrong AnsWer: This did not reveal the people's fear. It showed the two ways that the stars and planets were studied.

D By describing what people of the Middle Ages wondered about

Wrong AnsWer: What they wondered about did not reveal the people's fear.

11 Which sentence shows that the people in the Middle Ages were eager to learn?

A Astronomy was looked upon as an exceedingly practical study because it was by this science that the festivals of the Church were dated.

Wrong AnsWer: This sentence has to do with the church, not people eager to learn.

B They studied the observations made by the wise men of the East for many centuries and learned a great deal.

right AnsWer: By studying observations of the wise men, they learned. This shows they were eager to learn.

C In the Middle Ages, however, people believed that the sun revolved around the earth.

Wrong AnsWer: This sentence shows the false thinking the people had.

D No one knew whence they came or whither they were going.

Wrong AnsWer: This sentence has nothing to do with learning. Learning is never mentioned.

12 The poet and the author would most likely agree about the unknowns of –

A the comets and the stars right AnsWer: Both selections show that the unknowns of the stars and comets are out there.

B church prayer requests Wrong AnsWer: The poem has no mention of the church and prayers in it.

C how the sun orbits the Earth Wrong AnsWer: The Sun orbiting the Earth is not a part of the poem.

D the power of the Sun Wrong AnsWer: The Sun is not specifically mentioned in the selection.

Sam

ple

Do Not

Dup

licat

e

Page 132: by GF Educators, Inc. Seventh Grade Reading Book … Sample Books/7th Reading Practice...Literary Text/Drama: Full length passage #2 ..... 54-58 Sample Reporting Category 2 Answer

128Seventh Grade Reading © 2016 GF Educators, Inc.

Exercise Reasoning13 How are the poem and the selection different?

A The poem focuses on the sun, while the selection focuses on the church.

Wrong AnsWer: The selection does not focus only on the church.

B The poem relates facts, while the selection relates a personal experience.

Wrong AnsWer: The poem is not about facts, and the selection is not a personal account.

C The poem focuses on having fun, while the selection focuses on learning.

right AnsWer: The poem is about a certain star having fun, and the selection is about learning about stars and comets.

D The poem discourages being obedient, while the selection encourages fear of the unknown.

Wrong AnsWer: The poem does discourage disobedience, but the selection does not encourage people to fear the unknown.

14 Read these lines from the poem.

She liked to make the planets stare, and wished no better mirth

Than just to see the telescopes aimed at her from the Earth.

Which sentence from the selection best matches an idea present in these lines?

A They wondered what made lightning and thunder, why men died in the water and fish in the air, what would cure their various illnesses, why the moon rises, where the sun goes when it sets, and hundreds of other questions.

Wrong AnsWer: This is what the people were wondering and wanting to learn about the sky.

B In the Middle Ages, however, people believed that the sun revolved around the earth.

Wrong AnsWer: This does not connect to the star enjoying being watched.

C Now when people believed that the whole starry universe was made for their especial benefit, it was not very unreasonable to take it for granted that the stars, their arrangement in the sky, and their movements had something to do with human affairs

right AnsWer: The line from the poem shows that the star knew the people on Earth were looking up at her. She was there for the humans. The same with the quote from the selection. The people believed that the stars were there for them to look at.

D They were uncanny, and even the educated feared some awful disaster when one of these fiery wanderers appeared in the sky

Wrong AnsWer: This sentence is about people's fear of comets.Sam

ple

Do Not

Dup

licat

e

Page 133: by GF Educators, Inc. Seventh Grade Reading Book … Sample Books/7th Reading Practice...Literary Text/Drama: Full length passage #2 ..... 54-58 Sample Reporting Category 2 Answer

129 © 2016 GF Educators, Inc.Seventh Grade Reading

Benjamin Franklin: My Story

Printer

1 As a child, I loved to read,

write, and collect books; so

when it came time to choose

a trade, my father decided I

would become a printer. At

the age of twelve, I started as

an apprentice with my older

brother, James. At the age of

twenty-two, I opened my own

printing shop. My newspaper,

The Pennsylvania Gazette, became very popular

and profitable. A few years later, Poor Richard’s

Almanack was released and soon became the best-

selling book in the colonies, selling over 10,000

copies a year. Soon, I became the most active

printer in the colonies and was appointed the official

printer of Pennsylvania. My duties included printing

money, laws, and documents for the colony. I then

became the public printer for Delaware, New Jersey,

and Maryland. I also helped establish newspapers in

New York, Connecticut, and two islands in the West

Indies.

Librarian

2 Books were very expensive during my time, and

not everyone was able to afford them. Some fellow

printers and I, known as the Leather Apron Club

because most of us wore leather aprons, discussed

ways we could help the community. Through my

suggestion, we started a lending library that was

Literary Text/Paired Passages Set #3: Autobiography & Fictional Adaptation Read the selections and choose the best answer to each question.

Question 1Which definition represents the meaning of establish as it is used in paragraph 1?

A Definition 1 (to bring into being)

Franklin created and brought about the newspapers.

Sam

ple

Do Not

Dup

licat

e

Page 134: by GF Educators, Inc. Seventh Grade Reading Book … Sample Books/7th Reading Practice...Literary Text/Drama: Full length passage #2 ..... 54-58 Sample Reporting Category 2 Answer

130Seventh Grade Reading © 2016 GF Educators, Inc.

open to everyone. We would pool our money and

buy books that people could borrow. In 1731, the

first lending library in America opened. Soon, other

towns began to imitate that first library until reading

became fashionable, even among the less educated.

Inventor

3 At the age of forty-two, I retired from printing

to explore my other interests. I devoted this time to

inventing. During my life, I invented many things,

including swim fins, bifocals, a glass harmonica,

watertight bulkheads for ships, the lightning rod, an

odometer, and the wood stove called the Franklin

stove.

4 In addition to inventing, I enjoyed experimenting.

One thing that always fascinated me was electricity.

One stormy night, my son William and I were

experimenting with lightning. You see, during my

time, people thought that lightning was a mysterious

force that was a punishment from God; however,

I wanted to prove that lightning is a form of

electricity.

5 I attached a pointed metal piece on the top of

my kite and a metal key to the base of the string.

Lightning struck the kite and traveled down the

kite string to the key. When I touched the key with

my knuckle, there was a spark! It’s a good thing I

didn’t electrocute myself. I had been knocked

unconscious twice before when experimenting

with electricity, but diligence is the mother of

good luck. I did prove that lightning is electricity!

Afterward, I invented the lightning rod to protect

buildings and ships from lightning damage. Although

I received much recognition from my inventions, I

did not profit from them. Instead, I chose to give my

inventions to the world freely.

Question 2Franklin uses this sentence to explain –

C how his persistence paid off

By being diligent, Franklin created the lightning rod. It was his "good luck" which came from being persistent even after he had been knocked out by lightning.Sa

mpl

e

Do Not

Dup

licat

e

Page 135: by GF Educators, Inc. Seventh Grade Reading Book … Sample Books/7th Reading Practice...Literary Text/Drama: Full length passage #2 ..... 54-58 Sample Reporting Category 2 Answer

131 © 2016 GF Educators, Inc.Seventh Grade Reading

Statesman

6 When I was born, America was made up of

thirteen colonies that were ruled by England.

Trouble between England and the thirteen colonies

started to unfold following the French and Indian

War. The imposition of the Stamp, Townshend,

and Intolerable Acts angered the colonists to rebel

against Mother England. On April 19, 1775, the

colonists went to war for their freedom.

7 During the fight for independence, I was sent

to Europe to represent the colonies. In 1776,

I signed the Declaration of Independence

and, in 1778, the Treaty of Alliance with France.

In addition, I negotiated with the French to

help the colonists and became the first United

States minister to France. I helped secure

guns, ammunition, and other provisions for

the army as well as volunteer troops. When the

colonists won their independence in 1781, I helped

negotiate peace with England and signed what

ultimately became known as Treaty of Peace with

Great Britain in 1782.

Timeline

8 Now that the colonists were free and

independent, it became necessary to decide

what type of government would be established.

I was not in my prime, and my health was

starting to fail; at eighty-one, I was the oldest

delegate at the Constitutional Convention. Upon

signing the Constitution on September 17, 1787, I

became the only Founding Father to have signed

all five documents that established American

independence: the Declaration of Independence,

the Treaty of Amity and Commerce with France,

the Treaty of Alliance with France, the Treaty of

Question 5Paragraph 7 is important to the selection because it shows –

B how Franklin contributed to making America independent

The information in the paragraph shows the things Franklin did to help prepare the US for the war for independence and how he negotiated the peace treaty.

Sam

ple

Do Not

Dup

licat

e

Page 136: by GF Educators, Inc. Seventh Grade Reading Book … Sample Books/7th Reading Practice...Literary Text/Drama: Full length passage #2 ..... 54-58 Sample Reporting Category 2 Answer

132Seventh Grade Reading © 2016 GF Educators, Inc.

Peace with Great Britain, and the Constitution of the

United States of America.

Source: Superintendent of Documents, U.S. Government Printing Office. http://bensguide.gpo.gov/

Exercise Reasoning3 What can the reader infer about Franklin?

A He did not want to be a printer when he was twelve.

Wrong AnsWer: There is nothing in the passage to let the reader infer this. In fact, he started several newspapers because of this.

B He thought that he should do things to contribute to humankind.

right AnsWer: This is inferred by the fact that he freely gave his inventions to the world.

C He was determined to be the only one to sign all five documents.

Wrong AnsWer: Franklin never set out to sign all five documents.

D He came to realize that inventing was his life’s calling

Wrong AnsWer: He enjoyed inventing and experimenting, but his life's calling was to be a statesman.

Question 4The timeline was included in the selection most likely to –

D highlight the milestones that occurred in Franklin’s life

The items listed on the timeline are the major events in Franklin's life.

Sam

ple

Do Not

Dup

licat

e

Page 137: by GF Educators, Inc. Seventh Grade Reading Book … Sample Books/7th Reading Practice...Literary Text/Drama: Full length passage #2 ..... 54-58 Sample Reporting Category 2 Answer

133 © 2016 GF Educators, Inc.Seventh Grade Reading

Books of His BoyhoodAn Excerpt from Chapter 13 of

From Boyhood to Manhood: Life of Benjamin Franklin, 1889

1 Not long after Benjamin became a printer,

a prominent citizen of Boston, Matthew Adams,

who had heard of his talents and love of reading,

met him in the printing office and entered into

conversation with him.

2 “You are a great reader, I learn,” he said.

3 “Yes, sir, I read considerable every day.”

4 “Do you find all of the books you want to read?”

5 “Not all. I should like to read some books I can’t

get.”

6 “Perhaps you can find them in my library; you

can come take out of it any book you would like.”

7 “Thank you, very much,” answered Benjamin,

exceedingly gratified by this unexpected offer. “I

shall take the first opportunity to call.”

8 “Boys who like to read as well as you do, ought

to have books enough,” continued Mr. Adams. “I

think you will find quite a number of entertaining

and useful ones. You will know when you examine

for yourself.”

9 “That I shall do very soon, and be very grateful

for the privilege,” answered Benjamin.

10 Within a few days, the printer-boy paid Mr.

Adams a visit. The latter gave him a cordial

welcome, causing him to feel at ease and enjoy

his call. He examined the library to his heart’s

content and found many books therein he

desired to read.

11 “Come out anytime, and take out any and all of

the books you please, and keep them until you are

done with them,” was Mr. Adams’ generous offer.

Question 6Which synonym would best replace the word prominent as it is used in paragraph 1?

B Number 2 (important)

By substituting, the best word to fit in the sentence is important.

Question 7The Latin gratus, meaning “pleasing, thankful,” helps the reader understand that gratified in paragraph 7 means –

A pleased

Because he was able to choose any book he wanted, Franklin was pleased with Adams' allowing him to borrow books.

Question 8The author uses this sentence to show that Benjamin-

D was excited about all the books available to him

By looking over the books to his “heart’s content,” and finding as many books as he “desired” to read, the reader can tell he was excited about the books he could read.

Sam

ple

Do Not

Dup

licat

e

Page 138: by GF Educators, Inc. Seventh Grade Reading Book … Sample Books/7th Reading Practice...Literary Text/Drama: Full length passage #2 ..... 54-58 Sample Reporting Category 2 Answer

134Seventh Grade Reading © 2016 GF Educators, Inc.

He had a great interest in the boy, and wanted to assist him, and Benjamin fully

appreciated his interest and kindness and paid the library many visits. As long as

he lived, he never forgot the generous aid of this man.

12 The printing office was frequented by booksellers’ apprentices, whose

employers wanted jobs of printing done. Benjamin made their acquaintance, and

they invited him to call at their stores to examine the books. There were several

bookstores in Boston at that time, although the number of books was very limited

as compared to with the present time.

13 “I will lend you that book tonight,” said one of the apprentices to Benjamin,

who was manifesting a deep interest in a certain volume. “You can return it in the

morning before the customers come in.”

14 “Very much obliged. I shall be glad to read it. I think I can read it through

before I go to bed, and I can leave it when I go to the office in the morning.”

15 “You won’t have time for much sleep if you read that book before going to bed.

But you are used to short naps, I expect.”

16 This offer of the apprentice was very generous, and Benjamin suitably

expressed his appreciation of it. “Your favor is so great. If I were in a bookstore as

you are, I fear that my love of reading would overcome my love of work. It would

just suit me to be in the company of books all the time.”

17 Benjamin made the most of this new opportunity. Borrowing the first book was

followed by borrowing from many of the apprentices at the bookstores. All of the

stores were patronized by him, and many a night was shortened at both ends that

he might devour a book. He fairly gorged himself with book knowledge.

18 The reader must not forget that books were very few in number at that time,

and it was long before a public library was known in the land. In Boston, there

were many literary people who had come hither from England, and they had a

limited supply of books. So that Boston was then better supplied with books than

any other part of the country, though its supply was as nothing compared with the

supply now. Bookstores, instead of being supplied with thousands of volumes to

suit every taste in the reading world, offered only a meager collection of volumes,

such as would be barely noticed now.

Source: From Boyhood to Manhood: Life of Benjamin Franklin, by William M. Thayer (New York: Hurst Publishing Co., 1889)Sam

ple

Do Not

Dup

licat

e

Page 139: by GF Educators, Inc. Seventh Grade Reading Book … Sample Books/7th Reading Practice...Literary Text/Drama: Full length passage #2 ..... 54-58 Sample Reporting Category 2 Answer

135 © 2016 GF Educators, Inc.Seventh Grade Reading

Exercise Reasoning9 Which sentence best summarizes the excerpt?

A Benjamin never had enough books to read, so he went around the town begging people to lend him books.

Wrong AnsWer: Franklin did not beg for books.

B Benjamin loved reading, and people decided to let him borrow books to satisfy his desire to read.

right AnsWer: Franklin was able to get books from a variety of people because they knew he loved to read.

C When Benjamin went to Mr. Adams’ house, he borrowed books and was inspired with the idea of opening a library.

Wrong AnsWer: This statement does not share important details from the entire passage.

D The apprentices let Benjamin borrow books, but he never returned them to the apprentices

Wrong AnsWer: Franklin did return the books.

10 Which sentence shows that Benjamin appreciated what the apprentices did for him?

A “I think I can read it through before I go to bed, and I can leave it when I got to the office in the morning."

Wrong AnsWer: This sentence has no words of appreciation.

B “Your favor is so great." right AnsWer: Saying that the favor is great is showing appreciation.

C Benjamin made the most of this new opportunity.

Wrong AnsWer: There is no mention of appreciation in this sentence.

D As long as he lived, he never forgot the generous aid of this man.

Wrong AnsWer: This sentence is only about one man - Adams - not the apprentices.

11 What is the significance of Benjamin going to all the apprentices to borrow books?

A It emphasizes that he wanted to learn and loved to read.

right AnsWer: Going to all the apprentices, who had different books they were responsible for, meant Franklin had access to more books to read and more from which he could learn.

B It suggests that he wanted to learn their trade.

Wrong AnsWer: He did not want to learn their trade. In fact, he said his love of reading would overcome his love of work.

C It suggests that they wanted him to start a library.

Wrong AnsWer: Although Franklin did eventually help establish a public library, this idea was not put in his mind by the apprentices.

D It emphasizes the apprentices’ large collections of books.

Wrong AnsWer: Books were few in numbers at the time, so they did not have large collections.

Sam

ple

Do Not

Dup

licat

e

Page 140: by GF Educators, Inc. Seventh Grade Reading Book … Sample Books/7th Reading Practice...Literary Text/Drama: Full length passage #2 ..... 54-58 Sample Reporting Category 2 Answer

136Seventh Grade Reading © 2016 GF Educators, Inc.

Exercise Reasoning12 How does “Benjamin Franklin: My Story” differ from “Books from His Boyhood?”

A “Benjamin Franklin: My Story” describes events in the order that they happened.

Wrong AnsWer: Both selections tell about events in order.

B “Benjamin Franklin: My Story” explains that Franklin started the first lending library.

Wrong AnsWer: It is not only about Franklin starting the first lending library.

C “Benjamin Franklin: My Story” focuses on only one event in Franklin’s life.

Wrong AnsWer: "Books From His Boyhood" focuses on one event in Franklin's life.

D “Benjamin Franklin: My Story” describes events occurring over a longer period.

right AnsWer: "Benjamin Franklin: My Story" does take place over a longer period of time.

13 How is Benjamin the same in “Benjamin Franklin: My Story” and “Books of His Boyhood”?

A Both selections describe him as a statesman.

Wrong AnsWer: Only "Benjamin Franklin, My Story" describes him as a statesmen.

B Both selections relate his desire to read and learn.

right AnsWer: The first sentence in "Benjamin Franklin: My Story" says he loved to read; the whole excerpt “Books of His Boyhood” is about his love of reading.

C Both selections recount his time as an apprentice.

Wrong AnsWer: He was an apprentice in “Benjamin Franklin: My Story.” Apprentice booksellers are in “Books of His Boyhood, but it does not mention that Franklin was an apprentice.”

D Both selections include his experience inventing things.

Wrong AnsWer: Only “Benjamin Franklin: My Story” refers to his experience with inventing.

14 Read this sentence from “Benjamin Franklin: My Story.”

As a child, I loved to read, write, and collect books;

Which sentence from “Books of His Boyhood” best matches the idea expressed by Franklin in the sentence from his autobiography?

A “Yes, sir, I read considerable every day.” Wrong AnsWer: This says he read, not that he loved to read.

B There were several bookstores in Boston at that time, although the number of books was very limited as compared to with the present time.

Wrong AnsWer: This tells about the limited number of books in Boston.

C “You won’t have time for much sleep if you read that book before going to bed.”

Wrong AnsWer: This states that it will take him a long time to read the book.

D “It would just suit me to be in the company of books all the time.”

right AnsWer: If he wants to be in the company of books all the time, it is because he loves them.

Sam

ple

Do Not

Dup

licat

e

Page 141: by GF Educators, Inc. Seventh Grade Reading Book … Sample Books/7th Reading Practice...Literary Text/Drama: Full length passage #2 ..... 54-58 Sample Reporting Category 2 Answer

137 © 2016 GF Educators, Inc.Seventh Grade Reading

The Mayo Family of Doctors

William Worrall Mayo, M.D., (1819-1911) center, with sons, Dr. Charlie (1865-1939), left, and Dr. Will (1861-1939). Both brothers often said, "Our father taught us," in explaining their commitment to excellence in patient care, research, and education. (Photo credit: MayoClinic.org)

1 In 1863, Dr. William Mayo settled in Rochester,

Minnesota and established a medical practice. His

two sons, William and Charles, joined the practice

in 1883 and 1888. Together they founded the

world famous Mayo Clinic. From the beginning,

they were doctors who brought the newest and best

practices in medicine to the United States from

all over the world. All three wanted to learn

more about medicine in order to save lives lost

from the spreading of infection. They decided

that one doctor would travel and learn new

discoveries and techniques while the other

two stayed to help the patients.

2 The Mayos studied Louis Pasteur’s findings on

sterilizing surgical instruments to kill germs that

caused infections. They set up a hospital that

was clean, and they protected the patients

from infection. They brought other new

discoveries, such as the microscope, to their

Informational Text/Paired Passages Set #4: Two Biographies Read the selections and choose the best answer to each question.

Question 5The image included with the article helps the reader understand –

A how doctors used to dress

The image allows the reader to see how doctors use to dress - suits and ties.

Question 1Paragraph 1 is mainly about the–

A early days of the Mayo Clinic

This is about the founding of the clinic and what the Mayos wanted to do at the beginning to make it excellent.

Question 3The Mayo doctors focused their attention on –

C healing their patients

With their careful treatment of infections and their willingness to travel to learn new discoveries, they were focused on healing their patients.Sa

mpl

e

Do Not

Dup

licat

e

Page 142: by GF Educators, Inc. Seventh Grade Reading Book … Sample Books/7th Reading Practice...Literary Text/Drama: Full length passage #2 ..... 54-58 Sample Reporting Category 2 Answer

138Seventh Grade Reading © 2016 GF Educators, Inc.

hospital and used them to save patients. The

Mayo Clinic became known for excellent

patient care. As time passed, more and more

people wanted the Mayos to be their doctors. To

meet this need, they asked other doctors and

science researchers to join them.

3 Soon doctors from all over the world were

coming to their hospital to learn more about

medicine. Their hospital was a place to heal the

sick and also a place for doctors to learn. During

this period, medicine advanced and began to save

more lives. The Mayo doctors were the doctors

that led the way in the team approach to medicine

where a team of specialists comes together to

serve patients. This technique is known as the

Mayo Clinic Model of Care. Today, the Mayo

Clinic carries on the Mayo family philosophy

of “giving back.” It employs more than 3,800

physicians and scientists, as well as 50,900

allied health staff members. While the clinic’s

primary focus is on patient care, it spends

over $500 million a year on medical research.

Exercise Reasoning4 Which of these is the best summary of the article?

A When they first established the Mayo Clinic, William Mayo and his sons could never have known the far-reaching effects of their efforts. With the help of Louis Pasteur, the Mayos wiped out infection and improved the quality of hospital care.

Wrong AnsWer: Pasteur did not personally work with the mayos. They studied his findings.

B When the Mayos opened their clinic, people were dying because hospitals were not clean. The Mayos changed all of that, so many people came to their clinic wanting the best medical care they could find. The people found this great care at the Mayo Clinic.

right AnsWer: This summary is missing pertinent information.

C Two brothers, William and Charles Mayo, followed the family tradition and entered the medical field. They worked with their father and began to research medicine and infection. Now, their clinic is the best medical clinic worldwide.

Wrong AnsWer: This summary is missing pertinent information.

D The Mayo family doctors established a medical clinic. This clinic was known for using modern medical practices and having excellent patient care. Doctors from all over the world came there to learn. Today, the Mayo Clinic carries on the tradition of outstanding patient care and research.

Wrong AnsWer: tThis summary provides all of the pertinent information from the selection.

Question 6Which sentence from the selection best suggests that the Mayos thought healthcare is important?

B The Mayo Clinic became known for excellent patient care.

Excellent patient care shows that they think healthcare is important. Patients are what is important to healthcare.

Question 2In paragraph 3, the word specialists means –

B doctors

The specialists are serving the patients, which in this case makes them doctors.

Question 7From the information in the selection, the reader can conclude that –

D the Mayo family was generous and charitable.

The Mayos spend $500 million a year on research, and they hire more than 54,000 physicians, scientists, and allied health staff members.

Sam

ple

Do Not

Dup

licat

e

Page 143: by GF Educators, Inc. Seventh Grade Reading Book … Sample Books/7th Reading Practice...Literary Text/Drama: Full length passage #2 ..... 54-58 Sample Reporting Category 2 Answer

139 © 2016 GF Educators, Inc.Seventh Grade Reading

Alexander Graham Bell

1 Alexander Graham Bell was an inventor whose

greatest invention was the telephone. Bell studied

the telegraph and worked to improve it. In the late

1800s, the telegraph received or sent one message

at a time using a series of dot-and-dash codes. Bell

tried to design a telegraph that would be able to

send more than one message at a time. Later, he

envisioned the possibility of sending voice messages

over an electrical wire. Bell worked for years trying

to transfer voices using the same principle that

clerk use to transmit taps on a telegraph. Then, in

1876, at the age of 29, he invented the telephone

— a special device to transmit and receive sound.

In 1877, he formed the Bell Telephone Company.

Alexander Graham Bell stuck to a seemingly

impossible idea until he made it work.

2 Alexander Graham Bell’s name stands out as

an inventor, but that was not his main profession.

He was a teacher. His father, Melville Bell, created

Visible Speech, a method of teaching people who

couldn’t hear how to talk. Since they couldn’t hear,

they couldn’t imitate sounds. Bell’s father drew

pictures of the positions of the tongue when certain

sounds are made. While using these pictures,

sounds could be combined into words. Visible

Speech was a system to aid people in speaking

any language even if they had not heard it. Deaf

pupils were trained to move their tongues in certain

positions and follow the chart to produce sounds.

3 In Boston in 1872, Alexander Bell opened

his private school using Visible Speech to teach

articulation to deaf people. By 1877, he abandoned

Visible Speech and developed his methods of

Question 8In paragraph 1, the word

envisioned means –

B formed a mental picture

If he envisioned the possibility of sending voice messages, he was able to form a picture in his mind to see that.

Question 9Which synonym would best replace the word articulation as it is used in paragraph 3?

C Number 3 (communication)

The best synonym to fit into the sentence is communication. He wanted Visible Speech to teach deaf people to communicate.

Sam

ple

Do Not

Dup

licat

e

Page 144: by GF Educators, Inc. Seventh Grade Reading Book … Sample Books/7th Reading Practice...Literary Text/Drama: Full length passage #2 ..... 54-58 Sample Reporting Category 2 Answer

140Seventh Grade Reading © 2016 GF Educators, Inc.

teaching speech and lip reading to deaf children. He

believed these skills were essential for deaf people

to participate in society and to advance personally

and professionally.

Before anything else, preparation

is the key to success.

~Alexander Graham Bell

4 Alexander Bell worked on his inventions after

school in his basement. The fathers of some of

Bell’s deaf students helped finance his experiments

and became involved in the formation of Bell

Telephone Company. Bell used his fame and wealth

from the telephone to promote education for deaf

students. More than any other American, Bell was

the most important person in the movement to

teach deaf children speaking and lip-reading. For

the remainder of his life until his death in 1922,

Bell promoted deaf education through publications,

conferences, and meetings. Even though most

people remember Alexander Graham Bell as a great

inventor, he believed his most important work was

teaching the deaf to speak. He used his wealth from

inventing the telephone as a way to finance his

efforts.

Question 10The image beside paragraph 4 is included in the selection most likely to –

B share Bell’s advice about the importance of being prepared

The image of him is in conjunction with the saying. This is why the image is with the selection.

Sam

ple

Do Not

Dup

licat

e

Page 145: by GF Educators, Inc. Seventh Grade Reading Book … Sample Books/7th Reading Practice...Literary Text/Drama: Full length passage #2 ..... 54-58 Sample Reporting Category 2 Answer

141 © 2016 GF Educators, Inc.Seventh Grade Reading

Exercise Reasoning11 What is the most likely reason that the author included information about Visible Speech?

A To show that Alexander Graham Bell’s father was deaf

Wrong AnsWer: The selection does not say his father was deaf.

B To clarify why deaf people have difficulty speaking

Wrong AnsWer: Visible Speech shows that people need help with communicating. It does not clarify why they have difficulty.

C To explain how Alexander Graham Bell became involved in teaching the deaf

right AnsWer: Visible Speech is what encouraged Bell to work on a way to help deaf people communicate.

D To persuade deaf people to use Visible Speech to communicate with others

Wrong AnsWer: This selection is not trying to persuade deaf people to use Visible Speech.

12 With which of the following statements would Alexander Graham Bell most likely agree?

A The most certain way to succeed is always to try just one more time.

right AnsWer: Bell worked on many inventions to help people communicate. So he did try one more time.

B Life is a series of baby steps. Wrong AnsWer: This quote does not relate to the selection.

C I do what I love to do at the moment. Wrong AnsWer: Although he did do what he loved to do, Bell didn’t necessarily do it at the moment.

D I had parents who believed I could do anything.

Wrong AnsWer: The selection does not show that he needed his parents' approval.

13 Which of the following sentences best expresses the controlling idea of the passage?

A The telephone Alexander Graham Bell invented was based on the telegraph.

Wrong AnsWer: The selection is not mainly about the telephone.

B Alexander Graham Bell and his father, Melville Bell, lived in Boston and taught speech to hearing-impaired individuals.

Wrong AnsWer: The selection is only about teaching. He invented as well.

C Alexander Graham Bell was a teacher and an inventor who used his talent and wealth to promote deaf education.

right AnsWer: The article is about Bell teaching and using his talents to help deaf people.

D Alexander Graham Bell is most known for his invention of the telephone, but he also was a teacher.

Wrong AnsWer: The selection is not just about the telephone.Sa

mpl

e

Do Not

Dup

licat

e

Page 146: by GF Educators, Inc. Seventh Grade Reading Book … Sample Books/7th Reading Practice...Literary Text/Drama: Full length passage #2 ..... 54-58 Sample Reporting Category 2 Answer

142Seventh Grade Reading © 2016 GF Educators, Inc.

Exercise Reasoning14 The Mayos and Alexander Graham Bell would most likely agree about the importance of –

A finding discoveries in your country Wrong AnsWer: The Mayos went to other countries to find new discoveries.

B parents have a great influence in our lives right AnsWer: The Mayo's father was the reason they got into medicine; Bell's father invented Visible Speech which caused Bell to work on helping deaf people.

C doctors being the ones who help physically challenged

Wrong AnsWer: Bell was not a doctor, and he helped the physically challenged.

D allowing patients to make important decisions

Wrong AnsWer: Neither selection mentioned this.

15 What is one difference between the selections?

A “The Mayo Family of Doctors” discusses traveling to find new discoveries, while “Alexander Graham Bell” focuses on staying in the United States.

Wrong AnsWer: “Alexander Graham Bell” never mentioned staying in the US or traveling.

B “The Mayo Family of Doctors” names an expert to support the idea that infections are bad, while “Alexander Graham Bell” relies on the telephone company to encourage his Visible Speech.

Wrong AnsWer: The telephone company is not mentioned as encouraging people to use Visible Speech in “Alexander Graham Bell."

C “The Mayo Family of Doctors” focuses on going out to find new discoveries and techniques to help people, while “Alexander Graham Bell” used his own methods to help deaf people.

right AnsWer: The Mayos had one person travel to find new discoveries, and Bell worked with his methods for teaching speech and lip reading.

D “The Mayo Family of Doctors” was only about one person who helped people, while “Alexander Graham Bell” was about a group of people who helped others

Wrong AnsWer: The Mayos was about a group of people, and Bell was about one person.

16 The information presented in “The Mayo Family of Doctors” and “Alexander Graham Bell” support the idea that -

A innovations and breakthroughs stretch the limit of what is possible

right AnsWer: Both the Mayos and Bell did many different things to help people. They wanted more to help people.

B companies are more successful than individuals when it comes to helping people

Wrong AnsWer: This is not inferred in either passage.

C improvements in the medical field will help people heal faster

Wrong AnsWer: Deaf people would not be healed by what Bell did, so this would not be supported in both.

D success with new techniques in the medical field is best left to doctors

Wrong AnsWer: Bell was not a doctor, so this would not be supported in both.

Sam

ple

Do Not

Dup

licat

e

Page 147: by GF Educators, Inc. Seventh Grade Reading Book … Sample Books/7th Reading Practice...Literary Text/Drama: Full length passage #2 ..... 54-58 Sample Reporting Category 2 Answer

143 © 2016 GF Educators, Inc.Seventh Grade Reading

The Musgrave Pencil Factory

1 With the start of

World War I, goods

were no longer freely

exchanged between the

United States and other

countries. James R.

Musgrave had an idea.

He turned to American manufacturers to start a

company. It would become the Musgrave Pencil

Company. His company started in Tennessee in the

early 20th century.

2 “The Colonel” (the name Musgrave’s Northern

colleagues gave him) decided Tennessee red cedar

was the ideal material for his pencils. He chose

to use "recycled" cedar which he acquired by

exchanging cedar rail fences for a modern, and

sought-after equivalent wire fence. With a plan

already in mind, Musgrave purchased some land.

He hired a crew that traded or bartered with

the farmers: the new wire and pole fence for the

old cedar rails. The crew did the work necessary

to install the new fence. In some cases of

extremely large tracts, money was exchanged.

The cedar rails were already dry and weathered.

They were perfect to be cut into pencil slats at

the mill in Shelbyville. The milled slats went from

Shelbyville to German manufacturers, Faber and

others.

3 In 1919 the Pencil Makers Association was

Informational Text/Paired Passages Set #5: Expository & Procedural Read the selections and choose the best answer to each question.

Question 1Paragraph 2 is mainly about –

A how the cedar was obtained to make pencils

This paragraph tells how cedar rails were traded or bartered or paid for to make pencils.

Question 2Which synonym would best replace the word colleagues as it is used in paragraph 2?

D Number 4 (associates)

The nickname shows that these colleagues were his equal, associates.

Sam

ple

Do Not

Dup

licat

e

Page 148: by GF Educators, Inc. Seventh Grade Reading Book … Sample Books/7th Reading Practice...Literary Text/Drama: Full length passage #2 ..... 54-58 Sample Reporting Category 2 Answer

144Seventh Grade Reading © 2016 GF Educators, Inc.

organized. It not only represented, but it unified

the pencil industry. Raw material and technology

were exchanged within the United States' markets.

The Colonel brought a German machinist and his

expertise to his factory in Tennessee. As time went

on, the Tennessee red cedar source of logs and rail

fences was depleted. It was replaced with a wood of

similar characteristics from California -- California

Incense Cedar. This cedar was a fast growing,

plentiful, and renewable substitute.

4 By the time of the 1929 Great Depression,

the company was making its pencils. It is difficult

to conceive how hard times were during the

depression. Musgrave decided to help others by

nurturing the establishment of other local pencil

manufacturers. With the start of World War II,

women became a part of the workforce. The women

compensated for husbands, brothers, or fathers

who were overseas in

the war effort.

5 The Musgrave Pencil

Company has seen

the community’s and

the state's industrial

economy grow up

around it. In the 1950s Governor Buford Ellington

named Shelbyville "The Pencil City." In a large part

this was due to the leadership at the Musgrave

Pencil Company.

6 A true Tennessee product, the Musgrave Pencil is

a result of commitment at all levels from materials

to management. The legacy of the Colonel's

entrepreneurial skills and instinct contributes,

through Musgrave Pencil Company, to this states'

diverse and intriguing story. Today, the Musgrave

Pencil Company still stands as a testament to hard

Question 3In paragraph 3, the word depleted means –

D used up

If the source was depleted, and was then replaced, depleted means used up.

Sam

ple

Do Not

Dup

licat

e

Page 149: by GF Educators, Inc. Seventh Grade Reading Book … Sample Books/7th Reading Practice...Literary Text/Drama: Full length passage #2 ..... 54-58 Sample Reporting Category 2 Answer

145 © 2016 GF Educators, Inc.Seventh Grade Reading

work, dedication, and the tradition of manufacturing

pencils of the highest quality. As you walk through

the mill, you will meet some who have been there

for decades, and some who are just learning. What

they have in common is love for what they do and

the company for which they work.

1967 2016

Source: Used with permission from Henry Hulan III, President of the Musgrave Pencil Company.

Question 4The images below paragraph 6 are included in the selection most likely to –

C highlight the changes over time at the Musgrave factory

The dates and the machinery show how the factory has changed.

Sam

ple

Do Not

Dup

licat

e

Page 150: by GF Educators, Inc. Seventh Grade Reading Book … Sample Books/7th Reading Practice...Literary Text/Drama: Full length passage #2 ..... 54-58 Sample Reporting Category 2 Answer

146Seventh Grade Reading © 2016 GF Educators, Inc.

Exercise Reasoning5 The author organizes “The Musgrave Pencil Factory” by –

A identifying the reasons Colonel Musgrave built the factory

Wrong AnsWer: The selection tells the reasons at the beginning of the selection and not throughout it.

B providing information about the factory in chronological order

right AnsWer: The selection starts with WWI and moves through modern times.

C comparing the factory to other industries at the timer

Wrong AnsWer: Factories are not being compared.

D listing the steps in making a pencil Wrong AnsWer: Listing steps is in the second selection.

6 Which of these ideas is reinforced throughout the selection?

A Colonel Musgrave and his factory are a testament to devotion and hard work.

right AnsWer: The selection shows how Musgrave started a company, how he helped families during WWII, and that the company is still active today. This is devotion and hard work.

B The Musgrave Pencil Factory makes the best pencils.

Wrong AnsWer: This is not reinforced throughout the selection.

C Pencils provide the best way to make money in small town America.

Wrong AnsWer: Being in a small town is mentioned in the selection.

D World War I created the need to generate jobs in a slower economy.

Wrong AnsWer: Only the beginning of the selection mentions WWI.

7 Which sentence expresses an opinion?

A He turned to American manufacturers to start a company.

Wrong AnsWer: This is a fact that can be proven.

B In 1919 the Pencil Makers Association was organized.

Wrong AnsWer: This is a fact that can be proven.

C It is difficult to conceive how hard times were during the depression.

right AnsWer: The word difficult makes this sentence an opinion - difficulty is a matter of opinion.

D In the 1950s, Governor Buford Ellington named Shelbyville "The Pencil City."

Wrong AnsWer: This is a fact that can be proven.

Sam

ple

Do Not

Dup

licat

e

Page 151: by GF Educators, Inc. Seventh Grade Reading Book … Sample Books/7th Reading Practice...Literary Text/Drama: Full length passage #2 ..... 54-58 Sample Reporting Category 2 Answer

147 © 2016 GF Educators, Inc.Seventh Grade Reading

From Tree to Pencils

Have you ever wondered how a pencil is made? Read on to learn about the 10 step process that transforms a tree into a pile of pencils.

Ten Steps to a Perfect Pencil1. The process of making cedar pencils begins

at a sawmill, where cedar logs are cut into lumber called pencil stock. This lumber product is dried before being shipped to the slat factory. At the slat factory, pencil stock is cut into pencil blocks that are slightly longer than the normal length of a pencil. The small amount of extra length is called trim allowance.

2. Specially designed circular saws are used to cut pencil blocks into pencil slats. The saw blades are very thin in order to reduce the amount of sawdust waste produced. Due to the natural grain and defect characteristics of the wood, slats are sorted by width and grade for further processing. Full ply slats are without defects. To eliminate the defects and to produce a variety of usable grades and plies of pencil slats, the remaining slats are cut to smaller widths called narrow ply, or shorter lengths known as memos.

3. The pencil slats are treated with wax and stain to obtain uniform color. The waxing helps the slats pass through the machinery more easily and improves the pencils’ sharpening characteristics. After treatment, the slats are packaged and shipped to pencil factories all over the world.

4. At the pencil factory, a grooving machine cuts grooves into the slats to accept the writing core, or lead.

Question 8Which definition of uniform is used in Step 3 of the pencil making process?

D Definition 4 (without variation)

If you want uniform colors in pencils, you want them not to have any variation.

Question 11This sentence indicates that –

D the pencil making process takes place in multiple factories

Sending the slats to other factories shows that pencils are made in several factories.

Question 9According to the information in “Ten Steps to a Perfect Pencil,” how does waxing help during the pencil making process?

A Makes it easier for slats to pass through the machinery

The answer is in step 3.

Question 13According to the illustration, during the pencil making process –

C the writing cores are added before the pencil slats go to the shaper machine

The cores are in prior to step 7 – shaping.

Sam

ple

Do Not

Dup

licat

e

Page 152: by GF Educators, Inc. Seventh Grade Reading Book … Sample Books/7th Reading Practice...Literary Text/Drama: Full length passage #2 ..... 54-58 Sample Reporting Category 2 Answer

148Seventh Grade Reading © 2016 GF Educators, Inc.

5. Writing cores, made from a mixture of graphite and clay, are placed into the grooves. Colored pencils use wax-based cores, while other formulations are used in cosmetic pencils.

6. A second grooved slat is glued onto the first – making a sandwich – by a machine called a lead layer. The sandwiches are then clamped and held together tightly while the glue dries.

7. Once the glue dries, the sandwiches go to a shaper machine. They are trimmed to make sure the sandwich is square and that all the pencils will be the correct length. Then the sandwich is cut into pencil shapes, which can be round, triangular, or hexagonal.

8. Single pencils cut from the sandwich are ready to be processed. Any pencils with defects, such as uncentered leads or chipped wood, are discarded at this point.

9. Next, each pencil is painted in a machine. Pencils receive from 4-10 coats of lacquer, depending on the desired quality of the finish. After painting, some pencils are wrapped in decorative film or foils with fancy designs, although most pencils are imprinted with the brand name by stamping a foil logo into the surface of the pencil.

10. Using a tipping machine, an eraser and the metal ring that holds the eraser are crimped into place on each pencil.

Question 12From the information in “Ten Steps to a Perfect Pencil,” the reader can conclude that –

B writing cores can be made from different materials

Colored pencils use wax-based, other formulations are used for cosmetic pencils.

Question 10In “Ten Steps to a Perfect Pencil,” the layers of the pencil are glued together in –

B Step 6

Answer is in the step – layers of pencils are glued together.

Sam

ple

Do Not

Dup

licat

e

Page 153: by GF Educators, Inc. Seventh Grade Reading Book … Sample Books/7th Reading Practice...Literary Text/Drama: Full length passage #2 ..... 54-58 Sample Reporting Category 2 Answer

149 © 2016 GF Educators, Inc.Seventh Grade Reading

Pencil Trivia

• A pencil will write in space, upside down, and under water.

• A pencil can write 45,000 words.

• More than 2 billion pencils are sold in the U.S. each year.

• The average pencil can draw a line 35 miles long.

Source: Pencil Making Today: How to Make a Pencil in 10 Steps (www.pencils.com)

Exercise Reasoning

14 How are “The Musgrave Pencil Factory” and “From Tree to Pencils” different?

A “The Musgrave Pencil Factory” tells the chronological history of the factory, while “From Tree to Pencils” records the steps to making a pencil.

right AnsWer: This is exactly what is done in each selection – chronological history in “The Musgrave Pencil Factory” and steps in “From Tree to Pencils.”

B “The Musgrave Pencil Factory” discusses Colonel Musgrave and his idea, while “From Tree to Pencils” gives examples of how pencils are used.

Wrong AnsWer: Colonel Musgrave is not the only thing discussed in “The Musgrave Pencil Factory”, and examples are only a part of “From Tree to Pencils.”

C “The Musgrave Pencil Factory” focuses on jobs, while “From Tree to Pencils” focuses on factories.

Wrong AnsWer: Jobs are a part of “The Musgrave Pencil Factory”, and factories just a part of the steps are accomplished in “From Tree to Pencils.”

D “The Musgrave Pencil Factory” discusses the need for women in the workforce, while “From Tree to Pencils” focuses on the history of pencils.

Wrong AnsWer: Women working is a small part of “The Musgrave Pencil Factory”, and the history of pencils is not discussed in “From Tree to Pencils.”Sa

mpl

e

Do Not

Dup

licat

e

Page 154: by GF Educators, Inc. Seventh Grade Reading Book … Sample Books/7th Reading Practice...Literary Text/Drama: Full length passage #2 ..... 54-58 Sample Reporting Category 2 Answer

150Seventh Grade Reading © 2016 GF Educators, Inc.

Exercise Reasoning15 Both “The Musgrave Pencil Factory” and “From Tree to Pencil” provide –

A a how-to for crafting a pencil Wrong AnsWer: Only “From Tree to Pencils” has a how-to.

B thoughts about both WWI and WWII Wrong AnsWer: WWI and WWII are only included in “The Musgrave Pencil Factory.”

C historical information about pencil manufacturing

right AnsWer: “The Musgrave Pencil Factory” is historically written about pencil making, and “From Tree to Pencils” has a timeline as part of the selection.

D trivia about pencils Wrong AnsWer: Only “From Tree to Pencils” has pencil trivia.

16 From the information on the timeline and “The Musgrave Pencil Factory,” the reader can conclude that –

A the French sold Musgrave the patent to the pencil

Wrong AnsWer: Nothing in either selection states this.

B the English manufactured pencils before Musgrave

right AnsWer: Musgrave started in the early 20th century, and England started in 1832.

C colored pencils were invented for school children

Wrong AnsWer: There is nothing about why colored pencils were invented in either selection.

D the French created metallic pencils in 1795 Wrong AnsWer: The French invented the modern pencil in 1795; the metallic pencil was created in 1999.

17 Read this line from the “The Musgrave Pencil Factory” selection. The cedar rails were already dry and weathered. They were perfect to be cut into pencil slats at the mill in Shelbyville.Which sentences from the “Tree to Pencils” selection best matches an idea present in these lines?

A The process of making cedar pencils begins at a sawmill, where cedar logs are cut into lumber called pencil stock. This lumber product is then dried before being shipped to the slat factory.

right AnsWer: Both are about drying the cedar and the sawmill cutting into slats.

B At the slat factory, pencil stock is cut into pencil blocks that are slightly longer than the normal length of a pencil. The small amount of extra length is called trim allowance.

Wrong AnsWer: This sentence does not have anything about the cedar or cutting slats.

C The saw blades are very thin in order to reduce the amount of sawdust waste produced. Due to the natural grain and defect characteristics of the wood, slats are sorted by width and grade for further processing.

Wrong AnsWer: This sentence does not include drying or cutting the slats.

D Single pencils cut from the sandwich are ready to be processed. Any pencils with defects, such as uncentered leads or chipped wood, are discarded at this point.

Wrong AnsWer: This sentence does not have anything about the cedar or cutting slats.

Sam

ple

Do Not

Dup

licat

e

Page 155: by GF Educators, Inc. Seventh Grade Reading Book … Sample Books/7th Reading Practice...Literary Text/Drama: Full length passage #2 ..... 54-58 Sample Reporting Category 2 Answer

151 © 2016 GF Educators, Inc.Seventh Grade Reading

The Remarkable Jane Long

1 Jane Wilkinson Long is

often referred to as the

“Mother of Texas.” At the

age of twenty-one, Jane

Wilkinson Long crossed the

Sabine River into Texas in the

year 1819. At that time,

Texas was in Spanish

territory, which was rough

and sparsely populated.

Pirates inhabited the Texas coast while

angry Comanche watched their homelands

being taken over by settlers. Texas’ southern

neighbor, Mexico, had begun to grow uneasy under

the rule of Spain, and Mexican rebels were plotting

a revolution. Groups of Texans had organized an

invasion to free Texas from Spanish rule. To say

the least, the political climate was volatile,

ready to explode at any minute. According to

most people of that day, Texas was no place for a

woman.

2 Jane Long’s husband, Dr. James Long, was

a leader in Texas’ fight for freedom. He led an

army of three hundred men in an expedition from

Natchez to Nacogdoches, Texas, where Long and

his men proclaimed the independence of Texas and

set up a government. After the birth of their

second child, Jane set out for Nacogdoches to

join her husband. She and her escorts traveled

over dirt roads and through rain-swollen streams,

Informational Text/Paired Passages Set #6: Two Historical Accounts Read the selections and choose the best answer to each question.

Question 6The reader can infer that when Jane Long first traveled to Texas, she felt –

C frightened

The paragraph has information about Texas being “rough,” pirates were on the coast, the Comanche were angry, and the political climate was volatile. This would frighten her.

Question 1Which of the following events happened to Jane immediately before she set out to join her husband in Nacogdoches?

C Jane gave birth to her second child.

After the birth of her son, Jane headed out to Nacogdoches.

Sam

ple

Do Not

Dup

licat

e

Page 156: by GF Educators, Inc. Seventh Grade Reading Book … Sample Books/7th Reading Practice...Literary Text/Drama: Full length passage #2 ..... 54-58 Sample Reporting Category 2 Answer

152Seventh Grade Reading © 2016 GF Educators, Inc.

arriving in Nacogdoches in October of 1819.

3 In December 1820, James moved his family to

Bolivar Point on Galveston Bay. Then in September

1821, James headed off to Mexico with his army,

leaving a pregnant Jane, their daughter, and

Kian, her servant girl. Only a handful of soldiers

remained to protect the residents of Bolivar Point.

Jane gave birth to her daughter, alleged to be the

first Anglo-American child born in Texas, giving

Jane her famous title. Fearful of Indian attacks,

all but Jane, her two daughters, and Kian

evacuated the fort. Jane remained, saying, “My

husband left me here to wait for him, and I shall

stay until he returns.” Jane and Kian spent almost

a year alone at Galveston Bay.

4 The Karankawa Indians, which occupied

nearby Galveston Island, were known for being

warlike. Kian and Jane kept a close watch on the

campfires of these Indians. One day Kian spotted

a Karankawa canoe coming toward Bolivar Point.

She pointed the one cannon they had toward the

Indians while Jane used her red petticoat as a

warning flag flying over the fort. Jane loaded and

fired the cannon, and the Indians turned back. The

women left the red flag flying over Fort Bolivar, and

Jane occasionally fired the cannon to frighten the

Indians. After this incident, Jane and Kian wore

soldiers’ uniforms when they went to the beaches

to gather oysters.

5 In 1822, Jane received word that her husband,

who had been captured and held prisoner in

Mexico, had been killed in Mexico City. Jane then

became the sole support of her family. While many

women in such a position would have returned

to the home of their parents, Jane Long chose to

remain in Texas. She and her children set out for

Question 4What caused the soldiers to abandon Jane Long at the fort in Bolivar Point?

A The fear of Indian attacks drove them off.

Only a handful of soldiers stayed to protect the residents of Bolivar Point. But there was a fear of attacks, so everyone but Jane, her daughters, and Kian left.

Sam

ple

Do Not

Dup

licat

e

Page 157: by GF Educators, Inc. Seventh Grade Reading Book … Sample Books/7th Reading Practice...Literary Text/Drama: Full length passage #2 ..... 54-58 Sample Reporting Category 2 Answer

153 © 2016 GF Educators, Inc.Seventh Grade Reading

San Antonio in the company of the James Jones

family. Jane then moved on to Stephen F. Austin’s

colony.

6 In 1831, Jane settled in Brazoria and opened

a boarding house. Jane Long’s boarding house

became famous throughout Stephen F. Austin’s

colony, and many early Texas leaders were

guests there. This boarding house became

the headquarters for planning Texas’ battle for

independence from Mexico. Jane Long later moved

to Richmond and opened a new hotel. Jane grew

wealthy there, and she became the owner of

several town lots, horses, and many acres of land.

Mrs. Long remained in Richmond until her death on

December 30, 1880. She spent sixty-one years of

her life in Texas, watching it grow from a wilderness

to a state in the United States.

Exercise Reasoning2 Which sentence expresses the controlling idea of the selection?

A Jane Long played an important role in the early settlement of Texas.

right AnsWer: Jane had much to do with Texas being settled, from supporting her husband to opening a boarding house.

B Jane Long became wealthy when she lived in Richmond.

Wrong AnsWer: She became wealthy in Brazoria.

C Early Texas was no place for a woman. Wrong AnsWer: This is not the controlling idea. She proved this wrong.

D James Long was a leader in the Texas fight for freedom.

Wrong AnsWer: The selection is not only about James Long.Sa

mpl

e

Do Not

Dup

licat

e

Page 158: by GF Educators, Inc. Seventh Grade Reading Book … Sample Books/7th Reading Practice...Literary Text/Drama: Full length passage #2 ..... 54-58 Sample Reporting Category 2 Answer

154Seventh Grade Reading © 2016 GF Educators, Inc.

Exercise Reasoning3 Which sentence expresses an opinion?

A Jane Wilkinson Long is often referred to as the “Mother of Texas.”

Wrong AnsWer: This is a fact that can be proven.

B According to most people of that day, Texas was no place for a woman.

right AnsWer: "Most people" makes this sentence an opinion.

C Fearful of Indian attack, all but Jane and Kian evacuated the fort.

Wrong AnsWer: This is a fact that can be proven.

D In 1831, Jane settled in Brazoria and opened a boarding house.

Wrong AnsWer: This is a fact that can be proven.

5 “Mother of Texas” is an appropriate name for Jane Long because she -

A ran a boarding house in Brazoria right AnsWer: Many Texas patriots gathered in Jane Long's boarding house and the plan for Texas independence was "born" there.

B gave birth to her child at Bolivar Point Wrong AnsWer: Many women had children in Texas. This would not make her the mother of Texas.

C left her parents to live in Texas Wrong AnsWer: Many other people left their parents to come to Texas.This would not make her the mother of Texas.

D became the sole support of her family Wrong AnsWer: This is specific to her family and not to Texas. It would not make her the mother of Texas.

7 Read these sentences from the selection.

She and her escorts traveled over dirt roads and through rain-swollen streams, arriving in Nacogdoches in October of 1819. (paragraph 2)

While many women in such a position would have returned to the home of their parents, Jane Long chose to remain in Texas. (paragraph 5)

Based on these sentences, the reader can infer that Jane was –

A stronger than most women of that time right AnsWer: Traveling over rough roads and not returning to her parents makes her stronger than most women.

B forced to stay in Texas Wrong AnsWer: She was not forced to stay in Texas.

C disappointed with her life in Texas Wrong AnsWer: Nothing about disappointment is in the sentences.

D cut off from her parents Wrong AnsWer: It does not say she was cut off from her parents.

Sam

ple

Do Not

Dup

licat

e

Page 159: by GF Educators, Inc. Seventh Grade Reading Book … Sample Books/7th Reading Practice...Literary Text/Drama: Full length passage #2 ..... 54-58 Sample Reporting Category 2 Answer

155 © 2016 GF Educators, Inc.Seventh Grade Reading

Removal to TexasAn excerpt from Chapter 3 of Six Decades in

Texas, The Memoirs of Francis Richard Lubbock

1 It was not long before

my Creole wife decided to

come with me to Texas. A

few days before Christmas

we accordingly embarked on

the schooner Corolla, bound

for Quintana, which port we

reached in good time after a

stormy voyage.

2 This [selling of captured

cargo] was my first labor for the government of

Texas, soon after I had come to make my home

in Texas, in December 1836. Velasco was then the

prospective seaport and commercial emporium

of the young republic that was cradled in the rich

valley of the Brazos, and now again after half a

century she lifts her head with the buoyant hope of

success.

3 This work at Velasco accomplished, to make

my wife comfortable, we went up to Brazoria and

boarded with Mrs. Jane Long, the widow of General

James Long. He invaded Texas with about 300 men;

taking Nacogdoches in 1819, but after a series of

misfortunes was captured at Goliad in 1821 and

taken off to Mexico. His faithful wife, left at Bolivar

Point, near Galveston Island, and deserted by all

but a servant girl, remained at her post during the

succeeding winter, vainly expecting the return of

General Long, who was murdered in the City of

Mexico. To keep off the Karanchua Indians, Mrs.

Question 8Which definition of Creole is used in paragraph 1 of the selection?

A Definition 1 (of French and African ancestry)

This is the definition that describes Lubbock’s wife – French and African ancestry.

Question 9In paragraph 2, the word rich means –

A fertile

A valley cannot be wealthy, powerful, or prosperous. Lubbock was talking about the fertile soils.

Sam

ple

Do Not

Dup

licat

e

Page 160: by GF Educators, Inc. Seventh Grade Reading Book … Sample Books/7th Reading Practice...Literary Text/Drama: Full length passage #2 ..... 54-58 Sample Reporting Category 2 Answer

156Seventh Grade Reading © 2016 GF Educators, Inc.

Long herself frequently fired off the cannon at

the fort. She was rescued the next year by some

of Austin's colonists. Mrs. Long's career had in

it a touch of romance very rare even in the

Southwest. She was a sensible, strong-minded

woman, and she highly entertained us with the

recital of her thrilling adventures.

4 We made some distinguished acquaintances

at Mrs. Long's — among others, Judge Ben C.

Franklin and General Lamar, then Vice-President

of the Republic, in the prime of life and the halo of

his glory won at San Jacinto. He was a man of the

French type, five feet seven or eight inches high,

with a dark complexion, black, long hair, inclined

to curl, and gray eyes. Lamar was peculiar in

his dress; wore his clothes very loose, his

pants being of that old style, very baggy,

and with large pleats, looking odd, as he was

the only person I ever saw in Texas in that

style of dress. I found the Vice-President

rather reserved in conversation; it was said,

however, that he was quite companionable

with his intimate friends. He had proved his

soldiership at San Jacinto — he was now trying the

role of statesman. One of the guests at Mrs. Long's

was telling a fishy story, with extra embellishments,

when Lamar dryly remarked: "I have known men

to add a little fiction to their stories to make them

interesting, but this fellow lies without metes,

bounds, or landmarks."

5 Shortly after this, coming from Quintana to

Brazoria on horseback, I was belated, got lost,

and had to spend the night in the Brazos River

bottom. The darkness was made hideous

by the yelping of wolves, the cries of the

Question 12Lubbock includes this sentence most likely to show –

B that Long had lived an extraordinary life full of allure

Her strong character and thrilling adventures made her life “romantic.”

Question 10The imagery in paragraph 4 suggests that –

C General Lamar was a man of unusual mannerisms

He dressed in a peculiar way, was reserved except with friends, and called a man a liar.

Sam

ple

Do Not

Dup

licat

e

Page 161: by GF Educators, Inc. Seventh Grade Reading Book … Sample Books/7th Reading Practice...Literary Text/Drama: Full length passage #2 ..... 54-58 Sample Reporting Category 2 Answer

157 © 2016 GF Educators, Inc.Seventh Grade Reading

Question 11How does the Lubbock reveal the reason for all his mosquito bites?

D By describing the experience of getting lost in the Brazos River bottom

He tells about the noises he hears: wolves, the panthers, and the mosquitoes.

Mexican panther, and the never-ending hum

of mosquitos. Being green from the States,

I almost despaired of life, while anxiously

waiting the issue. The welcome morning

brought me deliverance, but on my arrival

at the boarding house, my face appeared so

disfigured by mosquito bites that my wife

scarcely recognized me. This horrible night's

experience in the Brazos bottom six decades ago is

still distinct in my memory.

Source: Six Decades in Texas, The Memoirs of Francis Richard Lubbock, edited by C.W. Raines (Austin: Ben C Jones & Co., 1900)

Exercise Reasoning13 What lesson does Lubbock learn while spending time in the Brazos bottom?

A Enjoy your comfortable life while you can. Wrong AnsWer: He only spent a short time there, which would not make this a lesson learned.

B Be careful not to cry, “Wolf!” Wrong AnsWer: He heard wolves cry; he did not cry wolf.

C People can be out of place in nature. right AnsWer: Lubbock did not know enough about nature, so he as out of place in it.

D Persistence pays off. Wrong AnsWer: One night did not make him persistent.

14 How does the author’s purpose in “The Remarkable Jane Long” differ from Lubbock’s observations about Long in “Removal to Texas”?

A The author communicates by sharing stories, while Lubbock communicates by relating history.

Wrong AnsWer: These are opposite of what they should be. “Removal to Texas” is stories, and “The Remarkable Jane Long” was relating history.

B The author reveals the important moments in Long’s life, while Lubbock highlights the idealism of Long’s life.

right AnsWer: “The Remarkable Jane Long” highlights her life in Texas, and “Removal to Texas” mentions the romanticism of her life.

C The author downplays everything Long accomplished, while Lubbock tells about Long’s guests at the boarding house.

Wrong AnsWer: The author speaks highly of Jane Long and does not downplay her accomplishments. Lubbock's description of Long's guests is not an observation about Jane Long.

D The author compares Long to other famous Texas women, while Lubbock compares her to his wife.

Wrong AnsWer: No other Texas women are in “The Remarkable Jane Long,” and Lubbock does not compare his wife to Long in “Removal to Texas.”

Sam

ple

Do Not

Dup

licat

e

Page 162: by GF Educators, Inc. Seventh Grade Reading Book … Sample Books/7th Reading Practice...Literary Text/Drama: Full length passage #2 ..... 54-58 Sample Reporting Category 2 Answer

158Seventh Grade Reading © 2016 GF Educators, Inc.

Exercise Reasoning15 Both Long and Lubbock were interested in –

A making Texas great right AnsWer: Both were interested in making Texas great, and they did make Texas great.

B providing places for people to rest Wrong AnsWer: Only Long opened a boarding house. -- a place for people to rest.

C becoming wealthy Wrong AnsWer: Even though Jane Long did become wealthy, neither text mentions that they had a goal of becoming wealthy.

D risking everything to join loved ones Wrong AnsWer: Only Long risked everything to join her husband.

16 How is Jane Long presented differently in “Removal to Texas” and “The Remarkable Jane Long?"

A “Removal to Texas” focuses on Jane Long’s boarding house.

Wrong AnsWer: It doesn’t only focus on her boarding house.

B “Removal to Texas” describes events that happen over a longer period.

Wrong AnsWer: This is actually a shorter time frame.

C “Removal to Texas” contains a firsthand account of encounters with Jane Long.

right AnsWer: “Removal to Texas” is a memoir, so it is a firsthand account.

D “Removal to Texas” shows how a governor offers help to Jane Long .

Wrong AnsWer: A governor does not offer Long help.

Sam

ple

Do Not

Dup

licat

e

Page 163: by GF Educators, Inc. Seventh Grade Reading Book … Sample Books/7th Reading Practice...Literary Text/Drama: Full length passage #2 ..... 54-58 Sample Reporting Category 2 Answer

159 © 2016 GF Educators, Inc.Seventh Grade Reading

Seventh Grade Reading Answer Key

The Three Admonitions/ The Prince Who Acquired Wisdom PageS 177-1841 B2 D3 A4 C5 B6 B7 D8 C9 A10 B11 C12 D13 D14 C

A Naughty Little Comet/Science in the Middle AgesPages 185-1901 A2 A3 B4 C5 D6 C7 A8 D9 C10 B11 B12 A13 C14 C

Benjamin Franklin: My Story/Books of His BoyhoodPages 191-1991 A2 C3 B4 D5 B6 B7 A8 D9 B10 B11 A12 D13 B14 D

The Mayo Family of Doctors/Alexander Graham BellPage 200-2071 A2 B3 C4 D5 C6 B7 D8 B9 C10 B11 C12 A13 C14 B15 C16 A

The Musgrave Pencil Factory/From Trees to PencilsPages 208-2151 A2 D3 D4 C5 B6 A7 C8 D9 A10 B11 D12 B13 C14 A15 C16 B17 A

The Remarkable Jane Long/Removal to TexasPages 216-2231 C2 A3 B4 A5 A6 C7 A8 A9 A10 C11 D12 B13 C14 B15 A16 C

*All page numbers refer to the Student Edition.

Reporting Category 1 Paired Passages

Sam

ple

Do Not

Dup

licat

e